SlideShare a Scribd company logo
1 of 73
Download to read offline
ISBN:
Copyright Notice:

Lecture Notes on
Differential Equations

Emre Sermutlu
˙
To my wife Nurten and my daughters Ilayda and Alara
vi

CONTENTS
5.2

Differential Operators . . . . . . . . . . . . . . . . . . . . . . . 34

5.3

Homogeneous Equations . . . . . . . . . . . . . . . . . . . . . 35

5.4

Nonhomogeneous Equations . . . . . . . . . . . . . . . . . . . 37

6 Series Solutions

Contents

41

6.1
6.2

1 First Order ODE
1.1 Definitions . . . . . . . .
1.2 Mathematical Modeling
1.3 Separable Equations . .
1.4 Transformations . . . . .

ix

.
.
.
.

.
.
.
.

.
.
.
.

2 Exact Equations
2.1 Exact Equations . . . . . . .
2.2 Integrating Factors . . . . . .
2.3 Linear First Order Equations
2.4 Bernoulli Equation . . . . . .

.
.
.
.

.
.
.
.

.
.
.
.

.
.
.
.

.
.
.
.

.
.
.
.

.
.
.
.

.
.
.
.

.
.
.
.

.
.
.
.

.
.
.
.

.
.
.
.

.
.
.
.

.
.
.
.

.
.
.
.

.
.
.
.

.
.
.
.

.
.
.
.

.
.
.
.

.
.
.
.

.
.
.
.

.
.
.
.

.
.
.
.

.
.
.
.

.
.
.
.

.
.
.
.

.
.
.
.

.
.
.
.

.
.
.
.

.
.
.
.

.
.
.
.

.
.
.
.

.
.
.
.

.
.
.
.

.
.
.
.

1
1
3
3
5

.
.
.
.

9
9
11
13
14

Classification of Points . . . . . . . . . . . . . . . . . . . . . . 43

6.3
Preface

Power Series . . . . . . . . . . . . . . . . . . . . . . . . . . . . 41
Power Series Method . . . . . . . . . . . . . . . . . . . . . . . 43

7 Frobenius’ Method

49

7.1

An Extension of Power Series Method . . . . . . . . . . . . . . 49

7.2

Examples . . . . . . . . . . . . . . . . . . . . . . . . . . . . . 50

8 Laplace Transform I

57

8.1

Definition . . . . . . . . . . . . . . . . . . . . . . . . . . . . . 57

8.2

Properties . . . . . . . . . . . . . . . . . . . . . . . . . . . . . 59

8.3

Initial Value Problems . . . . . . . . . . . . . . . . . . . . . . 61

9 Laplace Transform II

69

.
.
.
.

.
.
.
.

.
.
.
.

.
.
.
.

.
.
.
.

.
.
.
.

.
.
.
.

.
.
.
.

.
.
.
.

.
.
.
.

.
.
.
.

.
.
.
.

.
.
.
.

.
.
.
.

.
.
.
.

.
.
.
.

.
.
.
.

.
.
.
.

17
17
19
20
22

4 Nonhomogeneous Equations
25
4.1 General and Particular Solutions . . . . . . . . . . . . . . . . 25
4.2 Method of Undetermined Coefficients . . . . . . . . . . . . . . 27
4.3 Method of Variation of Parameters . . . . . . . . . . . . . . . 29
5 Higher Order Equations
33
5.1 Linear Equations of Order n . . . . . . . . . . . . . . . . . . . 33
v

Convolution . . . . . . . . . . . . . . . . . . . . . . . . . . . . 69

9.2

Unit Step Function . . . . . . . . . . . . . . . . . . . . . . . . 72

9.3
3 Second Order Equations
3.1 Linear Differential Equations .
3.2 Reduction of Order . . . . . .
3.3 Constant Coefficients . . . . .
3.4 Cauchy-Euler Equation . . . .

9.1

Differentiation of Transforms . . . . . . . . . . . . . . . . . . . 73

9.4

Partial Fractions Expansion . . . . . . . . . . . . . . . . . . . 74

9.5

Applications . . . . . . . . . . . . . . . . . . . . . . . . . . . . 75

10 Fourier Analysis I

81

10.1 Fourier Series . . . . . . . . . . . . . . . . . . . . . . . . . . . 81
10.2 Convergence of Fourier Series . . . . . . . . . . . . . . . . . . 84
10.3 Parseval’s Identity . . . . . . . . . . . . . . . . . . . . . . . . 85
11 Fourier Analysis II

91

11.1 Fourier Cosine and Sine Series . . . . . . . . . . . . . . . . . . 91
11.2 Complex Fourier Series . . . . . . . . . . . . . . . . . . . . . . 94
11.3 Fourier Integral Representation . . . . . . . . . . . . . . . . . 96
CONTENTS

vii

12 Partial Differential Equations
12.1 Introduction . . . . . . . . . . . . . . . . . . . . . . . . . . .
12.2 Modeling a Vibrating String . . . . . . . . . . . . . . . . . .
12.3 Method of Separation of Variables . . . . . . . . . . . . . . .

101
. 101
. 103
. 104

13 Heat Equation
13.1 Modeling Heat Flow . . . . . . . .
13.2 Homogeneous Boundary Conditions
13.3 Nonzero Boundary Conditions . . .
13.4 Two Dimensional Problems . . . .

111
. 111
. 113
. 115
. 117

.
.
.
.

.
.
.
.

.
.
.
.

.
.
.
.

.
.
.
.

.
.
.
.

.
.
.
.

.
.
.
.

.
.
.
.

.
.
.
.

.
.
.
.

.
.
.
.

.
.
.
.

.
.
.
.

14 Laplace Equation
121
14.1 Rectangular Coordinates . . . . . . . . . . . . . . . . . . . . . 121
14.2 Polar Coordinates . . . . . . . . . . . . . . . . . . . . . . . . . 126
To the Student

133

References

135

Index

137
Preface
This set of lecture notes for ordinary and partial differential equations grew
out of the course Engineering Mathematics I have taught at Cankaya Univer¸
sity since 1999. It is a one-semester course for second year students. The main
audience for this text, of course, is students. Presentation is user-friendly.
There are more examples and fewer theorems than usual.
The material is based on a solid background in calculus. The student
is assumed to be familiar with algebra, trigonometry, functions and graphs,
series, differentiation, and most importantly, integration techniques of various kinds. It is my (and my students’) sad experience that if you cannot
differentiate and integrate, you cannot solve differential equations. Knowledge of Linear Algebra, except for the determinants of a simple nature, is
not assumed.
There are 14 chapters. Each chapter can be covered in one week. After
a summary of methods and solved exercises, there are a number of end of
chapter exercises with answers. The exercises that take exceptionally longer
times are marked with a star. ( ) Nobody can learn how to solve problems
by watching someone else solve problems. So I advise the students to try
each problem on their own.
I would like to thank all my students who helped me write this book
by the valuable feedback they provided. In particular, special thanks are for
˙
Nuh Co¸kun, Nevrez Imamo˜lu, Nilg¨n Din¸arslan and I¸ıl Lelo˜lu who have
s
g
u
c
s
g
made a very extensive and meticulous check of the whole manuscript.
You may send all kinds of comments, suggestions and error reports to
sermutlu@cankaya.edu.tr.
Assist. Prof. Dr. Emre Sermutlu
ix
2

CHAPTER 1. FIRST ORDER ODE

are partial differential equations. (Partial Differential Equations are usually
much more difficult)
Order: The order of a differential equation is the order of the highest derivative that occurs in the equation.
A first order differential equation contains y , y and x so it is of the form
F (x, y, y ) = 0 or y = f (x, y).
For example, the following differential equations are first order:

Chapter 1
First Order Differential
Equations

y + x2 y = ex
xy = (1 + y 2 )
y 2 = 4xy
While these are second order:

The subject of differential equations is an important part of applied mathematics. Many real life problems can be formulated as differential equations.
In this chapter we will first learn the basic concepts and classification of
differential equations, then we will see where they come from and how the
simplest ones are solved. The concepts and techniques of calculus, especially
integration, will be necessary to understand differential equations.

1.1

Definitions

Ordinary Differential Equation: An ordinary differential equation is an
equation that contains derivatives of an unknown function y(x).
Partial Differential Equation: A partial differential equation is a differential equation involving an unknown function of two or more variables, like
u(x, y).
For example,
y − 4y + y = 0
y 2 + 1 = x2 y + sin x
are ordinary differential equations.
uxx + uyy = 0
u2 + u2 = ln u
x
y
1

y − x2 y + y = 1 + sin x
y + 6yy = x3
General and Particular Solutions: A general solution of a differential
equation involves arbitrary constants. In a particular solution, these constants are determined using initial values.
As an example, consider the differential equation y = 2x.
y = x2 + c is a general solution ,
y = x2 + 4 is a particular solution .
Example 1.1 Find the general solution of the differential equation y = 0.
Then find the particular solution that satisfies y(0) = 5, y (0) = 3.
y = 0 ⇒ y = c ⇒ y = cx + d. This is the general solution.
y (0) = 3 ⇒ c = 3, y(0) = 5 ⇒ d = 5
Therefore y = 3x + 5. This is the particular solution.
Explicit and Implicit Solutions: y = f (x) is an explicit solution,
F (x, y) = 0 is an implicit solution. We have to solve equations to obtain y
for a given x in implicit solutions, whereas it is straightforward for explicit
solutions.
For example, y = e4x is an explicit solution of the equation y = 4y.
x3 + y 3 = 1 is an implicit solution of the equation y 2 y + x2 = 0
1.2. MATHEMATICAL MODELING

1.2

3

Mathematical Modeling

4

CHAPTER 1. FIRST ORDER ODE

Example 1.4 Solve the initial value problem y + y 2 xex = 0, y(0) = 2

Differential equations are the natural tools to formulate, solve and understand many engineering and scientific systems. The mathematical models of
most of the simple systems are differential equations.

y = −y 2 xex

dP
= αP
dt

ex dx

Example 1.3 The downward acceleration of an object in free fall is g. Find
the height as a function of time if the initial height is y0 and initial speed is
v0 .

2=

d2 y
= −g
dt2

1
= xex − ex + c
y

⇒

c=

1
xex − ex +

3
2

3
2

Example 1.5 Find the general solution of the differential equation
y + y 2 = 1.
dy
+ y2 = 1
dx

1
y = − gt2 + v0 t + y0
2

⇒

dy
= 1 − y2
dx
dy
=
1 − y2

Separable Equations

If we can separate x and y in a first order differential equation and put them
to different sides as g(y)dy = f (x)dx, it is called a separable equation. We
can find the solution by integrating both sides. (Don’t forget the integration
constant!)
f (x)dx + c

⇒

xex

1
−1 + c
y=

dy
= −gt + v0
dt

g(y)dy =

xex dx

1
− ex + c
This is the general solution. Now we will use the condition y(0) = 2 to
determine the constant c.
y=

where P0 = P (0)

1.3

dy
= xex dx
y2

Using integration by parts, we have u = x, dv = ex dx, du = dx, v = ex
therefore
1
= xex −
y

P = P0 eαt

−

dy
=
y2

−
Example 1.2 The rate of growth of a population is proportional to itself.
Find the population as a function of time.

⇒

(1.1)

1
2

1
1
+
1−y 1+y

⇒

dy
= dx
1 − y2

dx

dy =

1
1+y
ln
=x+c
2
1−y
1+y
= e2x+2c
1−y

dx
1.4. TRANSFORMATIONS

5

6

CHAPTER 1. FIRST ORDER ODE

After some algebra, we obtain
y=

u3/2
= 3 ln x + c
3/2

ke2x − 1
ke2x + 1

where k = e2c

u=

Example 1.6 Solve the initial value problem y = x3 e−y , y(1) = 0.
ey dy =
ey =

9
ln x + c1
2

e0 =

Let’s use the substitution u = x + y. Then,

1
+c
4

y = u − x, ⇒

1.4

x4 3
+
4
4

(u + 6)dx = (−u − 3)(du − dx)
3dx = (−u − 3)du

Sometimes a change of variables simplifies a differential equation just as
y
substitutions simplify integrals. For example if y = f
, the substitution
x
y
u = will make the new equation separable.
x
y
+3
x

x
.
y

√

u du =

3dx =

3x = −

3x = −

If y = ux, then y = u x + u and u x + u = u + 3
ux=3

dy = du − dx

and the equation can be expressed in terms of u and x.

Transformations

Example 1.7 Solve y =

2/3

Example 1.8 Solve the differential equation (x + y + 6)dx = (−x − y − 3)dy.

3
c=
4
y = ln

2/3

x3 dx

x4
+c
4

⇒

y(1) = 0

y=x

9
ln x + c1
2

1
u
3dx
x

1
u

This is an implicit solution.

(−u − 3) du
u2
− 3u + c
2

(x + y)2
− 3(x + y) + c
2
EXERCISES

7

Exercises

CHAPTER 1. FIRST ORDER ODE

Answers

Solve the following differential equations.
1) y 3 y + x3 = 0
2) y + 4x3 y 2 = 0
y
3) xy = x + y Hint: y = f
x
4) (x2 + y 2 ) dx + xydy = 0 Hint: y = f

8

y
x

1) x4 + y 4 = c
1
2) y = 4
x +c
3) y = x(ln |x| + c)
c
x2
4) y 2 = 2 −
x
2
5) y = − ln c +

2

5) y = xey−x
1 + ln x
6) y =
4y 3
7) y = 3x2 sec2 y
8) y = y(y + 1)
9) y + 2y = y 2 + 1
10) (1 + y 2 )dx + x2 dy = 0
y
x

12) y = eax+by
13) y = x2 y 2 − 2y 2 + x2 − 2
14) y = −

2x + y
x

Solve the following initial value problems:
15) (y 2 + 5xy + 9x2 )dx + x2 dy = 0, y(1) = −4
16) y 3 y + x3 = 0, y(0) = 1
17) y = −2xy, y(0) = 3
18) y = 1 + 4y 2 , y(0) = 0
19) (x2 + 1)1/2 y = xy 3 , y(0) = 2
20)

dx
x x2
= − , x(0) = 1
dt
5 25

2

6) y 4 = x ln x + c
7) 2y + sin 2y = 4x3 + c
ex
8) y =
c − ex
9) y = 1 −

1
x+c

10) y = tan c +
11) y = a

e−x
2

1
x

11) y = cxa
12)

eax e−by
+
=c
a
b

13) y = tan

x3
− 2x + c
3

14) y = −x +

c
x

15) y =
16)
17)
18)
19)

x
− 3x
ln x − 1

x4 + y 4 = 1
2
y = 3e−x
1
y = 2 tan 2x
√
y = ( 9 − 2 x2 + 1)−1/2
4

20) x =

5et/5
4 + et/5
10

CHAPTER 2. EXACT EQUATIONS

So, the solution of this equation is very simple, if du is zero, u must be a
constant, therefore
x4 + x2 y 2 + y 4 = c
∂N
∂M
=
is necessary and sufficient for the
∂y
∂x
equation M (x, y)dx + N (x, y)dy = 0 to be exact.
Method of Solution: To solve M dx + N dy = 0,
Theorem 2.1: The condition

Chapter 2

• Check for Exactness

Exact and Linear Differential
Equations

• If the equation is exact, find u by integrating either M or N .
u=

M dx + k(y) or u =

N dy + l(x)

Note that we have arbitrary functions as integration constants.
In this chapter, we will learn how to recognize and solve three different types
of equations: Exact, linear, and Bernoulli. All of them are first order equations, therefore we expect a single integration constant in the solution.
At this stage it seems like there’s a special trick for every different kind
of question. You will gain familiarity with exercise and experience.

• Determine the arbitrary functions using the original equation. The
solution is u(x, y) = c
Example 2.1 Solve the equation 3y 2 dx + (3y 2 + 6xy)dy = 0.
Let’s check for exactness first.

2.1

∂(3y 2 )
= 6y,
∂y

Exact Equations

A first order differential equation of the form
M (x, y)dx + N (x, y)dy = 0

The equation is exact.
(2.1)
u(x, y) =

is called an exact differential equation if there exists a function u(x, y) such
that
∂u
∂u
= M,
=N
(2.2)
∂x
∂y
In other words, du = M dx + N dy, so M dx + N dy is a total differential.
For example, the equation
(4x3 + 2xy 2 )dx + (4y 3 + 2x2 y)dy = 0
is exact, and
u = x4 + x2 y 2 + y 4
9

∂(3y 2 + 6xy)
= 6y
∂x

3y 2 dx + k(y) = 3y 2 x + k(y)

∂u
= 6yx + k (y) = 3y 2 + 6xy
∂y
k (y) = 3y 2

⇒

k(y) = y 3

We do not need an integration constant here because u(x, y) = c already
contains one
u(x, y) = 3y 2 x + y 3 = c
2.2. INTEGRATING FACTORS

2.2

11

Integrating Factors

12

CHAPTER 2. EXACT EQUATIONS

But this equation is more difficult than the one we started with. If we make
a simplifying assumption that F is a function of one variable only, we can
solve for F and obtain the following theorem:

Consider the equation

Theorem 2.2: Consider the equation P dx + Qdy = 0. Define
P dx + Qdy = 0

(2.3)

that is not exact. If it becomes exact after multiplying by F , i.e. if
F P dx + F Qdy = 0

R=
(2.4)

is exact, then F is called an integrating factor. (Note that P, Q and F are
functions of x and y)
1
For example, ydx − xdy = 0 is not exact, but F = 2 is an integrating
x
factor.
Example 2.2 Solve (2xex − y 2 )dx + 2ydy = 0. Use F = e−x .

a) If R depends only on x, then F (x) = exp
factor.
˜
b) If R depends only on y, then F (y) = exp
factor.

∂Q ∂P
−
∂x
∂y

R=

˜
R(y)dy

is an integrating

8x2 y + 2 − 6x2 y − 1
2x2 y + 1
1
= 3
=
3y + x
2x
2x y + x
x
F (x) = e

R(x)dx

= eln x = x

Multiply the equation by x to obtain the exact equation

∂(2x − y 2 e−x )
∂(2ye−x )
= −2ye−x ,
= −2ye−x
∂y
∂x

(4x3 y 2 + 2yx)dx + (2x4 y + x2 )dy = 0

Now the equation is exact. We can solve it as we did the previous example
and obtain the result
u(x, y) =

x2 + y 2 e−x = c
How To Find an Integrating Factor: Let P dx+Qdy = 0 be a differential
equation that is not exact, and let F = F (x, y) be an integrating factor. By
definition,
(2.5)

(2.6)

R(x) dx is an integrating

Example 2.3 Solve (4x2 y 2 + 2y)dx + (2x3 y + x)dy = 0

(2x − y 2 e−x )dx + 2ye−x dy = 0

Fy P + F P y = Fx Q + F Qx

1
˜
and R =
P

The equation is not exact.

The equation is not exact. Let’s multiply both sides by e−x . The new
equation is:

⇒

∂P
∂Q
−
∂y
∂x

∂(2x3 y + x)
∂(4x2 y 2 + 2y)
= 8x2 y + 2,
= 6x2 y + 1
∂y
∂x

∂(2y)
∂(2xex − y 2 )
= −2y,
=0
∂y
∂x

(F P )y = (F Q)x

1
Q

(4x3 y 2 + 2yx) dx + k(y) = x4 y 2 + yx2 + k(y)

∂u
= 2x4 y + x2 + k (y) = 2x4 y + x2
∂y
u(x, y) = x4 y 2 + x2 y = c

⇒

k(y) = 0
2.3. LINEAR FIRST ORDER EQUATIONS

2.3

13

Linear First Order Equations

2.4

If a first order differential equation can be written in the form
y + p(x)y = r(x)

CHAPTER 2. EXACT EQUATIONS

Bernoulli Equation

The equation
y + p(x)y = g(x)y a

(2.7)

it is called a linear differential equation. If r(x) = 0, the equation is homogeneous, otherwise it is nonhomogeneous.
We can express the equation (2.7) as [p(x)y − r(x)]dx + dy = 0. This is
not exact but it has an integrating factor:
R = p(x), F = e

14

p dx

is called Bernoulli equation. It is nonlinear. Nonlinear equations are usually
much more difficult than linear ones, but Bernoulli equation is an exception.
It can be linearized by the substitution
u(x) = [y(x)]1−a

(2.14)

(2.8)
Then, we can solve it as other linear equations.

Method of Solution:
• Given a first order linear equation, express it in the following form:
y + p(x)y = r(x)

p dx

y +e

p dx

py = re

2

p dx

y −

ex
2x
y=
3
3xy 2

p(x) dx to

obtain
e

Example 2.5 Solve the equation

(2.9)

• Multiply both sides by the integrating factor F (x) = exp

(2.10)

Here a = −2 therefore u = y 1−(−2) = y 3 ⇒ u = 3y 2 y
Multiplying both sides of the equation by 3y 2 we obtain
2

• Express the left hand side as a single parenthesis.
e

p dx

y

= re

p dx

3y 2 y − 2xy 3 =
(2.11)

y(x) = e−h

eh r dx + c

(2.12)

2

ex
x

⇒

u − 2xu =

e

−2x dx

= e−x

2

2

Multiplying both sides by e−x , we get
2

p dx.

2

e−x u − 2xe−x u =

Example 2.4 Solve y + 4y = 1
2

The integrating factor is F = e
equation by e4x to obtain

4 dx

(e−x u) =

4x

= e . Multiply both sides of the
2

e4x y + 4e4x y =e4x

e−x u = ln x + c

⇒

⇒

1
y = + ce−4x
4

1
x

1
x

u = (ln x + c)ex

(e4x y) =e4x
e4x
e4x y =
+c
4

ex
x

This equation is linear. Its integrating factor is

• Integrate both sides. Don’t forget the integration constant. The solution is:

where h =

(2.13)

y = (ln x + c)ex

2

1/3

2
EXERCISES

15

16

Exercises

Answers

Solve the following differential equations. (Find an integrating factor if
necessary)
1) (yex + xyex + 1)dx + xex dy = 0
2) (2r + 2 cos θ)dr − 2r sin θdθ = 0
3) (sin xy + xy cos xy)dx + (x2 cos xy)dy = 0
4) 2 cos ydx = sin ydy
5) 5dx − ey−x dy = 0
6) (2xy + 3x2 y 6 ) dx + (4x2 + 9x3 y 5 ) dy = 0
7) (3xey + 2y) dx + (x2 ey + x) dy = 0
1
5
8) y + y =
x
x
9) y +

1
1
y=
x ln x
ln x

c
− 1 e−x
x
r2 + 2r cos θ = c
x sin xy = c
F = e2x , e2x cos y = c
F = ex , 5ex − ey = c
F = y 3 , x 2 y 4 + x3 y 9 = c
F = x, x3 ey + x2 y = c
c
1
y= + 5
5 x

1) y =
2)
3)
4)
5)
6)
7)
8)

9) y =

x+c
ln x

10) y = −1 +

x4

12) y = 4 − 5e− 4

Reduce to linear form and solve the following equations:
2 sin x 1/2
13) y − 4y tan x =
y
cos3 x
x
y
5 ln x 4/5
25
y=
y
15) y +
x
x5

13) y =
14) y =

c − ln cos x
cos2 x
1
2

2

− x + ce−2x

15) y =

y
1
=− 9 3
x
xy

x ln x − x + c
x5

16) y =

14) y + y = −

1
c
+ 4
8
x
x

19) x = y −2
2

1/4

1
3

cosh 3y + c

Hint: x ↔ y
20) y =

20) 2xyy + (x − 1)y 2 = x2 ex

5

17) y = arcsin[c(x − 1)]
1
c
+ 3
18) F = y, x =
2y y

tan y
17) y =
x−1
18) y 2 dx + (3xy − 1)dy = 0
19) y (sinh 3y − 2xy) = y

c
cos x

11) y = x4 cos x + c cos x

10) y − y tan x = tan x
11) y + y tan x = 4x3 cos x
12) y + x3 y = 4x3 , y(0) = −1

16) y +

CHAPTER 2. EXACT EQUATIONS

Hint: z = y 2

cxe−x + 1 xex
2
18

Chapter 3
Second Order Homogeneous
Differential Equations
For first order equations, concepts from calculus and some extensions were
sufficient. Now we are starting second order equations and we will learn many
new ideas, like reduction of order, linear independence and superposition of
solutions.
Many differential equations in applied science and engineering are second
order and linear. If in addition they have constant coefficients, we can solve
them easily, as explained in this chapter and the next. For nonconstant
coefficients, we will have limited success.

3.1

Linear Differential Equations

If we can express a second order differential equation in the form
y + p(x)y + q(x)y = r(x)

(3.1)

it is called linear. Otherwise, it is nonlinear.
Consider a linear differential equation. If r(x) = 0 it is called homogeneous, otherwise it is called nonhomogeneous. Some examples are:
y + y 2 = x2 y
Nonlinear
sin xy + cos xy = 4 tan x Linear Nonhomogeneous
x2 y + y = 0
Linear Homogeneous
17

CHAPTER 3. SECOND ORDER EQUATIONS

Linear Combination: A linear combination of y1 , y2 is y = c1 y1 + c2 y2 .
Theorem 3.1: For a homogeneous linear differential equation any linear
combination of solutions is again a solution.
The above result does NOT hold for nonhomogeneous equations.
For example, both y = sin x and y = cos x are solutions to y + y = 0, so
is y = 2 sin x + 5 cos x.
Both y = sin x + x and y = cos x + x are solutions to y + y = x, but
y = sin x + cos x + 2x is not.
This is a very important property of linear homogeneous equations, called
superposition. It means we can multiply a solution by any number, or add
two solutions, and obtain a new solution.
Linear Independence: Two functions y1 , y2 are linearly independent if
c1 y1 + c2 y2 = 0 ⇒ c1 = 0, c2 = 0. Otherwise they are linearly dependent.
(One is a multiple of the other).
For example, ex and e2x are linearly independent. ex and 2ex are linearly
dependent.
General Solution and Basis: Given a second order, linear, homogeneous
differential equation, the general solution is:
y = c1 y1 + c2 y2

(3.2)

where y1 , y2 are linearly independent. The set {y1 , y2 } is called a basis, or a
fundamental set of the differential equation.
As an illustration, consider the equation x2 y − 5xy + 8y = 0. You can
easily check that y = x2 is a solution. (We will see how to find it in the
last section) Therefore 2x2 , 7x2 or −x2 are also solutions. But all these are
linearly dependent.
We expect a second, linearly independent solution, and this is y = x4 . A
combination of solutions is also a solution, so y = x2 + x4 or y = 10x2 − 5x4
are also solutions. Therefore the general solution is
y = c1 x 2 + c2 x 4
and the basis of solutions is {x2 , x4 }.

(3.3)
3.2. REDUCTION OF ORDER

3.2

19

Reduction of Order

3.3

If we know one solution of a second order homogeneous differential equation,
we can find the second solution by the method of reduction of order.
Consider the differential equation
y + py + qy = 0

(3.4)

Suppose one solution y1 is known, then set y2 = uy1 and insert in the equation. The result will be
y1 u + (2y1 + py1 )u + (y1 + py1 + qy1 )u = 0

(3.5)

(3.6)

This is still second order, but if we set w = u , we will obtain a first order
equation:
y1 w + (2y1 + py1 )w = 0
(3.7)
Solving this, we can find w, then u and then y2 .
Example 3.1 Given that y1 = x2 is a solution of

CHAPTER 3. SECOND ORDER EQUATIONS

Homogeneous Equations with Constant
Coefficients

Up to now we have studied the theoretical aspects of the solution of linear homogeneous differential equations. Now we will see how to solve the constant
coefficient equation y + ay + by = 0 in practice.
We have the sum of a function and its derivatives equal to zero, so the
derivatives must have the same form as the function. Therefore we expect
the function to be eλx . If we insert this in the equation, we obtain:
λ2 + aλ + b = 0

But y1 is a solution, so the last term is canceled. So we have
y1 u + (2y1 + py1 )u = 0

20

(3.8)

This is called the characteristic equation of the homogeneous differential
equation y + ay + by = 0.
If we solve the characteristic equation, we will see three different possibilities:
Two real roots, double real root and complex conjugate roots.
Two Real Roots: The general solution is
y = c1 eλ1 x + c2 eλ2 x

(3.9)

2

x y − 3xy + 4y = 0
Example 3.2 Solve y − 3y − 10y = 0
find a second linearly independent solution.
Let y2 = ux2 . Then
y2 = u x2 + 2xu
and
2

Try y = eλx . The characteristic equation is λ2 − 3λ − 10 = 0 with solution
λ1 = 5, λ = −2, so the general solution is
y = c1 e5x + c2 e−2x

y2 = u x + 4xu + 2u
Inserting these in the equation, we obtain
x4 u + x3 u = 0
If w = u then

1
x4 w + x3 w = 0 or w + w = 0
x
1
This linear first order equation gives w = , therefore u = ln x and
x
2
y2 = x ln x

Example 3.3 Solve the initial value problem y −y = 0, y(0) = 2, y (0) = 4
We start with y = eλx as usual. The characteristic equation is λ2 − 1 = 0.
Therefore λ = ±1. The general solution is: y = c1 ex + c2 e−x
Now, we have to use the initial values to determine the constants.
y(0) = 2 ⇒ c1 + c2 = 2 and y (0) = 4 ⇒ c1 − c2 = 4.
By solving this system, we obtain c1 = 3, c2 = −1 so the particular solution
is:
y = 3ex − e−x
3.3. CONSTANT COEFFICIENTS

21

Double Real Root: One solution is eλx but we know that a second order
equation must have two independent solutions. Let’s use the method of
reduction of order to find the second solution.
y − 2ay + a2 y = 0

⇒

y1 = eax

(3.10)

22

CHAPTER 3. SECOND ORDER EQUATIONS

3.4

Cauchy-Euler Equation

The equation x2 y + axy + by = 0 is called the Cauchy-Euler equation. By
inspection, we can easily see that the solution must be a power of x. Let’s
substitute y = xr in the equation and try to determine r. We will obtain

Let’s insert y2 = ueax in the equation.

r(r − 1)xr + arxr + bxr = 0

ax

ax

e u + (2a − 2a)e u = 0
Obviously, u = 0 therefore u = c1 + c2 x. The general solution is
y = c1 eλx + c2 xeλx

(3.12)

Example 3.4 Solve y + 2y + y = 0
λx

r2 + (a − 1)r + b = 0

(3.11)

(3.17)
(3.18)

This is called the auxiliary equation. Once again, we have three different
cases according to the types of roots. The general solution is given as follows:
• Two real roots

2

y = e . The characteristic equation is λ + 2λ + 1 = 0. Its solution is the
double root λ = −1, therefore the general solution is
y = c1 e−x + c2 xe−x

(3.13)

This can be proved using Taylor series expansions.
If the solution of the characteristic equation is
λ1 = α + iβ, λ2 = α − iβ

y = c1 e

(cos βx + i sin βx) + c2 e

αx

(cos βx − i sin βx)

(3.14)

(3.15)

By choosing new constants A, B, we can express this as
y=e

αx

(A cos βx + B sin βx)

y = c1 xr + c2 xr ln x

(3.20)

• Complex conjugate roots where r1 , r2 = r ± si
y = xr [c1 cos(s ln x) + c2 sin(s ln x)]

then the general solution of the differential equation will be
αx

(3.19)

• Double real root

Complex Conjugate Roots: We need the complex exponentials for this
case. Euler’s formula is
eix = cos x + i sin x

y = c1 xr1 + c2 xr2

(3.21)

Example 3.6 Solve x2 y + 2xy − 6y = 0
Insert y = xr . Auxiliary equation is r2 + r − 6 = 0. The roots are
r = 2, r = −3 therefore
y = c1 x2 + c2 x−3

(3.16)

Example 3.5 Solve y − 4y + 29y = 0.

Example 3.7 Solve x2 y − 9xy + 25y = 0

y = eλx . The characteristic equation is λ2 −4λ+29 = 0. Therefore λ = 2±5i.
The general solution is

Insert y = xr . Auxiliary equation is r2 − 10r + 25 = 0. Auxiliary equation
has the double root r = 5 therefore the general solution is

y = e2x (A cos 5x + B sin 5x)

y = c1 x5 + c2 x5 ln x
EXERCISES

23

24

Exercises

CHAPTER 3. SECOND ORDER EQUATIONS

Answers
1)
2)
3)
4)
5)

Are the following sets linearly independent?
1) {x4 , x8 }
2) {sin x, sin2 x}
3) {ln(x5 ), ln x}
Use reduction of order to find a second linearly independent solution:
4) x2 (ln x − 1) y − xy + y = 0,
y1 = x
1
5) x2 ln x y + (2x ln x − x)y − y = 0,
y1 =
x
6) y + 3 tan x y + (3 tan2 x + 1)y = 0,
y1 = cos x

Yes
Yes
No
y2 = ln x
y2 = ln x − 1

6) y2 = sin x cos x
7) y = (1 + x)e−x
1

8) y = c1 e−2x + c2 e− 2 x
Solve the following equations:
7) y + 2y + y = 0, y(0) = 1, y (0) = 0

9) y = e8x

5
8) y + y + y = 0
2

10) y = c1 e−12x + c2 xe−12x

9) y − 64y = 0, y(0) = 1,
10) y + 24y + 144y = 0

y (0) = 8

11) y = 4e−x + 3xe−x

7
11) y + 2y + y = 0, y(−1) = e, y(1) =
e
12) 5y − 8y + 5y = 0
π2
13) y + 2y + 1 +
y = 0, y(0) = 1, y (0) = −1
4
14) y − 2y + 2y = 0, y(π) = 0, y(−π) = 0
15) xy + y = 0
16) x2 y − 3xy + 5y = 0
17) x2 y − 10xy + 18y = 0
18) x2 y − 13xy + 49y = 0
19) Show that y1 = u and y2 = u
y −

v
u
+2
v
u

y +

12) y = e0.8x [A cos(0.6x) + B sin(0.6x)]
13) y = e−x cos
14) y = ex sin x
15) y = c1 + c2 ln x
16) y = x2 [c1 cos(ln x) + c2 sin(ln x)]

vdx are solutions of the equation
vu
u2 u
+2 2 −
vu
u
u

π
x
2

y=0

20) Show that y1 = u and y2 = v are solutions of the equation
(uv − vu )y + (vu − uv )y + (u v − v u )y = 0

17) y = c1 x2 + c2 x9
18) y = c1 x7 + c2 x7 ln x
26

CHAPTER 4. NONHOMOGENEOUS EQUATIONS

the nonhomogeneous one. The general solution is of the form
y = yh + yp

(4.3)

Example 4.1 Find the general solution of y − 3y + 2y = 2x − 3 using
yp = x.

Chapter 4

Let’s solve y − 3y + 2y = 0 first. Let yh = eλx . Then
λ2 − 3λ + 2 = 0

Second Order Nonhomogeneous
Equations

which means λ = 2 or λ = 1. The homogenous solution is
yh = c1 ex + c2 e2x
therefore the general solution is:
y = x + c1 ex + c2 e2x

In this chapter we will start to solve the nonhomogeneous equations, and
see that we will need the homogeneous solutions we found in the previous
chapter.
Of the two methods we will learn, undetermined coefficients is simpler,
but it can be applied to a restricted class of problems. Variation of parameters
is more general but involves more calculations.

4.1

Consider the nonhomogeneous equation
(4.1)

Let yp be a solution of this equation. Now consider the corresponding homogeneous equation
y + p(x)y + q(x)y = 0

(4.2)

Let yh be the general solution of this one. If we add yh and yp , the result
will still be a solution for the nonhomogeneous equation, and it must be the
general solution because yh contains two arbitrary constants. This interesting
property means that we need the homogeneous equation when we are solving
25

The solution of y = 0 is simply yh = c1 x + c2 , therefore the general solution
must be
y = − cos x + c1 x + c2
As you can see, once we have a particular solution, the rest is straightforward, but how can we find yp for a given equation?
Example 4.3 Find a particular solution of the following differential equations. Try the suggested functions. (Success not guaranteed!)

General and Particular Solutions

y + p(x)y + q(x)y = r(x)

Example 4.2 Find the general solution of y = cos x using yp = − cos x.

a) y + y = ex ,
Try yp = Aex
b) y − y = ex ,
Try yp = Aex
c) y + 2y + y = x
Try yp = Ax + B
d) y + 2y = x
Try yp = Ax + B
e) y + 2y + y = 2 cos x Try yp = A cos x and yp = A cos x + B sin x
As you can see, some of the suggestions work and some do not.
yp is usually similar to r(x). We can summarize our findings as:
• Start with a set of functions that contains not only r(x), but also all
derivatives of r(x).
• If one of the terms of yp candidate occurs in yh , there is a problem.
4.2. METHOD OF UNDETERMINED COEFFICIENTS

4.2

27

Method of Undetermined Coefficients

28

CHAPTER 4. NONHOMOGENEOUS EQUATIONS

The homogeneous equation is
3y + 10y + 3y = 0

To solve the constant coefficient equation
d2 y
dy
+ a + by = r(x)
2
dx
dx

(4.4)

• Solve the corresponding homogeneous equation, find yh .
• Find a candidate for yp using the following table:
Term in r(x)

Choice for yp

xn
eax
cos bx or sin bx
xn eax
xn cos bx or xn sin bx

An xn + · · · + A1 x + A0
Aeax
A cos bx + B sin bx
(An xn + · · · + A1 x + A0 )eax
(An xn + · · · + A0 ) cos bx
+(Bn xn + · · · + B0 ) sin bx
eax cos bx or eax sin bx
Aeax cos bx + Beax sin bx
xn eax cos bx or xn eax sin bx
(An xn + · · · + A0 )eax cos bx
+(Bn xn + · · · + B0 )eax sin bx
(You don’t have to memorize the table. Just note that the choice
consists of r(x) and all its derivatives)
• If your choice for yp occurs in yh , you have to change it. Multiply it
by x if the solution corresponds to a single root, by x2 if it is a double
root.
• Find the constants in yp by inserting it in the equation.
• The general solution is y = yp + yh
Note that this method works only for constant coefficient equations, and
only when r(x) is relatively simple.

Its solution is
yh = c1 e−3x + c2 e−x/3
To find a particular solution, let’s try yp = Ax + B. Inserting this in the
equation, we obtain:
10A + 3Ax + 3B = 9x
Therefore, A = 3, B = −10. The particular solution is:
yp = 3x − 10
The general solution is:
y = c1 e−3x + c2 e−x/3 + 3x − 10
Example 4.5 Find the general solution of y − 4y + 4y = e2x
The solution of the associated homogeneous equation
y − 4y + 4y = 0
is
yh = c1 e2x + c2 xe2x
Our candidate for yp is yp = Ae2x . But this is already in the yh so we have
to change it. If we multiply by x, we will obtain Axe2x but this is also in yh .
Therefore we have to multiply by x2 . So our choice for yp is yp = Ax2 e2x .
Now we have to determine A by inserting in the equation.
yp = 2Ax2 e2x + 2Axe2x
yp = 4Ax2 e2x + 8Axe2x + 2Ae2x

Example 4.4 Find the general solution of the equation
3y + 10y + 3y = 9x

4Ax2 e2x + 8Axe2x + 2Ae2x − 4(2Ax2 e2x + 2Axe2x ) + 4Ax2 e2x = e2x
4.3. METHOD OF VARIATION OF PARAMETERS

2Ae

2x

=e

2x

⇒

29

CHAPTER 4. NONHOMOGENEOUS EQUATIONS

Therefore the particular solution is

1
1
A = , yp = x2 e2x
2
2

yp (x) = −y1

1
y = yh + yp = c1 e2x + c2 xe2x + x2 e2x
2

4.3

30

y1 r
dx
aW

(4.11)

e−x
Example 4.6 Find the general solution of y + 2y + y = √
x
yh = c1 e−x + c2 xe−x

Method of Variation of Parameters

Consider the linear second order nonhomogeneous differential equation
a(x)y + b(x)y + c(x)y = r(x)

y2 r
dx + y2
aW

W =
(4.5)

e−x
xe−x
−x
−x
−e
e − xe−x

yp = −e−x
If a(x), b(x) and c(x) are not constants, or if r(x) is not among the functions
given in the table, we can not use the method of undetermined coefficients. In
this case, the variation of parameters can be used if we know the homogeneous
solution.
Let yh = c1 y1 + c2 y2 be the solution of the associated homogeneous equation
a(x)y + b(x)y + c(x)y = 0
(4.6)
Let us express the particular solution as:

xe−x e−x
√ dx + xe−x
e−2x x

yp = −e−x

√
x dx + xe−x

= e−2x
e−x e−x
√ dx
e−2x x

1
√ dx
x

4
x3/2
x1/2
+ xe−x
= e−x x3/2
3/2
1/2
3
4
y = yh + yp = c1 e−x + c2 xe−x + e−x x3/2
3
2
Example 4.7 Find the general solution of x y − 5xy + 8y = x5
yp = −e−x

We can find the homogeneous solution of the Cauchy-Euler equation as:
yp = v1 (x)y1 + v2 (x)y2

(4.7)

yh = c1 x4 + c2 x2

There are two unknowns, so we may impose an extra condition. Let’s choose
v1 y1 + v2 y2 = 0 for simplicity. Inserting yp in the equation, we obtain
r
a
= 0

v1 y1 + v2 y2 =
v1 y1 + v2 y2

−y2 r
,
aW

v2 =

y1 r
aW

(4.8)

(4.9)

where W is the Wronskian
W =

y1 y2
y1 y2

= y1 y2 − y2 y1

x4 x 2
4x3 2x

= −2x5

Therefore the particular solution is

The solution to this linear system is
v1 =

W =

(4.10)

yp (x) = −x4
1
yp (x) = x4
2
1 5
yp (x) = x
3
The general solution is

x2 x5
dx + x2
x2 (−2x5 )
1
dx − x2 x2 dx
2

1
y = c1 x 4 + c2 x 2 + x 5
3

x4 x5
dx
x2 (−2x5 )
EXERCISES

31

Exercises
1)
2)
3)
4)
5)
6)

32

CHAPTER 4. NONHOMOGENEOUS EQUATIONS

Answers

Find the general solution of the following differential equations
y + 4y = x cos x
y − 18y + 81y = e9x
y = −4x cos 2x − 4 cos 2x − 8x sin 2x − 8 sin 2x
y + 3y − 18y = 9 sinh 3x
y + 16y = x2 + 2x
y − 2y + y = x2 ex

7) 2x2 y − xy + y =

1
x

1) y = c1 sin 2x + c2 cos 2x + 1 x cos x + 2 sin x
3
9
1
2) y = c1 e9x + c2 xe9x + x2 e9x
2
3) y = c1 + c2 x + x cos 2x + 3 cos 2x + 2x sin 2x + sin 2x
1
1
4) y = c1 e3x + c2 e−6x + e−3x + xe3x
4
2
1 2 1
1
5) y = c1 sin 4x + c2 cos 4x + x + x −
16
8
128
1
6) y = c1 ex + c2 xex + x4 ex
12
√
1
7) y = c1 x + c2 x +
6x

8) x y + xy − 4y = x ln x
9) y − 8y + 16y = 16x
10) y = x3
11) y + 7y + 12y = e2x + x

1
1
x2
8) y = c1 x2 + c2 x−2 + x2 ln2 x − x2 ln x +
8
16
64
1
9) y = c1 e4x + c2 xe4x + x +
2
x5
10) y =
+ c1 + c2 x
20

12) y + 12y + 36y = 100 cos 2x

11) y = c1 e−3x + c2 e−4x +

2

2

13) y + 9y = ex + cos 3x + 2 sin 3x

1
7
1 2x
e + x−
30
12
144

12) y = c1 e−6x + c2 xe−6x + 2 cos 2x +

3
sin 2x
2

1 x 1
1
e − x cos 3x + x sin 3x
10
3
6

14) y + 10y + 16y = e−2x

13) y = c1 cos 3x + c2 sin 3x +

15) y − 4y + 53y = (53x)2

1
14) y = c1 e−2x + c2 e−8x + xe−2x
6

16) y + y = (x2 + 1)e3x

15) y = e2x (c1 cos 7x + c2 sin 7x) + 53x2 + 8x −

17) y + y = csc x

16) y = e3x (0.1x2 − 0.12x + 0.152) + c1 sin x + c2 cos x
17) y = c1 sin x + c2 cos x − x cos x + sin x ln | sin x|
18) y = c1 sin x + c2 cos x − cos x ln | sec x + tan x| − sin x ln | csc x + cot x|

18) y + y = csc x sec x
19) y − 4y + 4y =

e2x ln x
x

20) y − 2y + y =

e2x
(ex + 1)2

19) y = c1 e2x + c2 xe2x + xe2x

74
53

(ln x)2
− ln x + 1
2

20) y = c1 ex + c2 xex + ex ln(1 + ex )
34

CHAPTER 5. HIGHER ORDER EQUATIONS

means that all the constants c1 , c2 , . . . , cn are zero, then this set of functions
is linearly independent. Otherwise, they are dependent.
For example, the functions x, x2 , x3 are linearly independent. The functions cos2 x, sin2 x, cos 2x are not.
Given n functions, we can check their linear dependence by calculating
the Wronskian. The Wronskian is defined as

Chapter 5
Higher Order Equations

W (y1 , y2 , . . . , yn ) =

y1
y1
.
.
.
(n−1)

y1
In this chapter, we will generalize our results about second order equations to
higher orders. The basic ideas are the same. We still need the homogeneous
solution to find the general nonhomogeneous solution. We will extend the two
methods, undetermined coefficients and variation of parameters, to higher
dimensions and this will naturally involve many more terms and constants
in the solution. We also need some new notation to express nth derivatives
easily.

...
...

yn
yn
.
.
.

(5.4)

(n−1)

. . . yn

and the functions are linearly dependent if and only if W = 0 at some point.

5.2

Differential Operators

We can denote differentiation with respect to x by the symbol D
Dy =

dy
=y,
dx

D2 y =

d2 y
=y
dx2

(5.5)

etc. A differential operator is

5.1
An n
form

Linear Equations of Order n
th

L = a0 Dn + a1 Dn−1 + · · · + an−1 D + an

order differential equation is called linear if it can be written in the

a0 (x)

dn y
dn−1 y
dy
+ a1 (x) n−1 + · · · + an−1 (x) + an (x)y = r(x)
n
dx
dx
dx

(5.1)

and nonlinear if it is not linear.(Note that a0 = 0)
If the coefficients a0 (x), a1 (x), . . . an (x) are continuous, then the equation has
exactly n linearly independent solutions. The general solution is
y = c1 y1 + c2 y2 + · · · + cn yn

(5.2)

We will only work with operators where coefficients are constant.
We can add, multiply, expand and factor constant coefficient differential operators using common rules of algebra. In this respect, they are like
polynomials. So, the following expressions are all equivalent:
(D − 2)(D − 3)y = (D − 3)(D − 2)y
= (D2 − 5D + 6)y
= y − 5y + 6y
Let’s apply some simple operators to selected functions:
(D − 2)ex = Dex − 2ex
= ex − 2ex = −ex

Linear Independence: If
c1 y1 + c2 y2 + · · · + cn yn = 0
33

(5.3)

(5.6)
5.3. HOMOGENEOUS EQUATIONS

35

36

CHAPTER 5. HIGHER ORDER EQUATIONS

(D − 2)e2x = De2x − 2e2x
= 2e2x − 2e2x = 0

Now we are in a position to solve very complicated-looking homogeneous
equations.

(D − 2)2 xe2x = (D − 2)(D − 2)xe2x
= (D − 2)(e2x + 2xe2x − 2xe2x )
= (D − 2)e2x = 0

Method of Solution:
• Express the given equation using operator notation (D notation).

(D2 − 4) sin(2x) = (D − 2)(D + 2) sin(2x)
= (D − 2)(2 cos(2x) + 2 sin(2x))
= −4 sin(2x) + 4 cos(2x) − 4 cos(2x) − 4 sin(2x)
= −8 sin 2x

• Factor the polynomial.
• Find the solution for each component.
• Add the components to obtain the general solution.

5.3

Homogeneous Equations

Example 5.1 Find the general solution of y (4) − 7y + y − 7y = 0.

Based on the examples in the previous section, we can easily see that:
The general solution of the equation (D − a)n y = 0 is
y = eax (c0 + c1 x + . . . + cn−1 xn−1 )

In operator notation, we have
(D4 − 7D3 + D2 − 7D)y = 0
(5.7)

Factoring this, we obtain

if a is real.
Some special cases are:

D(D − 7)(D2 + 1)y = 0

Dn y = 0 ⇒ y = c0 + c1 x + . . . + cn−1 xn−1
(D − a)y = 0 ⇒ y = eax
(D − a)2 y = 0 ⇒ y = c1 eax + c2 xeax

We know that
(5.8)

We can extend these results to the case of complex roots. If z = a + ib is a
root of the characteristic polynomial, then so is z = a − ib. (Why?)
Consider the equation
(D − a − ib)n (D − a + ib)n y = (D2 − 2aD + a2 + b2 )n y = 0

(5.9)

The solution is
y = eax cos bx(c0 + c1 x + . . . + cn−1 xn−1 )
+eax sin bx(k0 + k1 x + . . . + kn−1 xn−1 )

(5.10)

2

(D + b )y = 0

⇒

Therefore the general solution is
y = c1 + c2 e7x + c3 sin x + c4 cos x
Note that the equation is fourth order and the solution has four arbitrary
constants.

Example 5.2 Solve D3 (D − 2)(D − 3)2 (D2 + 4)y = 0.
Using the same method, we find:

A special case is obtained if a = 0.
2

Dy = 0 ⇒ y = c
(D − 7)y = 0 ⇒ y = ce7x
(D2 + 1)y = 0 ⇒ y = c1 sin x + c2 cos x

y = c1 cos bx + c2 sin bx

(5.11)

y = c1 + c2 x + c3 x2 + c4 e2x + c5 e3x + c6 e3x x + c7 cos 2x + c8 sin 2x
5.4. NONHOMOGENEOUS EQUATIONS

5.4

37

Nonhomogeneous Equations

38

CHAPTER 5. HIGHER ORDER EQUATIONS

Then we will proceed similarly to simplify the steps. Eventually, we will
obtain the system

In this section, we will generalize the methods of undetermined coefficients
and variation of parameters to nth order equations.
Undetermined Coefficients: Method of undetermined coefficients is the
same as given on page 27. We will use the same table, but this time the
modification rule is more general. It should be:
• In case one of the terms of yp occurs in yh , multiply it by xk where k
is the smallest integer which will eliminate any duplication between yp
and yh .

v1 y1
v1 y1
.
.
.
(n−1)
v1 y1
(n)
v1 y1

yp = y1

The homogeneous solution is yh = (c0 + c1 x + c2 x2 + c3 x3 )ex . According to
the table, we should choose yp as Aex + Bxex , but this already occurs in
the homogeneous solution. Multiplying by x, x2 , x3 are not enough, so, we
should multiply by x4 .
yp = Ax4 ex + Bx5 ex
Inserting this in the equation, we obtain:
24Aex + 120Bxex = xex

1 5 x
xe
120

n−1

d y
d y
dy
+ a1 (x) n−1 + · · · + an−1 (x) + an (x)y = r(x)
n
dx
dx
dx

(5.12)

Let the homogeneous solution be yh = c1 y1 + · · · + cn yn
Then the particular solution is yp = v1 y1 + · · · + vn yn
Here, vi are functions of x. Since we have n functions, we can impose n − 1
conditions on them. The first condition will be
v1 y1 + · · · + vn yn = 0

=
=

+
+

(n−1)
vn y n
(n)
vn yn

=
=

0
0
.
.
.

(5.14)

0
r(x)
a0 (x)

v1 dx + · · · + yn

vn dx

(5.15)

x3 y − 6x2 y + 15xy − 15y = 8x6
We can find the homogeneous solution yh = c1 x + c2 x3 + c3 x5 using our
method for Cauchy-Euler equations. Then, the particular solution will be
yp = xv1 + x3 v2 + x5 v3 . Using the above equations, we obtain the system
xv1 + x3 v2 + x5 v3 = 0
v1 + 3x2 v2 + 5x4 v3 = 0
6xv2 + 20x3 v3 = 8x3

Variation of Parameters: The idea is the same as in second order equations, but there are more unknowns to find and more integrals to evaluate.
Consider
a0 (x)

vn yn
vn yn
.
.
.

Example 5.4 Find the general solution of

Therefore A = 0, B = 1/120 and the general solution is

n

+ ···
+ ···

+
+

Then, we will solve this linear system to find vi , and integrate them to
obtain yp .

Example 5.3 Solve the equation (D − 1)4 y = xex .

y = (c0 + c1 x + c2 x2 + c3 x3 )ex +

+ ···
+ ···

(5.13)

The solution of this system is v1 = x4 , v2 = −2x2 , v3 = 1 therefore the
particular solution is
yp = x

x4 dx + x3

(−2x2 ) dx + x5

and the general solution is
y = c1 x + c2 x 3 + c3 x 5 +

8 6
x
15

dx =

8 6
x
15
EXERCISES

39

Exercises

CHAPTER 5. HIGHER ORDER EQUATIONS

Answers
1) y = c0 + c1 x + c2 x2 + c3 x3 + c4 x4

1) D5 y = 0

2) y = c1 ex + c2 xex + c3 x2 ex

2) (D − 1)3 y = 0
3) y − 4y + 13y = 0
4) (D − 2)2 (D + 3)3 y = 0
5) (D2 + 2)3 y = 0
d4 y
d2 y
+ 5 2 + 4y = 0
dx4
dx
7) (D2 + 9)2 (D2 − 9)2 y = 0

6)

4

40

3

2

dy
dy
dy
−2 3 +2 2 =0
4
dx
dx
dx
9) y − 3y + 12y − 10y = 0
8)

3) y = c1 e2x cos 3x + c2 e2x sin 3x + c3
4) y = c1 e2x + c2 xe2x + c3 e−3x + c4 xe−3x + c5 x2 e−3x
√
√
√
√
5) y = c1 cos 2x + c2 sin 2x + c3 x cos 2x + c4 x sin 2x
√
√
+ c5 x2 cos 2x + c6 x2 sin 2x
6) y = c1 cos 2x + c2 sin 2x + c3 cos x + c4 sin x
7) y = c1 e3x + c2 xe3x + c3 e−3x + c4 xe−3x + c5 cos 3x + c6 sin 3x
+ c7 x cos 3x + c8 x sin 3x
8) y = c1 + c2 x + c3 ex cos x + c4 ex sin x

10) (D2 + 2D + 17)2 y = 0

9) y = c1 ex + c2 ex cos 3x + c3 ex sin 3x

11) (D4 + 2D2 + 1)y = x2

10) y = c1 e−x sin 4x + c2 e−x cos 4x + c3 xe−x sin 4x + c4 xe−x cos 4x

12) (D3 + 2D2 − D − 2)y = 1 − 4x3

11) y = c1 cos x + c2 sin x + c3 x cos x + c4 x sin x + x2 − 4

√
√
√
13) (2D4 + 4D3 + 8D2 )y = 40e−x [ 3 sin( 3x) + 3 cos( 3x)]
14) (D3 − 4D2 + 5D − 2)y = 4 cos x + sin x
15) (D3 − 9D)y = 8xex

12) y = c1 ex + c2 e−x + c3 e−2x + 2x3 − 3x2 + 15x − 8
√
√
√
13) y = c1 + c2 x + c3 e−x cos 3x + c4 e−x sin 3x + 5xe−x cos 3x
14) y = c1 ex + c2 xex + c3 e2x + 0.2 cos x + 0.9 sin x
3
15) y = c1 + c2 e3x + c3 e−3x + ex − xex
4
42

CHAPTER 6. SERIES SOLUTIONS
term wise, i.e.
∞

(an ± bn )(x − x0 )n

f (x) ± g(x) =
n=0

• In the interval of convergence, the series can be multiplied or divided
to give another power series.

Chapter 6

∞

cn (x − x0 )n

f (x)g(x) =

Series Solutions

n=0

where
cn = a0 bn + a1 bn−1 + · · · + an b0
If none of the methods we have studied up to now works for a differential
equation, we may use power series. This is usually the only choice if the
solution cannot be expressed in terms of the elementary functions. (That
is, exponential, logarithmic, trigonometric and polynomial functions). If the
solution can be expressed as a power series, in other words, if it is analytic,
this method will work. But it takes time and patience to reach the solution.
Remember, we are dealing with infinitely many coefficients!

6.1

Power Series

• In the interval of convergence, derivatives and integrals of f (x) can be
found by term wise differentiation and integration, for example
∞

n an (x − x0 )n−1

f (x) = a1 + 2a2 (x − x0 ) + · · · =
n=1
(n)

(x
• The series ∞ f n! 0 ) (x − x0 )n is called the Taylor Series of the funcn=0
tion f (x). The function f (x) is called analytic if its Taylor series
converges.

Examples of some common power series are:
∞

Let’s remember some facts about the series

ex =

∞

an (x − x0 )n = a0 + a1 (x − x0 ) + a2 (x − x0 )2 + · · ·

n=0

(6.1)

n=0

∞

cos x =
n=0

from calculus.

∞

sin x =
• There is a nonnegative number ρ, called the radius of convergence, such
that the series converges absolutely for |x − x0 | < ρ and diverges for
|x − x0 | > ρ . The series defines a function f (x) = ∞ an (x − x0 )n
n=0
in its interval of convergence.

n=0

1
=
1−x

41

=1+x+

(−1)n x2n
2n!

=1−

x2
+ ···
2!

x2 x4
+
− ···
2!
4!

(−1)n x2n+1
x3 x5
=x−
+
− ···
(2n + 1)!
3!
5!

∞

xn

= 1 + x + x2 + · · ·

(−1)n+1 xn
n

=x−

n=0
∞

ln(1 + x) =
• In the interval of convergence, the series can be added or subtracted

xn
n!

n=1

x2 x3
+
− ···
2
3
6.2. CLASSIFICATION OF POINTS

6.2

43

Classification of Points

44

CHAPTER 6. SERIES SOLUTIONS

Example 6.1 Solve y + 2xy + 2y = 0 around x0 = 0.
First we should classify the point. Obviously, x = 0 is an ordinary point, so
we can use power series method.

Consider the equation
R(x)y + P (x)y + Q(x)y = 0

(6.2)

∞

∞
n

y=
If both of the functions

P (x)
,
R(x)

Q(x)
R(x)

an x , y =
n=0

(6.3)

are analytic at x = x0 , then the point x0 is an ordinary point. Otherwise, x0
is a singular point.
Suppose that x0 is a singular point of the above equation. If both of the
functions
Q(x)
P (x)
, (x − x0 )2
(6.4)
(x − x0 )
R(x)
R(x)
are analytic at x = x0 , then the point x0 is called a regular singular point.
Otherwise, x0 is an irregular singular point.
For example, the functions 1+x+x2 , sin x, ex (1+x4 ) cos x are all analytic
cos x 1 ex 1 + x2
at x = 0. But, the functions
, ,
,
are not.
x
x x
x3
We will use power series method around ordinary points and Frobenius’
method around regular singular points. We will not consider irregular singular points.

∞

nan x

n−1

n(n − 1)an xn−2

, y =

n=1

n=2

Inserting these in the equation, we obtain
∞

∞

n(n − 1)an x

n−2

∞

+ 2x

n=2

nan x

n−1

an x n = 0

+2

n=1

∞

n=0

∞

∞

n(n − 1)an xn−2 +
n=2

2nan xn +
n=1

2an xn = 0
n=0

To equate the powers of x, let us replace n by n + 2 in the first sigma.
(n → n + 2)
∞

∞

∞

n=1

n=0

2an xn = 0

2nan xn +

(n + 2)(n + 1)an+2 xn +

n=0

Now we can express the equation using a single sigma, but we should start
the index from n = 1. Therefore we have to write n = 0 terms separately.
∞

[(n + 2)(n + 1)an+2 + (2n + 2)an ] xn = 0

2a2 + 2a0 +
n=1

6.3

Power Series Method

If x0 is an ordinary point of the equation R(x)y + P (x)y + Q(x)y = 0, then
the general solution is
∞

an (x − x0 )n

y=

−2(n + 1)
−2
an =
an
(n + 2)(n + 1)
(n + 2)
This is called the recursion relation. Using it, we can find all the constants
in terms of a0 and a1 .
a2 = −a0 , an+2 =

(6.5)

2
1
a4 = − a2 = a0
4
2

2
1
a6 = − a4 = − a0
6
6

2
a3 = − a1 ,
3

2
4
a 5 = − a3 = a1
5
15

n=0

The coefficients an can be found by inserting y in the equation and setting
the coefficients of all powers to zero. Two coefficients (Usually a0 and a1 )
must be arbitrary, others must be defined in terms of them. We expect two
linearly independent solutions because the equation is second order linear.

We can find as many coefficients as we want in this way. Collecting them
together, the solution is :
1
1
y = a0 1 − x 2 + x 4 − x 6 + · · ·
2
6

2
4
+ a1 x − x 3 + x 5 + · · ·
3
15
6.3. POWER SERIES METHOD

45

In most applications, we want a solution close to 0, therefore we can neglect
the higher order terms of the series.
Remark: Sometimes we can express the solution in closed form (in terms
of elementary functions rather than an infinite summation) as in the next
example:
Example 6.2 Solve (x − 1)y + 2y = 0 around x0 = 0.
Once again, first we should classify the given point. The function
analytic at x = 0, therefore x = 0 is an ordinary point.
∞

∞

∞

an x n , y =

y=
n=0

2
is
x−1

nan xn−1 , y =
n=1

n(n − 1)an xn−2
n=2

Inserting these in the equation, we obtain
∞

(x − 1)

∞

n(n − 1)an x

n−2

nan xn−1 = 0

+2

n=2

n=1
∞

∞

∞

n(n − 1)an xn−2 +

n(n − 1)an xn−1 −

n=1

n=2

n=2

2nan xn−1 = 0

46

CHAPTER 6. SERIES SOLUTIONS

Exercises
Find the general solution of the following differential equations in the
form of series. Find solutions around the origin (use x0 = 0). Write the
solution in closed form if possible.
1) (1 − x2 )y − 2xy = 0
2) y + x4 y + 4x3 y = 0
3) (2 + x3 )y + 6x2 y + 6xy = 0
4) (1 + x2 )y − xy − 3y = 0
5) (1 + 2x2 )y + xy + 2y = 0
6) y − xy + ky = 0
7) (1 + x2 )y − 4xy + 6y = 0
8) (1 − 2x2 )y + (2x + 4x3 )y − (2 + 4x2 )y = 0
9) (1 + 8x2 )y − 16y = 0
10) y + x2 y = 0
The following equations give certain special functions that are very important in applications. Solve them for n = 1, 2, 3 around origin. Find
polynomial solutions only.

To equate the powers of x, let us replace n by n+1 in the second summation.
∞

∞

n(n − 1)an x

n−1

−

n=2

∞

(n + 1)nan+1 x

n−1

+

n=1

2nan x

n−1

=0

n=1

Now we can express the equation using a single sigma.

11)
12)
13)
14)

(1 − x2 )y − 2xy + n(n + 1)y = 0
y − 2xy + 2ny = 0
xy + (1 − x)y + ny = 0
(1 − x2 )y − xy + n2 y = 0

(Legendre’s Equation)
(Hermite’s Equation)
(Laguerre’s Equation)
(Chebyshev’s Equation)

∞

[(n(n − 1) + 2n)an − n(n + 1)an+1 ] xn−1 = 0

(−2a2 + 2a1 ) +
n=2

a2 = a1 , an+1 =

n2 − n + 2n
an for n
n(n + 1)

2

So the recursion relation is:
an+1 = an
All the coefficients are equal to a1 , except a0 . We have no information about
it, so it must be arbitrary. Therefore, the solution is:
y = a0 + a1 x + x 2 + x 3 + · · ·
x
y = a0 + a1
1−x

Solve the following initial value problems. Find the solution around the
point where initial conditions are given.

15)
16)
17)
18)

xy + (x + 1)y − 2y = 0,
y + 2xy − 4y = 0,
4y + 3xy − 6y = 0,
(x2 − 4x + 7)y + y = 0,

x0
x0
x0
x0

= −1,
= 0,
=0
=2

y(−1) = 1,
y(0) = 1,
y(0) = 4,
y(2) = 4,

y (−1) = 0
y (0) = 0
y (0) = 0
y (2) = 10

19) Find the recursion relation for (p + x2 )y + (1 − q − r)xy + qry = 0
around x = 0. (Here p, q, r are real numbers, p = 0)
20) Solve (1 + ax2 )y + bxy + cy = 0 around x0 = 0
EXERCISES

47

Answers
x3 x5
+
+ ···
3
5

n=3

x3 x6 x9
+
−
+ ···
2
4
8
a1 x
+
3
1+ x
2

3) y = a0 1 −
a0
3
1+ x
2

3
1
3
4) y = a0 1 + x2 + x4 − x6 + · · ·
2
8
16
2
2
5) y = a0 1 − x2 + x4 − x6 + · · ·
3
3
6) y = a0 1 −
+a1 x −

2
+ a1 x + x3
3

17
1
+ a1 x − x 3 + x 5 + · · ·
2
40

k − 1 3 (k − 1)(k − 3) 5 (k − 1)(k − 3)(k − 5) 7
x +
x −
x + ···
3!
5!
7!
x3
3

x4 x6
1+x +
+
+ ···
2
6
2

10) y = a0 1 −
⇒
⇒

n=3

⇒

x8
x4
+
+ ···
12 672

+ a1 x

y = a1 x
y = a0 (1 − 3x2 )
5
y = a1 (x − x3 )
3

+ a1 x −

⇒

14) n = 1
n=2

⇒
⇒
⇒

x5
x9
+
+ ···
20 1440

y = a1 x
y = a0 (1 − 2x2 )
2
y = a1 (x − x3 )
3
y = a0 (1 − x)
1
y = a0 (1 − 2x + x2 )
2
3
1
y = a0 (1 − 3x + x2 − x3 )
2
6
y = a1 x
y = a0 (1 − 2x2 )
4
y = a1 (x − x3 )
3

1
1
15) y = 1 − (x + 1)2 − (x + 1)3 − (x + 1)4 − · · ·
3
6
16) y = 1 + 2x2
17) y = 4 + 3x2
1
1
18) y = 4 1 − (x − 2)2 + (x − 2)4 + · · ·
6
72
1
7
+ 10 (x − 2) − (x − 2)3 +
(x − 2)5 + · · ·
18
1080
19) an+2 = −

8
64
9) y = a0 (1 + 8x2 ) + a1 x + x3 − x5 + · · ·
3
15

11) n = 1
n=2

OR

k 2 k(k − 2) 4 k(k − 2)(k − 4) 6
x +
x −
x + ···
2!
4!
6!

7) y = a0 (1 − 3x2 ) + a1 x −

8) y = a0

x4 x7 x10
+
−
+ ···
2
4
8

⇒

n=3

+ a1 x −

⇒

n=3

x5
x10
x15
+
−
+ ···
5
5 · 10 5 · 10 · 15
x11
x16
x6
+
−
+ ···
x−
6
6 · 11 6 · 11 · 16

⇒

n=2

1 1+x
ln
2 1−x

⇒
⇒

13) n = 1

OR y = a0 + a1

2) y = a0 1 −

y=

CHAPTER 6. SERIES SOLUTIONS

12) n = 1
n=2

1) y = a0 + a1 x +

+a1

48

(n − q)(n − r)
an
p(n + 2)(n + 1)

x4
x2
+ c(2a + 2b + c)
2
4!
x6
+ ···
− c(2a + 2b + c)(12a + 4b + c)
6!
x3
x5
+ a1 x − (b + c)
+ (b + c)(6a + 3b + c)
3!
5!
x7
−(b + c)(6a + 3b + c)(20a + 5b + c)
+ ···
7!

20) y = a0 1 − c
50

CHAPTER 7. FROBENIUS’ METHOD

Case 2 - Equal roots: A basis of solutions is
∞

∞

an x n ,

y 1 = xr

y2 = y1 ln x + xr

bn x n

(7.3)

n=1

n=0

Case 3 - Roots differing by an integer: A basis of solutions is

Chapter 7

∞

∞

an xn ,

y 1 = xr 1

bn x n

y2 = ky1 ln x + xr2

n=1

(7.4)

n=0

In this chapter, we will extend the methods of the previous chapter to regular
singular points. The calculations will be considerably longer, but the basic
ideas are the same. The classification of the given point is necessary to make
a choice of methods.

where r1 − r2 = N > 0 (r1 is the greater root) and k may or may not be zero.
In all three cases, there is at least one relatively simple solution of the
form y = xr ∞ an xn . The equation is second order, so there must be a
n=0
second linearly independent solution. In Cases 2 and 3, it may be difficult
to find the second solution. You may use the method of reduction of order.
This is convenient especially if y1 is simple enough. Alternatively, you may
use the above formulas directly, and determine bn one by one using the an
and the equation.

7.1

7.2

Frobenius’ Method

An Extension of Power Series Method

Suppose x0 is a regular singular point. For simplicity, assume x0 = 0. Then
p(x)
q(x)
the differential equation can be written as y +
y + 2 y = 0 where
x
x
p(x) and q(x) are analytic. We can try a solution of the form
∞

y = xr

an x n

(7.1)

n=0

The equation corresponding to the lowest power xr−2 , in other words
r(r − 1) + p0 r + q0 = 0 is called the indicial equation, where p0 = p(0), and
q0 = q(0). Now we can find r, insert it in the series formula, and proceed as
we did in the previous chapter.
We can classify the solutions according to the roots of the indicial equation.
Case 1 - Distinct roots not differing by an integer: A basis of solutions
is
∞
∞
y 1 = xr 1

an x n ,

y2 = xr2

bn x n
n=0

n=0

49

(7.2)

Examples

Example 7.1 Solve 4xy + 2y + y = 0 around x0 = 0.
2
First we should classify the given point. The function 4x is not analytic at
x = 0 therefore x = 0 is a singular point. We should make a further test to
determine whether it is regular or not.
x2
The functions 2x and 4x are analytic therefore x = 0 is a R.S.P., we can
4x
use the method of Frobenius.
∞

∞

an xn+r , y =

y=
n=0

∞

(n + r)an xn+r−1 , y =
n=0

(n + r)(n + r − 1)an xn+r−2
n=0

Note that the summation for the derivatives still starts from 0, because r
does not have to be an integer. This is an important difference between
methods of power series and Frobenius.
Inserting these in the equation, we obtain
∞

∞

(n + r)(n + r − 1)an xn+r−2 + 2

4x
n=0

∞

(n + r)an xn+r−1 +
n=0

an xn+r = 0
n=0
7.2. EXAMPLES

51

∞

∞

4(n + r)(n + r − 1)an x

n+r−1

∞

+

n=0

2(n + r)an x
n=0

n+r−1

an xn+r = 0

+

52

CHAPTER 7. FROBENIUS’ METHOD

For simplicity, we may choose a0 = 1. Then
an =

We want to equate the powers of x, so n → n + 1 in the first two terms.
∞

∞

∞

4(n + r + 1)(n + r)an+1 xn+r +
n=−1

2(n + r + 1)an+1 xn+r +
n=−1

an xn+r = 0

Therefore the second solution is :
∞

n=0

Now we can express the equation using a single sigma, but the index of the
common sigma must start from n = 0. Therefore we have to write n = −1
terms separately.
[4r(r−1)+2r]a0 xr−1 +

{[4(n + r + 1)(n + r) + 2(n + r + 1)]an+1 + an } xn+r = 0
n=0

We know that a0 = 0, therefore 4r2 − 2r = 0. This is the indicial equation.
Its solutions are r = 0, r = 1 . Therefore this is Case 1.
2
If r = 0, the recursion relation is

n=0

The general solution is y = c1 y1 + c2 y2

2

First we should classify the given point. The function x x−x is not analytic
2
at x = 0 therefore x = 0 is a singular point. The functions x − 1 and
1 + x are analytic at x = 0 therefore x = 0 is a R.S.P., we can use the
method of Frobenius. Evaluating the derivatives of y and inserting them in
the equation, we obtain

−1
1 an
4(n + 1)(n + 2 )

an+1 =

∞

For simplicity, we may choose a0 = 1. Then

n=0
∞

−

(n + r)an x

(−1)
2n!

y1 =
n=0

n=0

+

an x

n+r

∞

an xn+r+1 = 0

+

n=0

n=0

∞

∞

(n + r)(n + r − 1)an x
n=0
∞

n n

(n + r)an xn+r+1

+

Let’s replace n by n − 1 in the second and fifth terms.

Therefore the first solution is:
∞

n+r

∞
n+r

n=0

n

an =

∞

(n + r)(n + r − 1)an x

a0
a1
a0
a2
a0
a1 = − , a 2 = −
, a3 = −
,...
3 =
5 = −
2
4!
6!
4.2. 2
4.3. 2

−

√

(−1) x
= cos x
2n!

1
If r = , the recursion relation is
2

a1 = −

√
(−1)n xn
= sin x
(2n + 1)!

y2 = x1/2

Example 7.2 Solve x2 y + (x2 − x)y + (1 + x)y = 0 around x0 = 0.

∞

an+1 =

(−1)n
(2n + 1)!

n=0

−1
−an
an =
(2n + 3)(2n + 2)
4(n + 3 )(n + 1)
2

a0
a1
a0
a2
a0
, a2 = −
= , a3 = −
= − ,...
3.2
5.4
5!
7.6
7!

n+r

n=1

∞

(n + r)an x
n=0

n+r

an x

+
n=0

(n + r − 1)an−1 xn+r

+
n+r

∞

an−1 xn+r = 0

+
n=1

[r2 − 2r + 1]a0 xr +
∞

{[(n + r)(n + r − 1) − (n + r) + 1]an + [(n + r − 1) + 1]an−1 } xn+r = 0
n=1

The indicial equation is r2 − 2r + 1 = 0 ⇒ r = 1 (double root). Therefore
this is Case 2. The recursion relation is
an = −

n+1
an−1
n2
7.2. EXAMPLES

53

54

CHAPTER 7. FROBENIUS’ METHOD

Exercises

For simplicity, let a0 = 1. Then
3
3
4
2
a1 = −2, a2 = − a1 = , a3 = − a2 = −
4
2
9
3

Find two linearly independent solutions of the following differential equations in the form of series. Find solutions around the origin (use x0 = 0).
Write the solution in closed form if possible.
1) 2x2 y − xy + (1 + x)y = 0

Therefore the first solution is :
3
2
y1 = x 1 − 2x + x2 − x3 + · · ·
2
3

2) 2xy + (1 + x)y − 2y = 0

To find the second solution, we will use reduction of order. Let y2 = uy1 .
Inserting y2 in the equation, we obtain

3) (x2 + 2x)y + (3x + 1)y + y = 0
4) xy − y − 4x3 y = 0

2

2

2

x y1 u + (2x y1 − xy1 + x y1 )u = 0
Let w = u then

−2

To evaluate the integral u =

1−x+
w=
u=

1
x

⇒

w=

8) (2x2 + 2x)y − y − 4y = 0
xe−x
2
y1

9) 2x2 y + (2x2 − x)y + y = 0

1
w dx we need to find 2 . This is also a series.
y1

3
2
1 − 2x + x2 − x3 + · · ·
2
3

xe−x
=x
2
y1

7) x2 y + (x2 − x)y + y = 0

y1 1
+ − 1 dx
y1 x

ln w = −2 ln y1 + ln x − x

w=

6) 3x2 y + (−10x − 3x2 )y + (14 + 4x)y = 0

1
y
w + 2 1 − +1 w =0
y1 x
dw
=
w

1
1
= 2
2
y1
x

5) xy + y − xy = 0

−2

=

x2 x3
−
+ ···
2!
3!
1 + 3x +

1
x2

1
x2

1 + 4x + 9x2 +
1 + 4x + 9x2 +

11 2 13 3
x + x + ···
2
6

w dx = ln x + 3x +

11 2 13 3
x + x + ···
4
18

13
3
y2 = uy1 = y1 ln x + x 3x − x2 + x3 + · · ·
4
2

46 3
x + ···
3

46 3
x + ···
3

10) 4x2 y + (2x2 − 10x)y + (12 − x)y = 0
11) (x2 + 2x)y + (4x + 1)y + 2y = 0
Use Frobenius’ method to solve the following differential equations around
origin. Find the roots of the indicial equation, find the recursion relation,
and two linearly independent solutions.
12) (x2 + cx)y + [(2 + b)x + c(1 − d)]y + by = 0
(b = 0, c = 0, d is not an integer).
13) x2 y + [(1 − b − d)x + cx2 ]y + [bd + (1 − b)cx]y = 0
(c = 0, b − d is not an integer).
14) x2 y + [(1 − 2d)x + cx2 ]y + (d2 + (1 − d)cx)y = 0
(c = 0)
15) xy + [1 − d + cx2 ]y + 2cxy = 0
(c = 0, d is not an integer).
EXERCISES

55

∞

n=1

(−1)n xn
n! · 3 · 5 · 7 · · · (2n + 1)

1

1+
n=1

2) y = c1

(−1)n xn
n! · 1 · 3 · 5 · · · (2n − 1)

1
1 + 2x + x2
3

∞

+ c2 x

1
2

4) y = a0
n=0

1+
n=1

6
2
3) y1 = 1−x+ x2 − x3 +· · · ,
3
15
∞

n+b
an
c (n + 1 − d)
b
b(b + 1)
y1 = 1 −
x+ 2
x2 − · · ·
c(1 − d)
c (1 − d)(2 − d)

12) r = 0

∞

+ c2 x 2

(−1)n 3xn
2n n!(2n − 3)(2n − 1)(2n + 1)

3
15
35 3
y2 = x1/2 1 − x + x2 −
x + ···
4
32
128
2

x4
x6
x2
+
+
+ ···
22 (2 · 4)2 (2 · 4 · 6)2
x2
3x4
11x6
y2 = y1 ln x −
−
−
− ···
4
8 · 16 64 · 6 · 36

9
27 3
3
6) y1 = x7/3 1 + x + x2 +
x + ···
4
28
280
x2 x3
y 2 = x2 1 + x +
+
+ · · · = x2 e x
2!
3!
x
x
−
+ · · · = xe−x
2!
3!
x2
x3
y2 = xe−x ln x + xe−x x +
+
+ ···
2 · 2! 3 · 3!
1
1
1
y2 = x3/2 1 + x − x2 + x3 − · · ·
2
8
16
∞

y2 = x 1 +
n=1

(−1)n (2x)n
1 · 3 · 5 · · · (2n + 1)
∞

10) y1 = x2 e−x/2 ,

⇒

an+1 = −

y2 = x3/2 1 +
n=1

(−1)n xn
1 · 3 · 5 · · · (2n − 1)

an+1 = −

⇒

an = −

n+b+d
an
c (n + 1)
d+b
(d + b)(d + b + 1) 2
1−
x+
x − ···
c
2! c2

y 2 = xd

c
an−1
n+b−d

c2
c
x+
x2 − · · ·
1+b−d
(1 + b − d)(2 + b − d)
c
r = d ⇒ an = − an−1
n
c2 2 c3 3
y2 = xd 1 − c x + x − x + · · · = xd e−cx
2!
3!
y 1 = xb 1 −

c
an−1
n
c3
c2
1 − c x + x2 − x3 + · · ·
2!
3!

14) r = d (double root) an = −
y 1 = xd

3

7) y1 = x 1 − x +

8) y1 = 1 − 4x − 8x2 ,

r=d

2

OR y = c1 ex + c2 e−x

5) y1 = 1 +

2

⇒

13) r = b

∞

x4n
x4n+2
+ a2
,
(2n)!
(2n + 1)!
n=0

9) y1 = x1/2 e−x ,

CHAPTER 7. FROBENIUS’ METHOD

8
11) y1 = 1 − 2x + 2x2 − x3 + · · ·
5
5
35 2 105 3
1/2
1− x+
y2 = x
x −
x + ···
4
32
128

Answers
1) y = c1 x 1 +

56

y2 = xd e−cx

ecx
dx
x

y2 = xd e−cx ln x + xd e−cx cx +
15) r = 0

⇒

= xd e−cx

an+2 = −

c2 2
c3 3
x +
x + ···
2 · 2!
3 · 3!

c
an
(n + 2 − d)

c
c2
c3
x2 +
x4 −
x6 + · · ·
2−d
(2 − d)(4 − d)
(2 − d)(4 − d)(6 − d)
c
r = d ⇒ an+2 = −
an
n+2
c
c2 4
c3
y 2 = xd 1 − x2 +
x −
x6 + · · ·
2
2·4
2·4·6
y1 = 1 −
58

CHAPTER 8. LAPLACE TRANSFORM I

Example 8.1 Evaluate the Laplace transform of the following functions:
a) f (t) = 1
∞
0

Chapter 8

0

1
= ,
s

s>0

b) f (t) = eat
∞

Laplace Transform I

0

c) f (t) =
Laplace transform provides an alternative method for many equations. We
first transform the differential equation to an algebraic equation, then solve
it, and then make an inverse transform. Laplace transform has a lot of
interesting properties that make these operations easy. In this chapter, we
will see the definition and the basic properties. We will also compare this
method to the method of undetermined coefficients, and see in what ways
Laplace transform is more convenient.

0
1

if
if

∞
1

∞

e−st f (t)dt

(8.1)

0

then, the inverse transform will be
f (t) = L−1 {F (s)}

(8.2)

Note that we use lowercase letters for functions and capital letters for their
transforms.

57

=
0

1
, s>a
s−a

∞

=
1

e−s
,
s

s>0

d) f (t) = t
∞

te−st dt

L {t} =
0

Using integration by parts, we obtain

L {t} = −

The Laplace transform of a function f (t) is defined as:

e−st
−s

e−st dt =

L {f } =

Definition, Existence and Inverse of Laplace
Transform

∞

0<t<1
1 t

L {t} = −t

F (s) = L {f (t)} =

e(a−s)t
a−s

eat e−st dt =

L eat =

8.1

∞

e−st
−s

e−st dt =

L {1} =

e−st
s

∞

e−st
s2

∞

∞

+
0

0

=
0

e−st
dt
s

1
, s>0
s2

The integral that defines the Laplace transform is an improper integral,
it may or may not converge. In the above examples, the transform is defined
for a certain range of s.
In practice, we can use Laplace transform on most of the functions we
encounter in differential equations. The following definitions and the theorem
answer the question Which functions have a Laplace transform?
Piecewise Continuous Functions: A function f (t) is piecewise continuous
on [a, b] if the interval can be subdivided into subintervals [ti , tj ],
a = t0 < t1 < t2 · · · < tn = b such that f (t) is continuous on each interval
and has finite one-sided limits at the endpoints (from the interior).
An example can be seen on Figure 8.1.
8.2. PROPERTIES

59

60

CHAPTER 8. LAPLACE TRANSFORM I
• Transform of Derivatives
L {f }
L {f }
L f (n)

= sL {f } − f (0)
= s2 L {f } − sf (0) − f (0)
= sn L {f } − sn−1 f (0) − sn−2 f (0) − · · · − f (n−1) (0)

• Transform of Integrals
t

L

f (x) dx

=

0

F (s)
s

Example 8.2 Find the Laplace transform of sin at and cos at. Hint: Use
Euler’s formula eix = cos x + i sin x and linearity.
Figure 8.1: A piecewise continuous function
Exponential Order: f (t) is of exponential order as t → ∞ if there exist
real constants M, c, T such that |f (t)| M ect for all t T . In other words,
a function is of exponential order if it does not grow faster than ect .
Theorem 8.1: If f (t) is of exponential order and piecewise continuous on
[0, k] for all k > 0, then its Laplace transform exists for all s > c.
For example, all the polynomials have a Laplace transform. The function
t2
e does NOT have a Laplace transform.

8.2

Basic Properties of Laplace Transforms

It is difficult to evaluate the Laplace transform of each function by performing an integration. Instead of this, we use various properties of Laplace
transform.
Let L {f (t)} = F (s), then, some basic properties are: (assuming these
transforms exists)
• Linearity
L {af + bg} = aL {f } + bL {g}
• Shifting

L {eiat } − L {e−iat }
2i
1
1
1
a
L {sin at} =
−
= 2
2i s − ia s + ia
s + a2
Similarly, we can show that the transform of f (t) = cos at is
s
F (s) = 2
s + a2
1
Example 8.3 Find the inverse Laplace transform of F (s) =
.
(s + 5)2
Hint: Use shifting.
sin at =

eiat − e−iat
2i

We know that L−1

1
s2

L−1 {F (s − a)} = eat f (t)

L {sin at} =

= t. Therefore
1
(s + 5)2

L−1

= te−5t

Example 8.4 Find the Laplace transform of f (t) = t2 . Hint: Use Derivatives.
Using L {f } = sL {f } − f (0), we obtain
L {2t}
2
= 3
s
s
Example 8.5 Find the Laplace transform of f (t) = t3 . Hint: Use Integrals.
L {2t} = sL t2 − 0

⇒

L t2 =

Using the integral rule, we see that
L

L eat f (t) = F (s − a)

⇒

L {t2 }
2
= 4
s
s
6
L t3 = 4
s

t3
3

=
8.3. INITIAL VALUE PROBLEMS

8.3

61

Initial Value Problems

Consider the constant-coefficient equation
y + ay + by = r(t)

(8.3)

y(0) = p, y (0) = q

62

CHAPTER 8. LAPLACE TRANSFORM I

The only disadvantage is that, sometimes finding the inverse Laplace
transform is too difficult.
We have to find roots of the polynomial s2 + as + b, which is the same as
the characteristic polynomial we would encounter if we were using method
of undetermined coefficients.

(8.4)

with initial values

Here y is a function of t (y = y(t)). We can solve it by the method of undetermined coefficients. The method of Laplace transform will be an alternative
that is more efficient in certain cases. It also works for discontinuous r(t).
Let us evaluate the Laplace transform of both sides.

Example 8.6 Solve the initial value problem
y + 4y = 0, y(0) = 5, y (0) = 3.
Let’s start by finding the transform of the equation.
L {y } + 4L {y} = 0

L {y } + aL {y } + bL {y} = L {r(t)}

(8.5)

Using L {y} = Y (s) and L {r(t)} = R(s)
s2 Y − sp − q + a(sY − p) + bY = R

(8.6)

(s2 + as + b)Y = R + (s + a)p + q

s2 Y − 5s − 3 + 4Y = 0

⇒ (s2 + 4)Y = 5s + 3
5s + 3
Y = 2
s +4
Now, we have to find the inverse transform of Y to obtain y(t).

(8.7)

Y =
y = L−1

R + (s + a)p + q
s2 + as + b
R + sp + ap + q
s2 + as + b

(8.8)
(8.9)

Y =

5s
3 2
+
+ 4 2 s2 + 4

s2

3
sin 2t
2
Note that we did not first find the general solution containing arbitrary constants. We directly found the result.
y(t) = L−1 {Y } = 5 cos 2t +

Note that this method can be generalized to higher order equations. The
advantages compared to the method of undetermined coefficients are:
Example 8.7 Solve the initial value problem
• The initial conditions are built in the solution, we don’t need to determine constants after obtaining the general solution.
• There is no distinction between homogeneous and nonhomogeneous
equations, or single and multiple roots. The same method works in all
cases the same way.

y − 4y + 3y = 1,

y(0) = 0,

y (0) = −

Transform both sides:
L {y − 4y + 3y} = L {1}
Use the derivative rule

• The function on the right hand side r(t) belongs to a wider class. For
example, it can be discontinuous.

s2 Y − s.0 +

1
1
− 4(sY − 0) + 3Y =
3
s

1
3
8.3. INITIAL VALUE PROBLEMS
Isolate Y

63

3−s
1 1
− =
s 3
3s
s−3
(s − 1)(s − 3)Y = −
3s
1
1 1
1
=
−
Y =−
3s(s − 1)
3 s s−1

64

CHAPTER 8. LAPLACE TRANSFORM I

(s2 − 4s + 3)Y =

f (t)

F (s)

f (t)

F (s)

1

1
s

eat − ebt
a−b

1
(s − a)(s − b)

t

1
s2

aeat − bebt
a−b

s
(s − a)(s − b)

n!

eat sin bt

b
(s − a)2 + b2
s−a
(s − a)2 + b2

Find the inverse transform
y(t) = L−1 {Y } =

1 1 t
− e
3 3

As you can see, there’s no difference between homogeneous and nonhomogeneous equations. Laplace transform works for both types in the same way.

tn

sn+1

eat

Example 8.8 Solve the initial value problem
y + 4y + 4y = 42te−2t , y(0) = 0, y (0) = 0

1
s−a

eat cos bt

teat

1
(s − a)2
a
s 2 + a2
s
s 2 + a2

tn eat

−2t

L {y } + 4L {y } + 4L {y} = 42L te
1
s2 Y + 4sY + 4Y = 42 ·
(s + 2)2
42
(s2 + 4s + 4)Y =
(s + 2)2
42
Y =
(s + 2)4

sin at
cos at

sinh at
cosh at

n!
(s − a)n+1
a
s 2 − a2
s
s 2 − a2

42 3 −2t
te
3!

2as
(s2 + a2 )2

t sinh at

2as
(s2 − a2 )2

t cos at

s 2 − a2
(s2 + a2 )2

t cosh at

s 2 + a2
(s2 − a2 )2

sin at − at cos at

y(t) = L−1 {Y (s)} =

t sin at

2a3
(s2 + a2 )2

sin at + at cos at

2as2
(s2 + a2 )2

y(t) = 7t3 e−2t
If you try the method of undetermined coefficients on this problem, you will
appreciate the efficiency of Laplace transforms better.

Table 8.1: A Table of Laplace Transforms
EXERCISES

65

Exercises

3) f (t) = 2e−t cos2 t

4) f (t) = (t + 1)2 et

5) f (t) = t3 e3t

6) f (t) =

7) f (t) =

CHAPTER 8. LAPLACE TRANSFORM I

Answers

Find the Laplace transform of the following functions:
t
2) f (t) = et sin 3t
1) f (t) = cos2 2

t 0<t<a
0
a<t

66

1 0<t<a
0
a<t


 t 0<t<a

8) f (t) =
1 a<t<b


0
b<t

1) F (s) =
2) F (s) =
3) F (s) =
4) F (s) =
5) F (s) =

1
s
+ 2
2s 2s + 2
3
(s − 1)2 + 9
1
s+1
+
s + 1 s2 + 2s + 5
2
2
1
+
+
(s − 1)3 (s − 1)2 s − 1
6
(s − 3)4

1 − e−as
s
1
ae−as e−as
7) F (s) = 2 −
− 2
s
s
s
−as
−as
1−e
e
− ae−as − e−bs
8) F (s) =
+
s2
s
9) f (t) = cosh 2t − 2 sinh 2t
6) F (s) =

Find the inverse Laplace transform of the following functions:
s−4
9) F (s) = 2
s −4
3
10) F (s) =
(s − 2)2
6
11) F (s) =
s(s + 4)
1
12) F (s) =
s(s2 + 9)
1
13) F (s) = 2
s (s + 1)
5s + 1
14) F (s) = 2
s +4
15) F (s) =

1
s+8

1
16) F (s) =
(s − a)n
Solve the following initial value problems using Laplace transform:
17) y − 2y + y = 0,
y(0) = 4, y (0) = −3
18) y − 2y + 2y = 0, y(0) = 0, y (0) = 1
19) y + 2y = 4t2 + 12, y(0) = 4, y (0) = 0
20) y + 6y + 9y = e−3t , y(0) = 0, y (0) = 0

10) f (t) = 3te2t
11) f (t) = (3 − 3e−4t )/2
12) f (t) = (1 − cos 3t)/9
13) f (t) = e−t + t − 1
1
14) f (t) = 5 cos 2t + sin 2t
2
−8t
15) f (t) = e
16) f (t) =

tn−1 eat
(n − 1)!

17) y(t) = 4et − 7tet
18) y(t) = et sin t
19) y(t) = 4 + 2t2
1
20) y(t) = e−3t t2
2
68

CHAPTER 9. LAPLACE TRANSFORM II

Reversing the order of integration, we obtain:
∞

∞

f (x) g(t − x)e−st dt dx

=
0

x

Making the substitution u = t − x, we obtain:
∞

∞

f (x) g(u)e−su−sx dudx

L {f ∗ g} =

Chapter 9

0

∞

0

∞
0

0

Laplace Transform II

g(u)e−su du

f (x)e−sx dx

=

=F (s) G(s)
Example 9.1 Find the inverse Laplace transform of F (s) =

In this chapter, we will study more advanced properties of Laplace transform.
At the end, we will be able to find transform and inverse transform of a wider
range of functions. This will enable us to solve almost any linear constant
coefficient equation, including discontinuous inputs.

9.1

L−1

1
s2

1
s+4

= t, L−1

t

(9.1)

0

The convolution operation is commutative, in other words f ∗ g = g ∗ f
Theorem 9.1: The transform of convolution of two functions is equal to
the product of their transforms, i.e.
L {f ∗ g} = F (s) · G(s)
−1

L

{F (s) · G(s)} = f ∗ g

where L {f } = F (s) and L {g} = G(s).
Proof: Using the definitions of convolution and Laplace transform,
t

L {f ∗ g} =L

f (x) g(t − x) dx
0
∞

t

f (x) g(t − x)e−st dx dt

=
0

0

67

L−1

1
1
·
2 s+4
s

(9.2)
(9.3)

= t ∗ e−4t

xe−4(t−x) dx
0

xe4x e4x
−
4
16
−4t
t
1
e
= −
+
4 16
16

t

= e−4t

0

Example 9.2 Find the inverse Laplace transform of F (s) =

The convolution of two functions f and g is defined as
f (x)g(t − x) dx

⇒

1
.
+ 4s2

t

f (t) = t ∗ e−4t =

Convolution

h(t) = (f ∗ g)(t) =

= e−4t

s3

s
.
(s2 + 1)2

s
1
· 2
= L {cos t} · L {sin t},
+ 1) (s + 1)
we will see that f (t) = L−1 {F } = cos t ∗ sin t.
If we express F as F (s) =

(s2

t

cos(x) sin(t − x) dx

f (t) =
0
t

=
0

1
=
2
=

1
[sin(t − x + x) + sin(t − x − x)] dx
2
t

[sin(t) + sin(t − 2x)] dx
0

1
cos(t − 2x)
x sin t +
2
2

t
0

1
1
= t sin t + (cos t − cos t)
2
2
1
= t sin t
2
9.2. UNIT STEP FUNCTION

9.2

69

70

CHAPTER 9. LAPLACE TRANSFORM II

Unit Step Function

∞

e−st f (t)dt

F (s) =
The Heaviside step function (or unit step function) is defined as

0

F (s) =
ua (t) = u(t − a) =

0
1

if
if

t<a
t a

(9.4)

This is a simple on off function. It is especially useful to express discontinuous inputs.

Theorem 9.2: [t−shifting] Let L {f (t)} = F (s), then
(9.5)

Proof: Using the definition,
∞

e−st f (t − a) u(t − a) dt
∞

e−st f (t − a) dt
a

∞

e−sa−sx f (x) dx

=
0
−as

=e

( where x = t − a)

F (s)

Example 9.3 Find the Laplace transform of g(t) =

0
t

if
if

t<5
t 5

We can express g(t) as g(t) = u(t − 5)f (t − 5) where f (t) = (t + 5). Then
F (s) = L {f (t)} =

9.3

In other words
L {tf (t)} = −F (s)

5
1
+
2
s
s

⇒

(9.7)

Repeating this procedure n times, we obtain:
dn
F (s)
dsn

Using the derivative formula, we find
L {t sin t} = −

=

f (t)dt

0

Example 9.4 Find the Laplace transform of f (t) = t sin t.

L {f (t − a) u(t − a)} = e−as F (s)

0

(−t)e

−st

L {tn f (t)} = (−1)n

Figure 9.1: u(t − a) and its effect on f (t)

L {f (t − a) u(t − a)} =

(9.6)

∞

L {g(t)} = e−5s

1
5
+
2
s
s

Differentiation of Transforms

If f (t) is piecewise continuous and of exponential order, then we can differentiate its Laplace transform integral.

d
ds

1
1 + s2

=

2s
(1 + s2 )2

(9.8)
9.4. PARTIAL FRACTIONS EXPANSION

9.4

71

Partial Fractions Expansion

CHAPTER 9. LAPLACE TRANSFORM II

9.5

In many applications of Laplace transform, we need to expand a rational
function in partial fractions. Here, we will review this technique by examples.
2x + 1
A
B
C
=
+
+
(x − 2)(x + 3)(x − 1)
x−2 x+3 x−1
2
x + 4x − 5
B
C
A
D
+
+
=
+
(x − 2)(x − 1)3
x − 2 x − 1 (x − 1)2 (x − 1)3
x3 + 1
Dx + E
A Bx + C
+ 2
+ 2
=
2 + 4)2
x(x
x
x +4
(x + 4)2
3
A
B
x3 − 4x2 + x + 9
= x+1+ 2
=x+1+
+
x2 − 5x + 6
x − 5x + 6
x−2 x−3
• We can express any polynomial as a product of first and second order
polynomials.
• For second order polynomials in the expansion, we have to use Ax + B
(not simply a constant) in the numerator.
• If numerator’s degree is greater or equal to the denominator, we should
first divide them using polynomial division.
2

Example 9.5 Find the inverse Laplace transform of F (s) =

72

−s + 7s − 1
.
(s − 2)(s − 5)2

Applications

Now we are in a position to solve a wider class of differential equations using
Laplace transform.

Example 9.6 Solve the initial value problem
y − 6y + 8y = 2e2t , y(0) = 11, y (0) = 37
We will first find the Laplace transform of both sides, then find Y (s)
L {y } − 6L {y } + 8L {y} = L 2e2t

s2 Y − 11s − 37 − 6(sY − 11) + 8Y =
(s2 − 6s + 8)Y =

−s2 + 7s − 1 = A(s − 5)2 + B(s − 2)(s − 5) + C(s − 2)
Inserting s = 2, we see that 9 = 9A ⇒ A = 1.
Inserting s = 5, we see that 9 = 3C ⇒ C = 3.
The coefficient of s2 : A + B = −1 therefore B = −2. So
1
−s2 + 7s − 1
2
3
=
−
+
2
(s − 2)(s − 5)
s − 2 s − 5 (s − 5)2
Now we can easily find the inverse Laplace transform:
L−1 {F (s)} = e2t − 2e5t + 3te5t

2
+ 11s − 29
s−2

The factors of s2 − 6s + 8 are (s − 2) and (s − 4), so
Y =

2
11s − 29
+
(s − 2)(s − 2)(s − 4) (s − 2)(s − 4)
Y =

First, we have to express F (s) in terms of simpler fractions:
−s2 + 7s − 1
A
B
C
=
+
+
(s − 2)(s − 5)2
s − 2 s − 5 (s − 5)2

2
s−2

11s2 − 51s + 60
(s − 2)2 (s − 4)

Now we need to find the inverse Laplace transform. Using partial fractions
expansion
Y =

A
C
B
+
+
2
s − 2 (s − 2)
s−4

After some algebra we find that A = 3, B = −1, C = 8 so
Y (s) =

8
1
3
−
+
2
s − 2 (s − 2)
s−4

y(t) = L−1 {Y (s)} = 3e2t − te2t + 8e4t
9.5. APPLICATIONS

73

Example 9.7 Solve the initial value problem

74

CHAPTER 9. LAPLACE TRANSFORM II

Example 9.8 Solve the initial value problem

y + y = f (t), y(0) = 0, y (0) = 3
where f (t) =

0
2 cos t

if
if

0 < t < 5π
5π < t

y + 2y + y = r(t), y(0) = 0, y (0) = 0
where r(t) =

As you can see, the input function is discontinuous, but this makes no
difference for Laplace transform.
L {y } + L {y} = L {f }

t
0

if
if

0<t<1
1<t

Once again we have a discontinuous input. This time we will use unit
step function. First, we have to express r(t) with a single formula.
r(t) = t − u(t − 1)t = t − u(t − 1)(t − 1) − u(t − 1)

2

s Y −3+Y =F
F +3
Y = 2
s +1
1
Using the fact that L {sin t} = 2
, we can obtain y(t) by convolution:
s +1
y(t) = L−1 {Y } = f (t) ∗ sin t + 3 sin t

Its Laplace transform is
R(s) = L {r(t)} =

Finding the Laplace transform of the equation, we obtain
(s2 + 2s + 1)Y = R

Using the definition of convolution,
t

f ∗ sin t =

Y =

f (x) sin(t − x) dx
0

If t < 5π, f = 0 therefore this integral is also zero. If t > 5π we have

Y =

t

f ∗ sin t =

1
e−s e−s
− 2 −
2
s
s
s

2 cos x sin(t − x) dx

s2 (s

R
(s + 1)2

1
e−s
− 2
2
+ 1)
s (s + 1)

Using partial fractions expansion

5π

Using the trigonometric identity 2 sin A cos B = sin(A + B) + sin(A − B) we
obtain
t
f ∗ sin t =

sin t + sin(t − 2x) dx

2
1
2
1
1
1
1
Y =− + 2 +
+
− e−s − + 2 +
s s
s + 1 (s + 1)2
s s
s+1
Using the fact that L−1 {e−as F (s)} = f (t − a)u(t − a), we obtain

5π

=

x sin t +

cos(t − 2x)
2

t
5π

= (t − 5π) sin t
Therefore

y(t) = −2 + t + 2e−t + te−t − u(t − 1) −1 + (t − 1) + e−(t−1)
We know that u(t − 1) = 0 for t > 1 and u(t − 1) = 1 for t > 1 so
y(t) =

y(t) =

3 sin t
(t − 5π + 3) sin t

if
if

0 < t < 5π
5π < t

−2 + t + 2e−t + te−t
(2 − e)e−t + te−t

if
if

0<t<1
1<t
EXERCISES

75

Exercises

1) F (s) =

3

s+3
(s2 + 4)2

8) F (s) =

2

2) F (s) =

s
s4 + 4a4

10) F (s) =

sin 4t − 4t cos 4t
128
4t sin 2t + 3 sin 2t − 6t cos 2t
7) f (t) =
16
1
t
9) f (t) = sin 2t + cos 2t
4
2
5) f (t) =

6) f (t) =

10) f (t) = e2t + 2 cos 3t +

12) y = 3 cos t + (4 + t) sin t
13) y = −25 + 8t2

Solve the following initial value problems : (where y = y(t))

1 −t
e − et cos t + 7et sin t
5
15) y = t − sin t
14) y =

11) y − y − 2y = 0, y(0) = 8, y (0) = 7
12) y + y = 2 cos t, y(0) = 3, y (0) = 4
13) y + 0.64y = 5.12t2 , y(0) = −25, y (0) = 0

1
0 < t < 2π
cos t
2π < t

16) y =

14) y − 2y + 2y = e−t , y(0) = 0, y (0) = 1
15) y + y = t, y(0) = 0, y (0) = 0
16) y + y = r(t), y(0) = 1, y (0) = 0 where r(t) =

1
0

if
if

0 < t < 2π
2π < t

5
0

if
if

17) y + y = e−2t sin t, y(0) = 0, y (0) = 0
18) y +2y +5y = r(t), y(0) = 0, y (0) = 0 where r(t) =

0<t<π
π<t

19) 4y + 4y + 17y = g(t), y(0) = 0, y (0) = 0
sin t
0

if
if

0 < t < 3π
1
7
, y(0) = , y (0) = −
50
50
3π < t

1
1
17) y = (sin t − cos t) + e−2t (sin t + cos t)
8
8


 1 − e−t cos 2t + sin 2t , 0 < t < π

2
18) y =
 −t π
 e (e − 1) cos 2t + sin 2t
π<t

2
19) y =

1
8




t

1

e− 2 (t−x) sin 2(t − x)g(x) dx
0

1
(cos t − 7 sin t)
50
20) y =
1 −9π 3t
2


e e − e6π e−2t
50
50

1 e−t e−3t
−
+
3
2
6

8) f (t) = cosh at cos at

11) y = 3e−t + 5e2t

3s − 2s + 5
(s − 2)(s2 + 9)

12s2 − 16
(s2 + 4)3

4) f (t) = u(t − 1) cos(2t − 2)

2

s
(s2 + 4)2

20) y − y − 6y =

s2 + 2s
(s2 + 2s + 2)2

3) f (t) = u(t − 3) sin(t − 3)

Find the inverse Laplace transform transform of the following functions:
se−s
e−3s
4) F (s) = 2
3) F (s) = 2
s +1
s +4
1
1
5) F (s) = 2
6) F (s) = 3
2
(s + 16)
s + 4s2 + 3s

9) F (s) =

CHAPTER 9. LAPLACE TRANSFORM II

Answers

Find the Laplace transform transform of the following functions:
1) f (t) = te−t cos t
2) f (t) = t2 sin 2t

7) F (s) =

76

if

0 < t < 3π

if

3π < t

2
sin 3t
3
78

CHAPTER 10. FOURIER ANALYSIS I

L

cos

nπx
mπx
cos
dx = 0 (m = n)
L
L

(10.3)

sin

nπx
mπx
sin
dx = 0 (m = n)
L
L

(10.4)

−L
L

Chapter 10

−L
L

cos2

Fourier Analysis I

−L

−L

nπx
mπx
sin
dx = 0 (for all m, n)
L
L
77

kπx
dx =
L

L

a0 cos
−L
∞

+
+

(10.1)

(10.2)

kπx
dx
L

L

cos

nπx
kπx
cos
dx
L
L

sin

an

nπx
kπx
cos
dx
L
L

−L

n=1
∞

∞

If possible, this expansion would be very useful in all kinds of applications.
Once we solve a question for sine and cosine functions, we will be able to
solve it for any periodic f . Here, an and bn are the coordinates of f in the
space of sine and cosine functions. But then how can we find an and bn ? The
following identities will help us:

(10.6)

Now, suppose the expansion (10.1) exists. To find ak , we will multiply
both sides by cos kπx and then integrate from −L to L.
L
L

nπx
nπx
f (x) = a0 +
an cos
+
bn sin
L
L
n=1
n=1

−L

(10.5)

2 cos A sin B = sin(A + B) − sin(A − B)

Let f (x) be a periodic function with period 2L. It is sufficient that f be
defined on [−L, L]. Is it possible to express f as a linear combination of sine
and cosine functions?

cos

−L

nπx
dx = L
L

2 sin A sin B = cos(A − B) − cos(A + B)

f (x) cos

L

sin2

2 cos A cos B = cos(A − B) + cos(A + B)

Fourier Series

∞

L

In the terminology of linear algebra, the trigonometric functions form
an orthogonal coordinate basis. We can easily prove these formulas if we
remember the following trigonometric identities:

The trigonometric functions sine and cosine are the simplest periodic functions. If we can express an arbitrary periodic function in terms of these,
many problems would be simplified. In this chapter, we will see how to
find the Fourier series of a periodic function. Fourier series is important in
many applications. We will also need them when we solve partial differential
equations.

10.1

nπx
dx =
L

L

bn
n=1

−L

(10.7)

Using the property of orthogonality, we can see that all those integrals
are zero, except the kth one. Therefore

L

f (x) cos
−L

kπx
dx = ak L
L

⇒

ak =

1
L

L

f (x) cos
−L

kπx
dx
L

(10.8)

We can apply the same procedure to find a0 and bn . In the end, we will
obtain the following formulas for a function f defined on [−L, L].
10.1. FOURIER SERIES

79

a0 =
Fourier coefficients:

an =
bn =

1
2L
1
L
1
L

CHAPTER 10. FOURIER ANALYSIS I

L

f (x) dx
−L
L

f (x) cos

nπx
dx
L

f (x) sin

nπx
dx
L

−L
L
−L

(10.9)

∞

∞

Fourier series: f (x) = a0 +

80

an cos
n=1

nπx
nπx
+
bn sin
L
L
n=1

(10.10)

Example 10.1 Find the Fourier series of the periodic function
f (x) = x2 , −L x L having period= 2L.

a0 =
=

1
2L

L

x2 dx
−L
3 L

1 x
2L 3

=
−L

L2
3

Using integration by parts two times we find:
an =

=
bn =

1
L

L

x2 cos
−L

nπx
dx
L

4L2 cos nπ
n2 π 2
1
L

L

x2 sin
−L

nπx
dx = 0
L

Therefore the Fourier series is:
∞

x2 =

4L2
nπx
L2
+
(−1)n 2 2 cos
3
nπ
L
n=1

The plot of the Fourier series up to n = 1, 2 and 3 is given in Figure 10.1.
Figure 10.1: Fourier Series of f = x2 for n = 1, 2, 3
10.2. CONVERGENCE OF FOURIER SERIES

10.2

81

82

CHAPTER 10. FOURIER ANALYSIS I

Convergence of Fourier Series

Like any infinite series, Fourier series is of no use if it is divergent. But
most functions that we are interested in have Fourier series that converge
and converge to the function.
Theorem 10.1: Let f be periodic with period 2L and let f and f be
piecewise continuous on the interval [−L, L]. Then the Fourier expansion of
f converges to:
• f (x) if f is continuous at x.
f (x+ ) + f (x− )
if f is discontinuous at x.
2
Example 10.2 Find the Fourier series of the periodic function
•

a
b

f (x) =

Figure 10.2: Convergence at a discontinuity

10.3

Theorem 10.2: Let f be continuous on [−L, L], f (L) = f (−L) and let f
be piecewise continuous. Then the Fourier coefficients of f satisfy:
∞

2a2 +
0

having period= 2L. Then evaluate the series at x = L.

a0 =

1
2L

a dx +
−L

1
2L

L

b dx =
0

a+b
2

L

1
L

f (x)2 dx

∞

f 2 (x) = a0 f (x) +

∞

L

f 2 (x) dx = a0

f (x) dx +
−L

∞
n=1

an cos nπx +
L

bn =

1
L

0

nπx
1
a cos
dx +
L
L
−L
0

a sin
−L

nπx
1
dx +
L
L

a L
nπx
=−
cos
L nπ
L

0

L
0

n=1

∞

L

an

f (x) cos
−L

nπx
bn
dx +
L
n=1

b sin
0

L
0

b−a
=
(1 − (−1)n )
nπ

Therefore the Fourier series is:
∞
a+b
b−a
nπx
+
[1 − (−1)n ] sin
f (x) =
2
nπ
L
n=1
a + b 2(b − a)
+
2
π

πx 1
3πx 1
5πx
+ sin
+ sin
+ ···
L
3
L
5
L
a+b
If we insert x = L in that series, we obtain f (L) =
. Thus the value at
2
discontinuity is the average of left and right limits. The summation of the
series up to n = 1, 5 and 9 is plotted on Figure 10.2.
=

f (x) sin
−L

1
1
1
= 1 + 4 + 4 + ···
4
n
2
3
n=1
2
(Hint: Use the Fourier series of f (x) = x on the interval −π < x < π)

Example 10.3 Find the sum of the series S =

nπx
dx
L

b L
nπx
−
cos
L nπ
L
−L

L

Using equation (10.9) to evaluate these integrals, we can obtain the result.

nπx
b cos
dx = 0
L

L

sin nπx .
L

nπx
nπx
+
bn f (x) sin
L
L
n=1

∞

1
an =
L

∞
n=1 bn

∞

an f (x) cos
n=1

L

(10.11)

−L

Proof: We can express f (x) as f (x) = a0 +
Now multiply both sides by f and integrate

−L
0

(a2 + b2 ) =
n
n
n=1

−L < x < 0
0<x<L

if
if

Parseval’s Identity

Evaluating the integrals in (10.9) for f (x) = x2 we obtain
π2
4(−1)n
and bn = 0 so
a0 = , a n =
3
n2
f (x) =

1
π2
1
− 4 cos x − cos 2x + cos 3x − · · ·
3
4
9

Using Parseval’s theorem, we have

sin

1
2π 4
1
+ 16 1 + 4 + 4 + · · ·
9
2
3

Therefore

1 π 4
x dx
π −π
2
= π4
5

=

nπx
d
L
10.3. PARSEVAL’S IDENTITY

1
1
2 2
−
16 1 + 4 + 4 + · · · = π 4
2
3
5 9
1
1
π4
S = 1 + 4 + 4 + ··· =
2
3
90

83

84

CHAPTER 10. FOURIER ANALYSIS I

Exercises
Find the Fourier series of the periodic function f (x) defined on the given
interval

1) f (x) = x, −π < x < π
3) f (x) =

0
1

2) f (x) = x, 0 < x < 2π

−π < x < 0
0<x<π

if
if

5) f (x) = sin2 x, −π < x < π
7) f (x) =

−π/4
π/4

if
if

−1 < x < 0
0<x<1

9) f (x) = |x|, −2 < x < 2
11) f (x) =

x
1−x

if
if

4) f (x) = x2 , 0 < x < 2π
6) f (x) = x + |x|, −π < x < π
8) f (x) =

π
x

if
if

10) f (x) = | sin x|, −π < x < π
0<x<1
1<x<2

13) f (x) = ax + b, −L < x < L
15) f (x) = x3 , −π < x < π

12) f (x) =

−a
a

1
π2
1
+
+ ··· = .
9 25
8

−L < x < 0
0<x<L

16) f (x) = ex , −π < x < π

x sin ax cos ax
+
a
a2
x cos ax sin ax
+
x sin ax dx = −
a
a2
2
x sin ax 2x cos ax 2 sin ax
x2 cos ax dx =
+
−
a
a2
a3
2
x cos ax 2x sin ax 2 cos ax
x2 sin ax dx = −
+
+
a
a2
a3
x cos ax dx =

if
if

14) f (x) = 1 − x2 , −1 < x < 1

17) Using integration by parts, show that:

18) Show that 1 +

−π < x < 0
0<x<π
EXERCISES

85

Answers
1
1
sin 2x + sin 3x − · · ·
2
3

2) f (x) = π − 2 sin x +

1
1
sin 2x + sin 3x + · · ·
2
3
1
1
sin x + sin 3x + sin 5x + · · ·
3
5

4π 2
1
1
+ 4 cos x + cos 2x + cos 3x + · · ·
3
4
9
−4π sin x +

5) f (x) =
6) f (x) =

π
4
−
2 π

1
1
sin 2x + sin 3x + · · ·
2
3

1 1
− cos 2x
2 2
cos x +

+2 sin x −
7) f (x) = sin πx +

1
1
cos 3x +
cos 5x + · · ·
9
25

1
1
1
sin 2x + sin 3x − sin 4x + · · ·
2
3
4

1
1
sin 3πx + sin 5πx + · · ·
3
5

∞

3π
(−1)n − 1
1
+
cos nx − sin nx
2
4
πn
n
n=1

9) f (x) = 1 −
10) f (x) =

8
π2

cos

2
4
−
π π

11) f (x) = −
+

4
π2
2
π

sin

13) f (x) = b +

2aL
π

14) f (x) =

2
4
+ 2
3 π

πx 1
3πx 1
5πx
+ sin
+ sin
+ ···
L
3
L
5
L
sin

∞

n=1

3πx
1
5πx
πx 1
+ cos
+
cos
+ ···
2
9
2
25
2
cos 2nx
4n2 − 1

cos πx +
sin πx +

1
1
cos 3πx +
cos 5πx + · · ·
9
25

1
sin 3πx + · · ·
3

(−1)n+1

15) f (x) = 2
n=1

πx
1
2πx 1
3πx
− sin
+ sin
− ···
L
2
L
3
L

cos πx −

∞

∞

1 − (−1)n
1 2
1
sin nx = +
3) f (x) = +
2 n=1
nπ
2 π

8) f (x) =

CHAPTER 10. FOURIER ANALYSIS I
4a
π

12) f (x) =

1) f (x) = 2π sin x −

4) f (x) =

86

1
1
cos 2πx + cos 3πx + · · ·
4
9

(nπ)2 − 6
sin nx
n3
∞

16) f (x) =

2 sinh π 1
(−1)n
+
(cos nx − n sin nx)
π
2 n=1 1 + n2

18) Use the function in exercise 12 in Parseval’s identity
88

CHAPTER 11. FOURIER ANALYSIS II
Figure 11.1: Plots of Some Even and Odd Functions

Chapter 11
Fourier Analysis II
In this chapter, we will study more advanced properties of Fourier series. We
will find the even and odd periodic extensions of a given function, we will
express the series using complex notation and finally, we will extend the idea
of Fourier series to nonperiodic functions in the form of a Fourier integral.

As you can see in Figure 11.1, an even function is symmetric with respect
to y−axis, an odd function is symmetric with respect to origin.
Half Range Extensions: Let f be a function defined on [0, L]. If we want
to expand it in terms of sine and cosine functions, we can think of it as
periodic with period 2L. Now we need to define f on the interval [−L, 0].
There are infinitely many possibilities, but for simplicity, we are interested
in making f an even or an odd function. If we define f for negative x
values as f (x) = f (−x), we obtain the even periodic extension of f , which
is represented by a Fourier cosine series. If we define f for negative x values
as f (x) = −f (−x), we obtain the odd periodic extension of f , which is
represented by a Fourier sine series.
Half-Range Cosine Expansion: (or Fourier cosine series)
∞

f (x) = a0 +

11.1

n=1

Fourier Cosine and Sine Series

If f (−x) = f (x), f is an even function. If f (−x) = −f (x), f is an odd
function. We can easily see that, for functions:
even × even = even,

odd × odd = even,

even × odd = odd

For example |x|, x2 , x4 , cos x, cos nx, cosh x are even functions. x, x3 , sin x, sin nx, sinh x
are odd functions. ex is neither even nor odd.
L

L

If f is even:

f (x) dx

f (x) dx = 2
−L
L

If f is odd:

(11.1)

0

f (x) dx = 0

(11.2)

−L

Using the above equations, we can see that in the Fourier expansion of an
even function, bn = 0, and in the expansion of an odd function, an = 0. This
will cut our work in half if we can recognize the given function as odd or
even.
87

an cos

where a0 =

1
L

nπx
, (0 < x < L)
L

L

f (x) dx,
0

an =

2
L

L

f (x) cos
0

nπx
dx
L

(11.3)

(11.4)
11.1. FOURIER COSINE AND SINE SERIES

89

∞

bn sin
n=1

nπx
, (0 < x < L)
L

CHAPTER 11. FOURIER ANALYSIS II

11.2

Half-Range Sine Expansion: (or Fourier sine series)
f (x) =

90

(11.5)

Complex Fourier Series

Consider the Fourier series of f (x):
∞

∞

f (x) = a0 +

where

L

2
L

bn =

f (x) sin
0

nπx
dx
L

0

f (x) =

bn sin nx

(11.7)

n=1

Using Euler’s formula eix = cos x + i sin x we can express the sine and cosine
functions as:

0<x< π
2
π
<x<π
2

if
if

π
2

n=1

(11.6)

Example 11.1 Find the half-range cosine and sine expansions of

an cos nx +

cos nx =

einx + e−inx
,
2

sin nx =

einx − e−inx
2i

(11.8)

an + ibn
2

(11.9)

Therefore
Here, L = π, therefore
1
a0 =
π
2
π

an =

π
π
2

π
π
2

sin nx
=
n

π
2

einx +

e−inx

If we define c0 = a0 and

π
cos nx dx
2
π

an − ibn
2

an cos nx + bn sin nx =

π
π
dx =
2
4

cn =

nπ
2

sin
=−
n

an + ibn
an − ibn
, c−n =
, n = 1, 2, 3, . . .
2
2

(11.10)

We will obtain
∞

cn einx

(11.11)

f (x)e−inx dx n = 0, ±1, ±2, . . .

(11.12)

f (x) =

Therefore half-range cosine series of f is

n=−∞
∞

sin nπ
π
π
1
1
2
f (x) = −
cos nx = − cos x − cos 3x + cos 5x − · · ·
4 n=1 n
4
3
5

where
cn =

On the other hand,
2
bn =
π

π
π
2

π
sin nx dx
2

− cos nx
=
n

π
π
2

cos nπ − cos nπ
2
=
n

Therefore half-range sine series of f is
∞

f (x) =
n=1

cos nπ − cos nπ
1
1
2
sin nx = sin x − sin 2x + sin 3x + sin 5x + · · ·
n
3
5

1
2π

π
−π

For a function of period 2L we have
∞

cn einπx/L ,

f (x) =

cn =

n=−∞

1
2L

L

f (x)e−inπx/L dx
−L

Example 11.2 Find the complex Fourier series of
f (x) = x if −π < x < π and f (x + 2π) = f (x).
We have to evaluate the integral
cn =

1
2π

π

xe−inx dx
−π

(11.13)
11.2. COMPLEX FOURIER SERIES

91

For n = 0 this integral is zero, so we have c0 = 0. For n = 0
cn =

−inx π

xe
−in

1
2π

π

−
−π
inπ

−π

1
2π

Therefore

CHAPTER 11. FOURIER ANALYSIS II

11.3

Fourier Integral Representation

−inx

e
dx
−in

πe−inπ + πe
−0
−in
1 einπ + e−inπ
cos nπ
=−
=−
in
2
in
i
n
= (−1)
n

=

92

In this section, we will apply the basic idea of the Fourier series to nonperiodic functions.
Consider a periodic function with period= 2L and its Fourier series. In
the limit L → ∞, the summation will be an integral, and f will be a nonperiodic function. Then we will obtain the Fourier integral representation:
∞

i
(−1)n einx ,
n
n=−∞

n=0

where
A(u) =

Note that we can obtain the real Fourier series from the complex one. If we
add nth and −nth terms we get
cos nx + i sin nx
cos(−nx) + i sin(−nx)
sin nx
i(−1)
+ i(−1)−n
= (−1)n+1
n
−n
n

B(u) =

n

∞

n+1 sin nx

(−1)

x=

n

n=1

1
π
1
π

∞

f (x) cos ux dx

(11.15)

f (x) sin ux dx

(11.16)

−∞
∞
−∞

Like the Fourier series, we have A(u) = 0 for odd functions and B(u) = 0 for
even functions.
Theorem 11.1: If f and f are piecewise continuous in every finite interval
∞

This is the real Fourier series.

|f | dx is convergent, then the Fourier integral of f converges to:

and if
−∞

Example 11.3 Find the complex Fourier series of f (x) = k

• f (x) if f is continuous at x.
π

1
ke−inx dx
2π −π
π
k e−inx
=
(n = 0)
2π −in −π

cn =

inπ

•

f (x+) + f (x−)
if f is discontinuous at x.
2

Example 11.4 Find the Fourier integral representation of

−inπ

k e −e
nπ
2i
k
=
sin nπ
nπ
=0
=

If n = 0 we have

(11.14)

0

∞

x=

[A(u) cos ux + B(u) sin ux] du

f (x) =

1
c0 =
2π
=k

f (x) =

π/2
0

if
if

|x| < 1
1 < |x|

Note that f is even therefore B(u) = 0

π

k dx

A(u) =

−π

∞

1
π

f (x) cos ux dx =
−∞
1

=

cos ux dx =
0

sin ux
u

1
π

1

=
0

1
−1

π
cos ux dx
2

sin u
u
11.3. FOURIER INTEGRAL REPRESENTATION

93

Therefore, Fourier integral representation of f is
∞

f (x) =
0

sin u
cos ux du
u

94

CHAPTER 11. FOURIER ANALYSIS II

Exercises
For the following functions defined on 0 < x < L, find the half-range
cosine and half-range sine expansions:

Example 11.5 Prove the following formulas using two different methods:
eax
e cos bx dx = 2
(a cos bx + b sin bx)
a + b2
eax
eax sin bx dx = 2
(a sin bx − b cos bx)
a + b2
We can obtain the formulas using integration by parts, but this is the
long way. A better method is to express the integrals as a single complex
integral using eibx = cos bx + i sin bx, then evaluate it at one step, and then
separate the real and imaginary parts.
ax

Example 11.6 Find the Fourier integral representation of
f (x) =

−ex cos x
e−x cos x

if
if

x<0
0<x

This function is odd therefore A(u) = 0.
2 ∞ −x
1 ∞
f (x) sin ux dx =
e cos x sin ux dx
π −∞
π 0
2 ∞ −x sin(ux + x) + sin(ux − x)
e
dx
=
π 0
2
∞
1
e−x
=
[− sin(u + 1)x − (u + 1) cos(u + 1)x]
π 1 + (u + 1)2
0
∞
e−x
1
[− sin(u − 1)x − (u − 1) cos(u − 1)x]
+
π 1 + (u − 1)2
0
1
u−1
u+1
=
+
π 1 + (u + 1)2 1 + (u − 1)2
2 u3
=
π u4 + 4

B(u) =

So
f (x) =

2
π

∞
0

u3
sin ux du
+4

u4

1) f (x) =

2kx/L
2k(L − x)/L

if
if

0 < x < L/2
2) f (x) = ex
L/2 < x < L

3) f (x) = k

4) f (x) = x4

5) f (x) = cos 2x 0 < x < π

6) f (x) =

0
k

if
if

0 < x < L/2
L/2 < x < L

Find the complex Fourier series of the following functions:
7) f (x) =

0
1

if
if

−π < x < 0
0<x<π

9) f (x) = sin x

8) f (x) = x2 , −L < x < L
10) f (x) = cos 2x

Find the Fourier integral representations of the following functions:
 π
π
 cos x, |x| <

π − x, 0 < x < π
2
2
11) f (x) =
(f odd)
12) f (x) =
π

0,
π<x

0,
|x| >
2
13) f (x) =

e−x , 0 < x
ex , x < 0

π
0

14) f (x) =

Prove the following formulas. (Hint: Define a suitable
then find its Fourier integral representation.)

 πx2 /2,



∞
2
cos ux
2
15)
1 − 2 sin u + cos u
du =
π/4,

u
u
u
0



0,

 0,
x<0

∞
cos ux + u sin ux
16)
du =
π/2, x = 0
 −x
1 + u2
0

πe , x > 0

if
if

0<x<1
Otherwise

function f and
0

x<1

x=1
1<x
258 lecnot2
258 lecnot2
258 lecnot2
258 lecnot2
258 lecnot2
258 lecnot2
258 lecnot2
258 lecnot2
258 lecnot2
258 lecnot2
258 lecnot2
258 lecnot2
258 lecnot2
258 lecnot2
258 lecnot2
258 lecnot2
258 lecnot2
258 lecnot2
258 lecnot2
258 lecnot2
258 lecnot2

More Related Content

What's hot

Complex Numbers
Complex NumbersComplex Numbers
Complex NumbersArun Umrao
 
Cl dys of p and s 5 ed
Cl dys of p and s 5 edCl dys of p and s 5 ed
Cl dys of p and s 5 edroctaiwan101
 
Image Processing
Image ProcessingImage Processing
Image Processingyuvhashree
 
C:\Documents And Settings\Pen Drive\Desktop\Maths Module
C:\Documents And Settings\Pen Drive\Desktop\Maths ModuleC:\Documents And Settings\Pen Drive\Desktop\Maths Module
C:\Documents And Settings\Pen Drive\Desktop\Maths ModuleNidhi
 
Solution of second order nonlinear singular value problems
Solution of second order nonlinear singular value problemsSolution of second order nonlinear singular value problems
Solution of second order nonlinear singular value problemsAlexander Decker
 
Compiled Report
Compiled ReportCompiled Report
Compiled ReportSam McStay
 
Independence Complexes
Independence ComplexesIndependence Complexes
Independence ComplexesRickard Fors
 
Exact solutions for weakly singular Volterra integral equations by using an e...
Exact solutions for weakly singular Volterra integral equations by using an e...Exact solutions for weakly singular Volterra integral equations by using an e...
Exact solutions for weakly singular Volterra integral equations by using an e...iosrjce
 
Free Ebooks Download ! Edhole
Free Ebooks Download ! EdholeFree Ebooks Download ! Edhole
Free Ebooks Download ! EdholeEdhole.com
 
Think Like Scilab and Become a Numerical Programming Expert- Notes for Beginn...
Think Like Scilab and Become a Numerical Programming Expert- Notes for Beginn...Think Like Scilab and Become a Numerical Programming Expert- Notes for Beginn...
Think Like Scilab and Become a Numerical Programming Expert- Notes for Beginn...ssuserd6b1fd
 
The Lights Out Puzzle and Its Variations
The Lights Out Puzzle and Its VariationsThe Lights Out Puzzle and Its Variations
The Lights Out Puzzle and Its VariationsPengfei Li
 
first order ode with its application
 first order ode with its application first order ode with its application
first order ode with its applicationKrishna Peshivadiya
 

What's hot (18)

Complex Numbers
Complex NumbersComplex Numbers
Complex Numbers
 
Cl dys of p and s 5 ed
Cl dys of p and s 5 edCl dys of p and s 5 ed
Cl dys of p and s 5 ed
 
Machine learning cheat sheet
Machine learning cheat sheetMachine learning cheat sheet
Machine learning cheat sheet
 
Diffy Q Paper
Diffy Q PaperDiffy Q Paper
Diffy Q Paper
 
Image Processing
Image ProcessingImage Processing
Image Processing
 
C:\Documents And Settings\Pen Drive\Desktop\Maths Module
C:\Documents And Settings\Pen Drive\Desktop\Maths ModuleC:\Documents And Settings\Pen Drive\Desktop\Maths Module
C:\Documents And Settings\Pen Drive\Desktop\Maths Module
 
Solution of second order nonlinear singular value problems
Solution of second order nonlinear singular value problemsSolution of second order nonlinear singular value problems
Solution of second order nonlinear singular value problems
 
Compiled Report
Compiled ReportCompiled Report
Compiled Report
 
Independence Complexes
Independence ComplexesIndependence Complexes
Independence Complexes
 
Exact solutions for weakly singular Volterra integral equations by using an e...
Exact solutions for weakly singular Volterra integral equations by using an e...Exact solutions for weakly singular Volterra integral equations by using an e...
Exact solutions for weakly singular Volterra integral equations by using an e...
 
Free Ebooks Download ! Edhole
Free Ebooks Download ! EdholeFree Ebooks Download ! Edhole
Free Ebooks Download ! Edhole
 
Ch02 4
Ch02 4Ch02 4
Ch02 4
 
Think Like Scilab and Become a Numerical Programming Expert- Notes for Beginn...
Think Like Scilab and Become a Numerical Programming Expert- Notes for Beginn...Think Like Scilab and Become a Numerical Programming Expert- Notes for Beginn...
Think Like Scilab and Become a Numerical Programming Expert- Notes for Beginn...
 
The Lights Out Puzzle and Its Variations
The Lights Out Puzzle and Its VariationsThe Lights Out Puzzle and Its Variations
The Lights Out Puzzle and Its Variations
 
DCT
DCTDCT
DCT
 
first order ode with its application
 first order ode with its application first order ode with its application
first order ode with its application
 
Operations Research - The Dual Simplex Method
Operations Research - The Dual Simplex MethodOperations Research - The Dual Simplex Method
Operations Research - The Dual Simplex Method
 
Sf math
Sf mathSf math
Sf math
 

Viewers also liked

Primer for ordinary differential equations
Primer for ordinary differential equationsPrimer for ordinary differential equations
Primer for ordinary differential equationsTarun Gehlot
 
Differential Equations Lecture: Non-Homogeneous Linear Differential Equations
Differential Equations Lecture: Non-Homogeneous Linear Differential EquationsDifferential Equations Lecture: Non-Homogeneous Linear Differential Equations
Differential Equations Lecture: Non-Homogeneous Linear Differential Equationsbullardcr
 
Pedagogia da autonomia
Pedagogia da autonomiaPedagogia da autonomia
Pedagogia da autonomiaSoares Junior
 
Differential equation
Differential equation Differential equation
Differential equation azyanmdzahri
 
Higher Differential Equation
Higher Differential Equation Higher Differential Equation
Higher Differential Equation Abdul Hannan
 

Viewers also liked (9)

Chapter 1 (maths 3)
Chapter 1 (maths 3)Chapter 1 (maths 3)
Chapter 1 (maths 3)
 
GFilosofi_solitons_2005
GFilosofi_solitons_2005GFilosofi_solitons_2005
GFilosofi_solitons_2005
 
Nts
NtsNts
Nts
 
Primer for ordinary differential equations
Primer for ordinary differential equationsPrimer for ordinary differential equations
Primer for ordinary differential equations
 
Differential Equations Lecture: Non-Homogeneous Linear Differential Equations
Differential Equations Lecture: Non-Homogeneous Linear Differential EquationsDifferential Equations Lecture: Non-Homogeneous Linear Differential Equations
Differential Equations Lecture: Non-Homogeneous Linear Differential Equations
 
Higher order differential equations
Higher order differential equationsHigher order differential equations
Higher order differential equations
 
Pedagogia da autonomia
Pedagogia da autonomiaPedagogia da autonomia
Pedagogia da autonomia
 
Differential equation
Differential equation Differential equation
Differential equation
 
Higher Differential Equation
Higher Differential Equation Higher Differential Equation
Higher Differential Equation
 

Similar to 258 lecnot2

Calculus Research Lab 3: Differential Equations!
Calculus Research Lab 3: Differential Equations!Calculus Research Lab 3: Differential Equations!
Calculus Research Lab 3: Differential Equations!A Jorge Garcia
 
Principle of Derivative Calculus - Differential Calculus - An Introduction by...
Principle of Derivative Calculus - Differential Calculus - An Introduction by...Principle of Derivative Calculus - Differential Calculus - An Introduction by...
Principle of Derivative Calculus - Differential Calculus - An Introduction by...ssuserd6b1fd
 
Differentiation
DifferentiationDifferentiation
DifferentiationArun Umrao
 
Introdution to differential forms
Introdution to differential formsIntrodution to differential forms
Introdution to differential formsDunga Pessoa
 
Calculus of variations & solution manual russak
Calculus of variations & solution manual   russakCalculus of variations & solution manual   russak
Calculus of variations & solution manual russakJosé Pallo
 
Algorithmic Mathematics.
Algorithmic Mathematics.Algorithmic Mathematics.
Algorithmic Mathematics.Dr. Volkan OBAN
 
SGC 2014 - Mathematical Sciences Tutorials
SGC 2014 - Mathematical Sciences TutorialsSGC 2014 - Mathematical Sciences Tutorials
SGC 2014 - Mathematical Sciences TutorialsDaniel Ogburn
 
Introduction to Theory of elasticity and plasticity Att 6521
Introduction to Theory of elasticity and plasticity Att 6521Introduction to Theory of elasticity and plasticity Att 6521
Introduction to Theory of elasticity and plasticity Att 6521Shekh Muhsen Uddin Ahmed
 
Math516 runde
Math516 rundeMath516 runde
Math516 rundesgcskyone
 
Partial differential equations, graduate level problems and solutions by igor...
Partial differential equations, graduate level problems and solutions by igor...Partial differential equations, graduate level problems and solutions by igor...
Partial differential equations, graduate level problems and solutions by igor...Julio Banks
 

Similar to 258 lecnot2 (20)

Calculus Research Lab 3: Differential Equations!
Calculus Research Lab 3: Differential Equations!Calculus Research Lab 3: Differential Equations!
Calculus Research Lab 3: Differential Equations!
 
Applied Math
Applied MathApplied Math
Applied Math
 
Principle of Derivative Calculus - Differential Calculus - An Introduction by...
Principle of Derivative Calculus - Differential Calculus - An Introduction by...Principle of Derivative Calculus - Differential Calculus - An Introduction by...
Principle of Derivative Calculus - Differential Calculus - An Introduction by...
 
Differentiation
DifferentiationDifferentiation
Differentiation
 
Introdution to differential forms
Introdution to differential formsIntrodution to differential forms
Introdution to differential forms
 
Calculus of variations & solution manual russak
Calculus of variations & solution manual   russakCalculus of variations & solution manual   russak
Calculus of variations & solution manual russak
 
Ahlfors sol1
Ahlfors sol1Ahlfors sol1
Ahlfors sol1
 
Algorithmic Mathematics.
Algorithmic Mathematics.Algorithmic Mathematics.
Algorithmic Mathematics.
 
Discrete strucures
Discrete strucures Discrete strucures
Discrete strucures
 
SGC 2014 - Mathematical Sciences Tutorials
SGC 2014 - Mathematical Sciences TutorialsSGC 2014 - Mathematical Sciences Tutorials
SGC 2014 - Mathematical Sciences Tutorials
 
Introduction to Theory of elasticity and plasticity Att 6521
Introduction to Theory of elasticity and plasticity Att 6521Introduction to Theory of elasticity and plasticity Att 6521
Introduction to Theory of elasticity and plasticity Att 6521
 
SMA206_NOTES
SMA206_NOTESSMA206_NOTES
SMA206_NOTES
 
Math516 runde
Math516 rundeMath516 runde
Math516 runde
 
Abstract algebra
Abstract algebraAbstract algebra
Abstract algebra
 
abstract algebra
abstract algebraabstract algebra
abstract algebra
 
Dn
DnDn
Dn
 
LDP_Notes_2015.pdf
LDP_Notes_2015.pdfLDP_Notes_2015.pdf
LDP_Notes_2015.pdf
 
Partial differential equations, graduate level problems and solutions by igor...
Partial differential equations, graduate level problems and solutions by igor...Partial differential equations, graduate level problems and solutions by igor...
Partial differential equations, graduate level problems and solutions by igor...
 
Thesis 2015
Thesis 2015Thesis 2015
Thesis 2015
 
RiemannTEX
RiemannTEXRiemannTEX
RiemannTEX
 

Recently uploaded

Data governance with Unity Catalog Presentation
Data governance with Unity Catalog PresentationData governance with Unity Catalog Presentation
Data governance with Unity Catalog PresentationKnoldus Inc.
 
Moving Beyond Passwords: FIDO Paris Seminar.pdf
Moving Beyond Passwords: FIDO Paris Seminar.pdfMoving Beyond Passwords: FIDO Paris Seminar.pdf
Moving Beyond Passwords: FIDO Paris Seminar.pdfLoriGlavin3
 
A Journey Into the Emotions of Software Developers
A Journey Into the Emotions of Software DevelopersA Journey Into the Emotions of Software Developers
A Journey Into the Emotions of Software DevelopersNicole Novielli
 
Rise of the Machines: Known As Drones...
Rise of the Machines: Known As Drones...Rise of the Machines: Known As Drones...
Rise of the Machines: Known As Drones...Rick Flair
 
Emixa Mendix Meetup 11 April 2024 about Mendix Native development
Emixa Mendix Meetup 11 April 2024 about Mendix Native developmentEmixa Mendix Meetup 11 April 2024 about Mendix Native development
Emixa Mendix Meetup 11 April 2024 about Mendix Native developmentPim van der Noll
 
Manual 508 Accessibility Compliance Audit
Manual 508 Accessibility Compliance AuditManual 508 Accessibility Compliance Audit
Manual 508 Accessibility Compliance AuditSkynet Technologies
 
Generative Artificial Intelligence: How generative AI works.pdf
Generative Artificial Intelligence: How generative AI works.pdfGenerative Artificial Intelligence: How generative AI works.pdf
Generative Artificial Intelligence: How generative AI works.pdfIngrid Airi González
 
(How to Program) Paul Deitel, Harvey Deitel-Java How to Program, Early Object...
(How to Program) Paul Deitel, Harvey Deitel-Java How to Program, Early Object...(How to Program) Paul Deitel, Harvey Deitel-Java How to Program, Early Object...
(How to Program) Paul Deitel, Harvey Deitel-Java How to Program, Early Object...AliaaTarek5
 
Scale your database traffic with Read & Write split using MySQL Router
Scale your database traffic with Read & Write split using MySQL RouterScale your database traffic with Read & Write split using MySQL Router
Scale your database traffic with Read & Write split using MySQL RouterMydbops
 
Merck Moving Beyond Passwords: FIDO Paris Seminar.pptx
Merck Moving Beyond Passwords: FIDO Paris Seminar.pptxMerck Moving Beyond Passwords: FIDO Paris Seminar.pptx
Merck Moving Beyond Passwords: FIDO Paris Seminar.pptxLoriGlavin3
 
Arizona Broadband Policy Past, Present, and Future Presentation 3/25/24
Arizona Broadband Policy Past, Present, and Future Presentation 3/25/24Arizona Broadband Policy Past, Present, and Future Presentation 3/25/24
Arizona Broadband Policy Past, Present, and Future Presentation 3/25/24Mark Goldstein
 
Unleashing Real-time Insights with ClickHouse_ Navigating the Landscape in 20...
Unleashing Real-time Insights with ClickHouse_ Navigating the Landscape in 20...Unleashing Real-time Insights with ClickHouse_ Navigating the Landscape in 20...
Unleashing Real-time Insights with ClickHouse_ Navigating the Landscape in 20...Alkin Tezuysal
 
Sample pptx for embedding into website for demo
Sample pptx for embedding into website for demoSample pptx for embedding into website for demo
Sample pptx for embedding into website for demoHarshalMandlekar2
 
How AI, OpenAI, and ChatGPT impact business and software.
How AI, OpenAI, and ChatGPT impact business and software.How AI, OpenAI, and ChatGPT impact business and software.
How AI, OpenAI, and ChatGPT impact business and software.Curtis Poe
 
New from BookNet Canada for 2024: Loan Stars - Tech Forum 2024
New from BookNet Canada for 2024: Loan Stars - Tech Forum 2024New from BookNet Canada for 2024: Loan Stars - Tech Forum 2024
New from BookNet Canada for 2024: Loan Stars - Tech Forum 2024BookNet Canada
 
Take control of your SAP testing with UiPath Test Suite
Take control of your SAP testing with UiPath Test SuiteTake control of your SAP testing with UiPath Test Suite
Take control of your SAP testing with UiPath Test SuiteDianaGray10
 
So einfach geht modernes Roaming fuer Notes und Nomad.pdf
So einfach geht modernes Roaming fuer Notes und Nomad.pdfSo einfach geht modernes Roaming fuer Notes und Nomad.pdf
So einfach geht modernes Roaming fuer Notes und Nomad.pdfpanagenda
 
Time Series Foundation Models - current state and future directions
Time Series Foundation Models - current state and future directionsTime Series Foundation Models - current state and future directions
Time Series Foundation Models - current state and future directionsNathaniel Shimoni
 
How to write a Business Continuity Plan
How to write a Business Continuity PlanHow to write a Business Continuity Plan
How to write a Business Continuity PlanDatabarracks
 
Transcript: New from BookNet Canada for 2024: Loan Stars - Tech Forum 2024
Transcript: New from BookNet Canada for 2024: Loan Stars - Tech Forum 2024Transcript: New from BookNet Canada for 2024: Loan Stars - Tech Forum 2024
Transcript: New from BookNet Canada for 2024: Loan Stars - Tech Forum 2024BookNet Canada
 

Recently uploaded (20)

Data governance with Unity Catalog Presentation
Data governance with Unity Catalog PresentationData governance with Unity Catalog Presentation
Data governance with Unity Catalog Presentation
 
Moving Beyond Passwords: FIDO Paris Seminar.pdf
Moving Beyond Passwords: FIDO Paris Seminar.pdfMoving Beyond Passwords: FIDO Paris Seminar.pdf
Moving Beyond Passwords: FIDO Paris Seminar.pdf
 
A Journey Into the Emotions of Software Developers
A Journey Into the Emotions of Software DevelopersA Journey Into the Emotions of Software Developers
A Journey Into the Emotions of Software Developers
 
Rise of the Machines: Known As Drones...
Rise of the Machines: Known As Drones...Rise of the Machines: Known As Drones...
Rise of the Machines: Known As Drones...
 
Emixa Mendix Meetup 11 April 2024 about Mendix Native development
Emixa Mendix Meetup 11 April 2024 about Mendix Native developmentEmixa Mendix Meetup 11 April 2024 about Mendix Native development
Emixa Mendix Meetup 11 April 2024 about Mendix Native development
 
Manual 508 Accessibility Compliance Audit
Manual 508 Accessibility Compliance AuditManual 508 Accessibility Compliance Audit
Manual 508 Accessibility Compliance Audit
 
Generative Artificial Intelligence: How generative AI works.pdf
Generative Artificial Intelligence: How generative AI works.pdfGenerative Artificial Intelligence: How generative AI works.pdf
Generative Artificial Intelligence: How generative AI works.pdf
 
(How to Program) Paul Deitel, Harvey Deitel-Java How to Program, Early Object...
(How to Program) Paul Deitel, Harvey Deitel-Java How to Program, Early Object...(How to Program) Paul Deitel, Harvey Deitel-Java How to Program, Early Object...
(How to Program) Paul Deitel, Harvey Deitel-Java How to Program, Early Object...
 
Scale your database traffic with Read & Write split using MySQL Router
Scale your database traffic with Read & Write split using MySQL RouterScale your database traffic with Read & Write split using MySQL Router
Scale your database traffic with Read & Write split using MySQL Router
 
Merck Moving Beyond Passwords: FIDO Paris Seminar.pptx
Merck Moving Beyond Passwords: FIDO Paris Seminar.pptxMerck Moving Beyond Passwords: FIDO Paris Seminar.pptx
Merck Moving Beyond Passwords: FIDO Paris Seminar.pptx
 
Arizona Broadband Policy Past, Present, and Future Presentation 3/25/24
Arizona Broadband Policy Past, Present, and Future Presentation 3/25/24Arizona Broadband Policy Past, Present, and Future Presentation 3/25/24
Arizona Broadband Policy Past, Present, and Future Presentation 3/25/24
 
Unleashing Real-time Insights with ClickHouse_ Navigating the Landscape in 20...
Unleashing Real-time Insights with ClickHouse_ Navigating the Landscape in 20...Unleashing Real-time Insights with ClickHouse_ Navigating the Landscape in 20...
Unleashing Real-time Insights with ClickHouse_ Navigating the Landscape in 20...
 
Sample pptx for embedding into website for demo
Sample pptx for embedding into website for demoSample pptx for embedding into website for demo
Sample pptx for embedding into website for demo
 
How AI, OpenAI, and ChatGPT impact business and software.
How AI, OpenAI, and ChatGPT impact business and software.How AI, OpenAI, and ChatGPT impact business and software.
How AI, OpenAI, and ChatGPT impact business and software.
 
New from BookNet Canada for 2024: Loan Stars - Tech Forum 2024
New from BookNet Canada for 2024: Loan Stars - Tech Forum 2024New from BookNet Canada for 2024: Loan Stars - Tech Forum 2024
New from BookNet Canada for 2024: Loan Stars - Tech Forum 2024
 
Take control of your SAP testing with UiPath Test Suite
Take control of your SAP testing with UiPath Test SuiteTake control of your SAP testing with UiPath Test Suite
Take control of your SAP testing with UiPath Test Suite
 
So einfach geht modernes Roaming fuer Notes und Nomad.pdf
So einfach geht modernes Roaming fuer Notes und Nomad.pdfSo einfach geht modernes Roaming fuer Notes und Nomad.pdf
So einfach geht modernes Roaming fuer Notes und Nomad.pdf
 
Time Series Foundation Models - current state and future directions
Time Series Foundation Models - current state and future directionsTime Series Foundation Models - current state and future directions
Time Series Foundation Models - current state and future directions
 
How to write a Business Continuity Plan
How to write a Business Continuity PlanHow to write a Business Continuity Plan
How to write a Business Continuity Plan
 
Transcript: New from BookNet Canada for 2024: Loan Stars - Tech Forum 2024
Transcript: New from BookNet Canada for 2024: Loan Stars - Tech Forum 2024Transcript: New from BookNet Canada for 2024: Loan Stars - Tech Forum 2024
Transcript: New from BookNet Canada for 2024: Loan Stars - Tech Forum 2024
 

258 lecnot2

  • 1. ISBN: Copyright Notice: Lecture Notes on Differential Equations Emre Sermutlu
  • 2. ˙ To my wife Nurten and my daughters Ilayda and Alara
  • 3. vi CONTENTS 5.2 Differential Operators . . . . . . . . . . . . . . . . . . . . . . . 34 5.3 Homogeneous Equations . . . . . . . . . . . . . . . . . . . . . 35 5.4 Nonhomogeneous Equations . . . . . . . . . . . . . . . . . . . 37 6 Series Solutions Contents 41 6.1 6.2 1 First Order ODE 1.1 Definitions . . . . . . . . 1.2 Mathematical Modeling 1.3 Separable Equations . . 1.4 Transformations . . . . . ix . . . . . . . . . . . . 2 Exact Equations 2.1 Exact Equations . . . . . . . 2.2 Integrating Factors . . . . . . 2.3 Linear First Order Equations 2.4 Bernoulli Equation . . . . . . . . . . . . . . . . . . . . . . . . . . . . . . . . . . . . . . . . . . . . . . . . . . . . . . . . . . . . . . . . . . . . . . . . . . . . . . . . . . . . . . . . . . . . . . . . . . . . . . . . . . . . . . . . . . . . . . . . . . . . . . . . . . . . . . . . . . . . . . . . . . 1 1 3 3 5 . . . . 9 9 11 13 14 Classification of Points . . . . . . . . . . . . . . . . . . . . . . 43 6.3 Preface Power Series . . . . . . . . . . . . . . . . . . . . . . . . . . . . 41 Power Series Method . . . . . . . . . . . . . . . . . . . . . . . 43 7 Frobenius’ Method 49 7.1 An Extension of Power Series Method . . . . . . . . . . . . . . 49 7.2 Examples . . . . . . . . . . . . . . . . . . . . . . . . . . . . . 50 8 Laplace Transform I 57 8.1 Definition . . . . . . . . . . . . . . . . . . . . . . . . . . . . . 57 8.2 Properties . . . . . . . . . . . . . . . . . . . . . . . . . . . . . 59 8.3 Initial Value Problems . . . . . . . . . . . . . . . . . . . . . . 61 9 Laplace Transform II 69 . . . . . . . . . . . . . . . . . . . . . . . . . . . . . . . . . . . . . . . . . . . . . . . . . . . . . . . . . . . . . . . . . . . . . . . . 17 17 19 20 22 4 Nonhomogeneous Equations 25 4.1 General and Particular Solutions . . . . . . . . . . . . . . . . 25 4.2 Method of Undetermined Coefficients . . . . . . . . . . . . . . 27 4.3 Method of Variation of Parameters . . . . . . . . . . . . . . . 29 5 Higher Order Equations 33 5.1 Linear Equations of Order n . . . . . . . . . . . . . . . . . . . 33 v Convolution . . . . . . . . . . . . . . . . . . . . . . . . . . . . 69 9.2 Unit Step Function . . . . . . . . . . . . . . . . . . . . . . . . 72 9.3 3 Second Order Equations 3.1 Linear Differential Equations . 3.2 Reduction of Order . . . . . . 3.3 Constant Coefficients . . . . . 3.4 Cauchy-Euler Equation . . . . 9.1 Differentiation of Transforms . . . . . . . . . . . . . . . . . . . 73 9.4 Partial Fractions Expansion . . . . . . . . . . . . . . . . . . . 74 9.5 Applications . . . . . . . . . . . . . . . . . . . . . . . . . . . . 75 10 Fourier Analysis I 81 10.1 Fourier Series . . . . . . . . . . . . . . . . . . . . . . . . . . . 81 10.2 Convergence of Fourier Series . . . . . . . . . . . . . . . . . . 84 10.3 Parseval’s Identity . . . . . . . . . . . . . . . . . . . . . . . . 85 11 Fourier Analysis II 91 11.1 Fourier Cosine and Sine Series . . . . . . . . . . . . . . . . . . 91 11.2 Complex Fourier Series . . . . . . . . . . . . . . . . . . . . . . 94 11.3 Fourier Integral Representation . . . . . . . . . . . . . . . . . 96
  • 4. CONTENTS vii 12 Partial Differential Equations 12.1 Introduction . . . . . . . . . . . . . . . . . . . . . . . . . . . 12.2 Modeling a Vibrating String . . . . . . . . . . . . . . . . . . 12.3 Method of Separation of Variables . . . . . . . . . . . . . . . 101 . 101 . 103 . 104 13 Heat Equation 13.1 Modeling Heat Flow . . . . . . . . 13.2 Homogeneous Boundary Conditions 13.3 Nonzero Boundary Conditions . . . 13.4 Two Dimensional Problems . . . . 111 . 111 . 113 . 115 . 117 . . . . . . . . . . . . . . . . . . . . . . . . . . . . . . . . . . . . . . . . . . . . . . . . . . . . . . . . 14 Laplace Equation 121 14.1 Rectangular Coordinates . . . . . . . . . . . . . . . . . . . . . 121 14.2 Polar Coordinates . . . . . . . . . . . . . . . . . . . . . . . . . 126 To the Student 133 References 135 Index 137
  • 5. Preface This set of lecture notes for ordinary and partial differential equations grew out of the course Engineering Mathematics I have taught at Cankaya Univer¸ sity since 1999. It is a one-semester course for second year students. The main audience for this text, of course, is students. Presentation is user-friendly. There are more examples and fewer theorems than usual. The material is based on a solid background in calculus. The student is assumed to be familiar with algebra, trigonometry, functions and graphs, series, differentiation, and most importantly, integration techniques of various kinds. It is my (and my students’) sad experience that if you cannot differentiate and integrate, you cannot solve differential equations. Knowledge of Linear Algebra, except for the determinants of a simple nature, is not assumed. There are 14 chapters. Each chapter can be covered in one week. After a summary of methods and solved exercises, there are a number of end of chapter exercises with answers. The exercises that take exceptionally longer times are marked with a star. ( ) Nobody can learn how to solve problems by watching someone else solve problems. So I advise the students to try each problem on their own. I would like to thank all my students who helped me write this book by the valuable feedback they provided. In particular, special thanks are for ˙ Nuh Co¸kun, Nevrez Imamo˜lu, Nilg¨n Din¸arslan and I¸ıl Lelo˜lu who have s g u c s g made a very extensive and meticulous check of the whole manuscript. You may send all kinds of comments, suggestions and error reports to sermutlu@cankaya.edu.tr. Assist. Prof. Dr. Emre Sermutlu ix
  • 6. 2 CHAPTER 1. FIRST ORDER ODE are partial differential equations. (Partial Differential Equations are usually much more difficult) Order: The order of a differential equation is the order of the highest derivative that occurs in the equation. A first order differential equation contains y , y and x so it is of the form F (x, y, y ) = 0 or y = f (x, y). For example, the following differential equations are first order: Chapter 1 First Order Differential Equations y + x2 y = ex xy = (1 + y 2 ) y 2 = 4xy While these are second order: The subject of differential equations is an important part of applied mathematics. Many real life problems can be formulated as differential equations. In this chapter we will first learn the basic concepts and classification of differential equations, then we will see where they come from and how the simplest ones are solved. The concepts and techniques of calculus, especially integration, will be necessary to understand differential equations. 1.1 Definitions Ordinary Differential Equation: An ordinary differential equation is an equation that contains derivatives of an unknown function y(x). Partial Differential Equation: A partial differential equation is a differential equation involving an unknown function of two or more variables, like u(x, y). For example, y − 4y + y = 0 y 2 + 1 = x2 y + sin x are ordinary differential equations. uxx + uyy = 0 u2 + u2 = ln u x y 1 y − x2 y + y = 1 + sin x y + 6yy = x3 General and Particular Solutions: A general solution of a differential equation involves arbitrary constants. In a particular solution, these constants are determined using initial values. As an example, consider the differential equation y = 2x. y = x2 + c is a general solution , y = x2 + 4 is a particular solution . Example 1.1 Find the general solution of the differential equation y = 0. Then find the particular solution that satisfies y(0) = 5, y (0) = 3. y = 0 ⇒ y = c ⇒ y = cx + d. This is the general solution. y (0) = 3 ⇒ c = 3, y(0) = 5 ⇒ d = 5 Therefore y = 3x + 5. This is the particular solution. Explicit and Implicit Solutions: y = f (x) is an explicit solution, F (x, y) = 0 is an implicit solution. We have to solve equations to obtain y for a given x in implicit solutions, whereas it is straightforward for explicit solutions. For example, y = e4x is an explicit solution of the equation y = 4y. x3 + y 3 = 1 is an implicit solution of the equation y 2 y + x2 = 0
  • 7. 1.2. MATHEMATICAL MODELING 1.2 3 Mathematical Modeling 4 CHAPTER 1. FIRST ORDER ODE Example 1.4 Solve the initial value problem y + y 2 xex = 0, y(0) = 2 Differential equations are the natural tools to formulate, solve and understand many engineering and scientific systems. The mathematical models of most of the simple systems are differential equations. y = −y 2 xex dP = αP dt ex dx Example 1.3 The downward acceleration of an object in free fall is g. Find the height as a function of time if the initial height is y0 and initial speed is v0 . 2= d2 y = −g dt2 1 = xex − ex + c y ⇒ c= 1 xex − ex + 3 2 3 2 Example 1.5 Find the general solution of the differential equation y + y 2 = 1. dy + y2 = 1 dx 1 y = − gt2 + v0 t + y0 2 ⇒ dy = 1 − y2 dx dy = 1 − y2 Separable Equations If we can separate x and y in a first order differential equation and put them to different sides as g(y)dy = f (x)dx, it is called a separable equation. We can find the solution by integrating both sides. (Don’t forget the integration constant!) f (x)dx + c ⇒ xex 1 −1 + c y= dy = −gt + v0 dt g(y)dy = xex dx 1 − ex + c This is the general solution. Now we will use the condition y(0) = 2 to determine the constant c. y= where P0 = P (0) 1.3 dy = xex dx y2 Using integration by parts, we have u = x, dv = ex dx, du = dx, v = ex therefore 1 = xex − y P = P0 eαt − dy = y2 − Example 1.2 The rate of growth of a population is proportional to itself. Find the population as a function of time. ⇒ (1.1) 1 2 1 1 + 1−y 1+y ⇒ dy = dx 1 − y2 dx dy = 1 1+y ln =x+c 2 1−y 1+y = e2x+2c 1−y dx
  • 8. 1.4. TRANSFORMATIONS 5 6 CHAPTER 1. FIRST ORDER ODE After some algebra, we obtain y= u3/2 = 3 ln x + c 3/2 ke2x − 1 ke2x + 1 where k = e2c u= Example 1.6 Solve the initial value problem y = x3 e−y , y(1) = 0. ey dy = ey = 9 ln x + c1 2 e0 = Let’s use the substitution u = x + y. Then, 1 +c 4 y = u − x, ⇒ 1.4 x4 3 + 4 4 (u + 6)dx = (−u − 3)(du − dx) 3dx = (−u − 3)du Sometimes a change of variables simplifies a differential equation just as y substitutions simplify integrals. For example if y = f , the substitution x y u = will make the new equation separable. x y +3 x x . y √ u du = 3dx = 3x = − 3x = − If y = ux, then y = u x + u and u x + u = u + 3 ux=3 dy = du − dx and the equation can be expressed in terms of u and x. Transformations Example 1.7 Solve y = 2/3 Example 1.8 Solve the differential equation (x + y + 6)dx = (−x − y − 3)dy. 3 c= 4 y = ln 2/3 x3 dx x4 +c 4 ⇒ y(1) = 0 y=x 9 ln x + c1 2 1 u 3dx x 1 u This is an implicit solution. (−u − 3) du u2 − 3u + c 2 (x + y)2 − 3(x + y) + c 2
  • 9. EXERCISES 7 Exercises CHAPTER 1. FIRST ORDER ODE Answers Solve the following differential equations. 1) y 3 y + x3 = 0 2) y + 4x3 y 2 = 0 y 3) xy = x + y Hint: y = f x 4) (x2 + y 2 ) dx + xydy = 0 Hint: y = f 8 y x 1) x4 + y 4 = c 1 2) y = 4 x +c 3) y = x(ln |x| + c) c x2 4) y 2 = 2 − x 2 5) y = − ln c + 2 5) y = xey−x 1 + ln x 6) y = 4y 3 7) y = 3x2 sec2 y 8) y = y(y + 1) 9) y + 2y = y 2 + 1 10) (1 + y 2 )dx + x2 dy = 0 y x 12) y = eax+by 13) y = x2 y 2 − 2y 2 + x2 − 2 14) y = − 2x + y x Solve the following initial value problems: 15) (y 2 + 5xy + 9x2 )dx + x2 dy = 0, y(1) = −4 16) y 3 y + x3 = 0, y(0) = 1 17) y = −2xy, y(0) = 3 18) y = 1 + 4y 2 , y(0) = 0 19) (x2 + 1)1/2 y = xy 3 , y(0) = 2 20) dx x x2 = − , x(0) = 1 dt 5 25 2 6) y 4 = x ln x + c 7) 2y + sin 2y = 4x3 + c ex 8) y = c − ex 9) y = 1 − 1 x+c 10) y = tan c + 11) y = a e−x 2 1 x 11) y = cxa 12) eax e−by + =c a b 13) y = tan x3 − 2x + c 3 14) y = −x + c x 15) y = 16) 17) 18) 19) x − 3x ln x − 1 x4 + y 4 = 1 2 y = 3e−x 1 y = 2 tan 2x √ y = ( 9 − 2 x2 + 1)−1/2 4 20) x = 5et/5 4 + et/5
  • 10. 10 CHAPTER 2. EXACT EQUATIONS So, the solution of this equation is very simple, if du is zero, u must be a constant, therefore x4 + x2 y 2 + y 4 = c ∂N ∂M = is necessary and sufficient for the ∂y ∂x equation M (x, y)dx + N (x, y)dy = 0 to be exact. Method of Solution: To solve M dx + N dy = 0, Theorem 2.1: The condition Chapter 2 • Check for Exactness Exact and Linear Differential Equations • If the equation is exact, find u by integrating either M or N . u= M dx + k(y) or u = N dy + l(x) Note that we have arbitrary functions as integration constants. In this chapter, we will learn how to recognize and solve three different types of equations: Exact, linear, and Bernoulli. All of them are first order equations, therefore we expect a single integration constant in the solution. At this stage it seems like there’s a special trick for every different kind of question. You will gain familiarity with exercise and experience. • Determine the arbitrary functions using the original equation. The solution is u(x, y) = c Example 2.1 Solve the equation 3y 2 dx + (3y 2 + 6xy)dy = 0. Let’s check for exactness first. 2.1 ∂(3y 2 ) = 6y, ∂y Exact Equations A first order differential equation of the form M (x, y)dx + N (x, y)dy = 0 The equation is exact. (2.1) u(x, y) = is called an exact differential equation if there exists a function u(x, y) such that ∂u ∂u = M, =N (2.2) ∂x ∂y In other words, du = M dx + N dy, so M dx + N dy is a total differential. For example, the equation (4x3 + 2xy 2 )dx + (4y 3 + 2x2 y)dy = 0 is exact, and u = x4 + x2 y 2 + y 4 9 ∂(3y 2 + 6xy) = 6y ∂x 3y 2 dx + k(y) = 3y 2 x + k(y) ∂u = 6yx + k (y) = 3y 2 + 6xy ∂y k (y) = 3y 2 ⇒ k(y) = y 3 We do not need an integration constant here because u(x, y) = c already contains one u(x, y) = 3y 2 x + y 3 = c
  • 11. 2.2. INTEGRATING FACTORS 2.2 11 Integrating Factors 12 CHAPTER 2. EXACT EQUATIONS But this equation is more difficult than the one we started with. If we make a simplifying assumption that F is a function of one variable only, we can solve for F and obtain the following theorem: Consider the equation Theorem 2.2: Consider the equation P dx + Qdy = 0. Define P dx + Qdy = 0 (2.3) that is not exact. If it becomes exact after multiplying by F , i.e. if F P dx + F Qdy = 0 R= (2.4) is exact, then F is called an integrating factor. (Note that P, Q and F are functions of x and y) 1 For example, ydx − xdy = 0 is not exact, but F = 2 is an integrating x factor. Example 2.2 Solve (2xex − y 2 )dx + 2ydy = 0. Use F = e−x . a) If R depends only on x, then F (x) = exp factor. ˜ b) If R depends only on y, then F (y) = exp factor. ∂Q ∂P − ∂x ∂y R= ˜ R(y)dy is an integrating 8x2 y + 2 − 6x2 y − 1 2x2 y + 1 1 = 3 = 3y + x 2x 2x y + x x F (x) = e R(x)dx = eln x = x Multiply the equation by x to obtain the exact equation ∂(2x − y 2 e−x ) ∂(2ye−x ) = −2ye−x , = −2ye−x ∂y ∂x (4x3 y 2 + 2yx)dx + (2x4 y + x2 )dy = 0 Now the equation is exact. We can solve it as we did the previous example and obtain the result u(x, y) = x2 + y 2 e−x = c How To Find an Integrating Factor: Let P dx+Qdy = 0 be a differential equation that is not exact, and let F = F (x, y) be an integrating factor. By definition, (2.5) (2.6) R(x) dx is an integrating Example 2.3 Solve (4x2 y 2 + 2y)dx + (2x3 y + x)dy = 0 (2x − y 2 e−x )dx + 2ye−x dy = 0 Fy P + F P y = Fx Q + F Qx 1 ˜ and R = P The equation is not exact. The equation is not exact. Let’s multiply both sides by e−x . The new equation is: ⇒ ∂P ∂Q − ∂y ∂x ∂(2x3 y + x) ∂(4x2 y 2 + 2y) = 8x2 y + 2, = 6x2 y + 1 ∂y ∂x ∂(2y) ∂(2xex − y 2 ) = −2y, =0 ∂y ∂x (F P )y = (F Q)x 1 Q (4x3 y 2 + 2yx) dx + k(y) = x4 y 2 + yx2 + k(y) ∂u = 2x4 y + x2 + k (y) = 2x4 y + x2 ∂y u(x, y) = x4 y 2 + x2 y = c ⇒ k(y) = 0
  • 12. 2.3. LINEAR FIRST ORDER EQUATIONS 2.3 13 Linear First Order Equations 2.4 If a first order differential equation can be written in the form y + p(x)y = r(x) CHAPTER 2. EXACT EQUATIONS Bernoulli Equation The equation y + p(x)y = g(x)y a (2.7) it is called a linear differential equation. If r(x) = 0, the equation is homogeneous, otherwise it is nonhomogeneous. We can express the equation (2.7) as [p(x)y − r(x)]dx + dy = 0. This is not exact but it has an integrating factor: R = p(x), F = e 14 p dx is called Bernoulli equation. It is nonlinear. Nonlinear equations are usually much more difficult than linear ones, but Bernoulli equation is an exception. It can be linearized by the substitution u(x) = [y(x)]1−a (2.14) (2.8) Then, we can solve it as other linear equations. Method of Solution: • Given a first order linear equation, express it in the following form: y + p(x)y = r(x) p dx y +e p dx py = re 2 p dx y − ex 2x y= 3 3xy 2 p(x) dx to obtain e Example 2.5 Solve the equation (2.9) • Multiply both sides by the integrating factor F (x) = exp (2.10) Here a = −2 therefore u = y 1−(−2) = y 3 ⇒ u = 3y 2 y Multiplying both sides of the equation by 3y 2 we obtain 2 • Express the left hand side as a single parenthesis. e p dx y = re p dx 3y 2 y − 2xy 3 = (2.11) y(x) = e−h eh r dx + c (2.12) 2 ex x ⇒ u − 2xu = e −2x dx = e−x 2 2 Multiplying both sides by e−x , we get 2 p dx. 2 e−x u − 2xe−x u = Example 2.4 Solve y + 4y = 1 2 The integrating factor is F = e equation by e4x to obtain 4 dx (e−x u) = 4x = e . Multiply both sides of the 2 e4x y + 4e4x y =e4x e−x u = ln x + c ⇒ ⇒ 1 y = + ce−4x 4 1 x 1 x u = (ln x + c)ex (e4x y) =e4x e4x e4x y = +c 4 ex x This equation is linear. Its integrating factor is • Integrate both sides. Don’t forget the integration constant. The solution is: where h = (2.13) y = (ln x + c)ex 2 1/3 2
  • 13. EXERCISES 15 16 Exercises Answers Solve the following differential equations. (Find an integrating factor if necessary) 1) (yex + xyex + 1)dx + xex dy = 0 2) (2r + 2 cos θ)dr − 2r sin θdθ = 0 3) (sin xy + xy cos xy)dx + (x2 cos xy)dy = 0 4) 2 cos ydx = sin ydy 5) 5dx − ey−x dy = 0 6) (2xy + 3x2 y 6 ) dx + (4x2 + 9x3 y 5 ) dy = 0 7) (3xey + 2y) dx + (x2 ey + x) dy = 0 1 5 8) y + y = x x 9) y + 1 1 y= x ln x ln x c − 1 e−x x r2 + 2r cos θ = c x sin xy = c F = e2x , e2x cos y = c F = ex , 5ex − ey = c F = y 3 , x 2 y 4 + x3 y 9 = c F = x, x3 ey + x2 y = c c 1 y= + 5 5 x 1) y = 2) 3) 4) 5) 6) 7) 8) 9) y = x+c ln x 10) y = −1 + x4 12) y = 4 − 5e− 4 Reduce to linear form and solve the following equations: 2 sin x 1/2 13) y − 4y tan x = y cos3 x x y 5 ln x 4/5 25 y= y 15) y + x x5 13) y = 14) y = c − ln cos x cos2 x 1 2 2 − x + ce−2x 15) y = y 1 =− 9 3 x xy x ln x − x + c x5 16) y = 14) y + y = − 1 c + 4 8 x x 19) x = y −2 2 1/4 1 3 cosh 3y + c Hint: x ↔ y 20) y = 20) 2xyy + (x − 1)y 2 = x2 ex 5 17) y = arcsin[c(x − 1)] 1 c + 3 18) F = y, x = 2y y tan y 17) y = x−1 18) y 2 dx + (3xy − 1)dy = 0 19) y (sinh 3y − 2xy) = y c cos x 11) y = x4 cos x + c cos x 10) y − y tan x = tan x 11) y + y tan x = 4x3 cos x 12) y + x3 y = 4x3 , y(0) = −1 16) y + CHAPTER 2. EXACT EQUATIONS Hint: z = y 2 cxe−x + 1 xex 2
  • 14. 18 Chapter 3 Second Order Homogeneous Differential Equations For first order equations, concepts from calculus and some extensions were sufficient. Now we are starting second order equations and we will learn many new ideas, like reduction of order, linear independence and superposition of solutions. Many differential equations in applied science and engineering are second order and linear. If in addition they have constant coefficients, we can solve them easily, as explained in this chapter and the next. For nonconstant coefficients, we will have limited success. 3.1 Linear Differential Equations If we can express a second order differential equation in the form y + p(x)y + q(x)y = r(x) (3.1) it is called linear. Otherwise, it is nonlinear. Consider a linear differential equation. If r(x) = 0 it is called homogeneous, otherwise it is called nonhomogeneous. Some examples are: y + y 2 = x2 y Nonlinear sin xy + cos xy = 4 tan x Linear Nonhomogeneous x2 y + y = 0 Linear Homogeneous 17 CHAPTER 3. SECOND ORDER EQUATIONS Linear Combination: A linear combination of y1 , y2 is y = c1 y1 + c2 y2 . Theorem 3.1: For a homogeneous linear differential equation any linear combination of solutions is again a solution. The above result does NOT hold for nonhomogeneous equations. For example, both y = sin x and y = cos x are solutions to y + y = 0, so is y = 2 sin x + 5 cos x. Both y = sin x + x and y = cos x + x are solutions to y + y = x, but y = sin x + cos x + 2x is not. This is a very important property of linear homogeneous equations, called superposition. It means we can multiply a solution by any number, or add two solutions, and obtain a new solution. Linear Independence: Two functions y1 , y2 are linearly independent if c1 y1 + c2 y2 = 0 ⇒ c1 = 0, c2 = 0. Otherwise they are linearly dependent. (One is a multiple of the other). For example, ex and e2x are linearly independent. ex and 2ex are linearly dependent. General Solution and Basis: Given a second order, linear, homogeneous differential equation, the general solution is: y = c1 y1 + c2 y2 (3.2) where y1 , y2 are linearly independent. The set {y1 , y2 } is called a basis, or a fundamental set of the differential equation. As an illustration, consider the equation x2 y − 5xy + 8y = 0. You can easily check that y = x2 is a solution. (We will see how to find it in the last section) Therefore 2x2 , 7x2 or −x2 are also solutions. But all these are linearly dependent. We expect a second, linearly independent solution, and this is y = x4 . A combination of solutions is also a solution, so y = x2 + x4 or y = 10x2 − 5x4 are also solutions. Therefore the general solution is y = c1 x 2 + c2 x 4 and the basis of solutions is {x2 , x4 }. (3.3)
  • 15. 3.2. REDUCTION OF ORDER 3.2 19 Reduction of Order 3.3 If we know one solution of a second order homogeneous differential equation, we can find the second solution by the method of reduction of order. Consider the differential equation y + py + qy = 0 (3.4) Suppose one solution y1 is known, then set y2 = uy1 and insert in the equation. The result will be y1 u + (2y1 + py1 )u + (y1 + py1 + qy1 )u = 0 (3.5) (3.6) This is still second order, but if we set w = u , we will obtain a first order equation: y1 w + (2y1 + py1 )w = 0 (3.7) Solving this, we can find w, then u and then y2 . Example 3.1 Given that y1 = x2 is a solution of CHAPTER 3. SECOND ORDER EQUATIONS Homogeneous Equations with Constant Coefficients Up to now we have studied the theoretical aspects of the solution of linear homogeneous differential equations. Now we will see how to solve the constant coefficient equation y + ay + by = 0 in practice. We have the sum of a function and its derivatives equal to zero, so the derivatives must have the same form as the function. Therefore we expect the function to be eλx . If we insert this in the equation, we obtain: λ2 + aλ + b = 0 But y1 is a solution, so the last term is canceled. So we have y1 u + (2y1 + py1 )u = 0 20 (3.8) This is called the characteristic equation of the homogeneous differential equation y + ay + by = 0. If we solve the characteristic equation, we will see three different possibilities: Two real roots, double real root and complex conjugate roots. Two Real Roots: The general solution is y = c1 eλ1 x + c2 eλ2 x (3.9) 2 x y − 3xy + 4y = 0 Example 3.2 Solve y − 3y − 10y = 0 find a second linearly independent solution. Let y2 = ux2 . Then y2 = u x2 + 2xu and 2 Try y = eλx . The characteristic equation is λ2 − 3λ − 10 = 0 with solution λ1 = 5, λ = −2, so the general solution is y = c1 e5x + c2 e−2x y2 = u x + 4xu + 2u Inserting these in the equation, we obtain x4 u + x3 u = 0 If w = u then 1 x4 w + x3 w = 0 or w + w = 0 x 1 This linear first order equation gives w = , therefore u = ln x and x 2 y2 = x ln x Example 3.3 Solve the initial value problem y −y = 0, y(0) = 2, y (0) = 4 We start with y = eλx as usual. The characteristic equation is λ2 − 1 = 0. Therefore λ = ±1. The general solution is: y = c1 ex + c2 e−x Now, we have to use the initial values to determine the constants. y(0) = 2 ⇒ c1 + c2 = 2 and y (0) = 4 ⇒ c1 − c2 = 4. By solving this system, we obtain c1 = 3, c2 = −1 so the particular solution is: y = 3ex − e−x
  • 16. 3.3. CONSTANT COEFFICIENTS 21 Double Real Root: One solution is eλx but we know that a second order equation must have two independent solutions. Let’s use the method of reduction of order to find the second solution. y − 2ay + a2 y = 0 ⇒ y1 = eax (3.10) 22 CHAPTER 3. SECOND ORDER EQUATIONS 3.4 Cauchy-Euler Equation The equation x2 y + axy + by = 0 is called the Cauchy-Euler equation. By inspection, we can easily see that the solution must be a power of x. Let’s substitute y = xr in the equation and try to determine r. We will obtain Let’s insert y2 = ueax in the equation. r(r − 1)xr + arxr + bxr = 0 ax ax e u + (2a − 2a)e u = 0 Obviously, u = 0 therefore u = c1 + c2 x. The general solution is y = c1 eλx + c2 xeλx (3.12) Example 3.4 Solve y + 2y + y = 0 λx r2 + (a − 1)r + b = 0 (3.11) (3.17) (3.18) This is called the auxiliary equation. Once again, we have three different cases according to the types of roots. The general solution is given as follows: • Two real roots 2 y = e . The characteristic equation is λ + 2λ + 1 = 0. Its solution is the double root λ = −1, therefore the general solution is y = c1 e−x + c2 xe−x (3.13) This can be proved using Taylor series expansions. If the solution of the characteristic equation is λ1 = α + iβ, λ2 = α − iβ y = c1 e (cos βx + i sin βx) + c2 e αx (cos βx − i sin βx) (3.14) (3.15) By choosing new constants A, B, we can express this as y=e αx (A cos βx + B sin βx) y = c1 xr + c2 xr ln x (3.20) • Complex conjugate roots where r1 , r2 = r ± si y = xr [c1 cos(s ln x) + c2 sin(s ln x)] then the general solution of the differential equation will be αx (3.19) • Double real root Complex Conjugate Roots: We need the complex exponentials for this case. Euler’s formula is eix = cos x + i sin x y = c1 xr1 + c2 xr2 (3.21) Example 3.6 Solve x2 y + 2xy − 6y = 0 Insert y = xr . Auxiliary equation is r2 + r − 6 = 0. The roots are r = 2, r = −3 therefore y = c1 x2 + c2 x−3 (3.16) Example 3.5 Solve y − 4y + 29y = 0. Example 3.7 Solve x2 y − 9xy + 25y = 0 y = eλx . The characteristic equation is λ2 −4λ+29 = 0. Therefore λ = 2±5i. The general solution is Insert y = xr . Auxiliary equation is r2 − 10r + 25 = 0. Auxiliary equation has the double root r = 5 therefore the general solution is y = e2x (A cos 5x + B sin 5x) y = c1 x5 + c2 x5 ln x
  • 17. EXERCISES 23 24 Exercises CHAPTER 3. SECOND ORDER EQUATIONS Answers 1) 2) 3) 4) 5) Are the following sets linearly independent? 1) {x4 , x8 } 2) {sin x, sin2 x} 3) {ln(x5 ), ln x} Use reduction of order to find a second linearly independent solution: 4) x2 (ln x − 1) y − xy + y = 0, y1 = x 1 5) x2 ln x y + (2x ln x − x)y − y = 0, y1 = x 6) y + 3 tan x y + (3 tan2 x + 1)y = 0, y1 = cos x Yes Yes No y2 = ln x y2 = ln x − 1 6) y2 = sin x cos x 7) y = (1 + x)e−x 1 8) y = c1 e−2x + c2 e− 2 x Solve the following equations: 7) y + 2y + y = 0, y(0) = 1, y (0) = 0 9) y = e8x 5 8) y + y + y = 0 2 10) y = c1 e−12x + c2 xe−12x 9) y − 64y = 0, y(0) = 1, 10) y + 24y + 144y = 0 y (0) = 8 11) y = 4e−x + 3xe−x 7 11) y + 2y + y = 0, y(−1) = e, y(1) = e 12) 5y − 8y + 5y = 0 π2 13) y + 2y + 1 + y = 0, y(0) = 1, y (0) = −1 4 14) y − 2y + 2y = 0, y(π) = 0, y(−π) = 0 15) xy + y = 0 16) x2 y − 3xy + 5y = 0 17) x2 y − 10xy + 18y = 0 18) x2 y − 13xy + 49y = 0 19) Show that y1 = u and y2 = u y − v u +2 v u y + 12) y = e0.8x [A cos(0.6x) + B sin(0.6x)] 13) y = e−x cos 14) y = ex sin x 15) y = c1 + c2 ln x 16) y = x2 [c1 cos(ln x) + c2 sin(ln x)] vdx are solutions of the equation vu u2 u +2 2 − vu u u π x 2 y=0 20) Show that y1 = u and y2 = v are solutions of the equation (uv − vu )y + (vu − uv )y + (u v − v u )y = 0 17) y = c1 x2 + c2 x9 18) y = c1 x7 + c2 x7 ln x
  • 18. 26 CHAPTER 4. NONHOMOGENEOUS EQUATIONS the nonhomogeneous one. The general solution is of the form y = yh + yp (4.3) Example 4.1 Find the general solution of y − 3y + 2y = 2x − 3 using yp = x. Chapter 4 Let’s solve y − 3y + 2y = 0 first. Let yh = eλx . Then λ2 − 3λ + 2 = 0 Second Order Nonhomogeneous Equations which means λ = 2 or λ = 1. The homogenous solution is yh = c1 ex + c2 e2x therefore the general solution is: y = x + c1 ex + c2 e2x In this chapter we will start to solve the nonhomogeneous equations, and see that we will need the homogeneous solutions we found in the previous chapter. Of the two methods we will learn, undetermined coefficients is simpler, but it can be applied to a restricted class of problems. Variation of parameters is more general but involves more calculations. 4.1 Consider the nonhomogeneous equation (4.1) Let yp be a solution of this equation. Now consider the corresponding homogeneous equation y + p(x)y + q(x)y = 0 (4.2) Let yh be the general solution of this one. If we add yh and yp , the result will still be a solution for the nonhomogeneous equation, and it must be the general solution because yh contains two arbitrary constants. This interesting property means that we need the homogeneous equation when we are solving 25 The solution of y = 0 is simply yh = c1 x + c2 , therefore the general solution must be y = − cos x + c1 x + c2 As you can see, once we have a particular solution, the rest is straightforward, but how can we find yp for a given equation? Example 4.3 Find a particular solution of the following differential equations. Try the suggested functions. (Success not guaranteed!) General and Particular Solutions y + p(x)y + q(x)y = r(x) Example 4.2 Find the general solution of y = cos x using yp = − cos x. a) y + y = ex , Try yp = Aex b) y − y = ex , Try yp = Aex c) y + 2y + y = x Try yp = Ax + B d) y + 2y = x Try yp = Ax + B e) y + 2y + y = 2 cos x Try yp = A cos x and yp = A cos x + B sin x As you can see, some of the suggestions work and some do not. yp is usually similar to r(x). We can summarize our findings as: • Start with a set of functions that contains not only r(x), but also all derivatives of r(x). • If one of the terms of yp candidate occurs in yh , there is a problem.
  • 19. 4.2. METHOD OF UNDETERMINED COEFFICIENTS 4.2 27 Method of Undetermined Coefficients 28 CHAPTER 4. NONHOMOGENEOUS EQUATIONS The homogeneous equation is 3y + 10y + 3y = 0 To solve the constant coefficient equation d2 y dy + a + by = r(x) 2 dx dx (4.4) • Solve the corresponding homogeneous equation, find yh . • Find a candidate for yp using the following table: Term in r(x) Choice for yp xn eax cos bx or sin bx xn eax xn cos bx or xn sin bx An xn + · · · + A1 x + A0 Aeax A cos bx + B sin bx (An xn + · · · + A1 x + A0 )eax (An xn + · · · + A0 ) cos bx +(Bn xn + · · · + B0 ) sin bx eax cos bx or eax sin bx Aeax cos bx + Beax sin bx xn eax cos bx or xn eax sin bx (An xn + · · · + A0 )eax cos bx +(Bn xn + · · · + B0 )eax sin bx (You don’t have to memorize the table. Just note that the choice consists of r(x) and all its derivatives) • If your choice for yp occurs in yh , you have to change it. Multiply it by x if the solution corresponds to a single root, by x2 if it is a double root. • Find the constants in yp by inserting it in the equation. • The general solution is y = yp + yh Note that this method works only for constant coefficient equations, and only when r(x) is relatively simple. Its solution is yh = c1 e−3x + c2 e−x/3 To find a particular solution, let’s try yp = Ax + B. Inserting this in the equation, we obtain: 10A + 3Ax + 3B = 9x Therefore, A = 3, B = −10. The particular solution is: yp = 3x − 10 The general solution is: y = c1 e−3x + c2 e−x/3 + 3x − 10 Example 4.5 Find the general solution of y − 4y + 4y = e2x The solution of the associated homogeneous equation y − 4y + 4y = 0 is yh = c1 e2x + c2 xe2x Our candidate for yp is yp = Ae2x . But this is already in the yh so we have to change it. If we multiply by x, we will obtain Axe2x but this is also in yh . Therefore we have to multiply by x2 . So our choice for yp is yp = Ax2 e2x . Now we have to determine A by inserting in the equation. yp = 2Ax2 e2x + 2Axe2x yp = 4Ax2 e2x + 8Axe2x + 2Ae2x Example 4.4 Find the general solution of the equation 3y + 10y + 3y = 9x 4Ax2 e2x + 8Axe2x + 2Ae2x − 4(2Ax2 e2x + 2Axe2x ) + 4Ax2 e2x = e2x
  • 20. 4.3. METHOD OF VARIATION OF PARAMETERS 2Ae 2x =e 2x ⇒ 29 CHAPTER 4. NONHOMOGENEOUS EQUATIONS Therefore the particular solution is 1 1 A = , yp = x2 e2x 2 2 yp (x) = −y1 1 y = yh + yp = c1 e2x + c2 xe2x + x2 e2x 2 4.3 30 y1 r dx aW (4.11) e−x Example 4.6 Find the general solution of y + 2y + y = √ x yh = c1 e−x + c2 xe−x Method of Variation of Parameters Consider the linear second order nonhomogeneous differential equation a(x)y + b(x)y + c(x)y = r(x) y2 r dx + y2 aW W = (4.5) e−x xe−x −x −x −e e − xe−x yp = −e−x If a(x), b(x) and c(x) are not constants, or if r(x) is not among the functions given in the table, we can not use the method of undetermined coefficients. In this case, the variation of parameters can be used if we know the homogeneous solution. Let yh = c1 y1 + c2 y2 be the solution of the associated homogeneous equation a(x)y + b(x)y + c(x)y = 0 (4.6) Let us express the particular solution as: xe−x e−x √ dx + xe−x e−2x x yp = −e−x √ x dx + xe−x = e−2x e−x e−x √ dx e−2x x 1 √ dx x 4 x3/2 x1/2 + xe−x = e−x x3/2 3/2 1/2 3 4 y = yh + yp = c1 e−x + c2 xe−x + e−x x3/2 3 2 Example 4.7 Find the general solution of x y − 5xy + 8y = x5 yp = −e−x We can find the homogeneous solution of the Cauchy-Euler equation as: yp = v1 (x)y1 + v2 (x)y2 (4.7) yh = c1 x4 + c2 x2 There are two unknowns, so we may impose an extra condition. Let’s choose v1 y1 + v2 y2 = 0 for simplicity. Inserting yp in the equation, we obtain r a = 0 v1 y1 + v2 y2 = v1 y1 + v2 y2 −y2 r , aW v2 = y1 r aW (4.8) (4.9) where W is the Wronskian W = y1 y2 y1 y2 = y1 y2 − y2 y1 x4 x 2 4x3 2x = −2x5 Therefore the particular solution is The solution to this linear system is v1 = W = (4.10) yp (x) = −x4 1 yp (x) = x4 2 1 5 yp (x) = x 3 The general solution is x2 x5 dx + x2 x2 (−2x5 ) 1 dx − x2 x2 dx 2 1 y = c1 x 4 + c2 x 2 + x 5 3 x4 x5 dx x2 (−2x5 )
  • 21. EXERCISES 31 Exercises 1) 2) 3) 4) 5) 6) 32 CHAPTER 4. NONHOMOGENEOUS EQUATIONS Answers Find the general solution of the following differential equations y + 4y = x cos x y − 18y + 81y = e9x y = −4x cos 2x − 4 cos 2x − 8x sin 2x − 8 sin 2x y + 3y − 18y = 9 sinh 3x y + 16y = x2 + 2x y − 2y + y = x2 ex 7) 2x2 y − xy + y = 1 x 1) y = c1 sin 2x + c2 cos 2x + 1 x cos x + 2 sin x 3 9 1 2) y = c1 e9x + c2 xe9x + x2 e9x 2 3) y = c1 + c2 x + x cos 2x + 3 cos 2x + 2x sin 2x + sin 2x 1 1 4) y = c1 e3x + c2 e−6x + e−3x + xe3x 4 2 1 2 1 1 5) y = c1 sin 4x + c2 cos 4x + x + x − 16 8 128 1 6) y = c1 ex + c2 xex + x4 ex 12 √ 1 7) y = c1 x + c2 x + 6x 8) x y + xy − 4y = x ln x 9) y − 8y + 16y = 16x 10) y = x3 11) y + 7y + 12y = e2x + x 1 1 x2 8) y = c1 x2 + c2 x−2 + x2 ln2 x − x2 ln x + 8 16 64 1 9) y = c1 e4x + c2 xe4x + x + 2 x5 10) y = + c1 + c2 x 20 12) y + 12y + 36y = 100 cos 2x 11) y = c1 e−3x + c2 e−4x + 2 2 13) y + 9y = ex + cos 3x + 2 sin 3x 1 7 1 2x e + x− 30 12 144 12) y = c1 e−6x + c2 xe−6x + 2 cos 2x + 3 sin 2x 2 1 x 1 1 e − x cos 3x + x sin 3x 10 3 6 14) y + 10y + 16y = e−2x 13) y = c1 cos 3x + c2 sin 3x + 15) y − 4y + 53y = (53x)2 1 14) y = c1 e−2x + c2 e−8x + xe−2x 6 16) y + y = (x2 + 1)e3x 15) y = e2x (c1 cos 7x + c2 sin 7x) + 53x2 + 8x − 17) y + y = csc x 16) y = e3x (0.1x2 − 0.12x + 0.152) + c1 sin x + c2 cos x 17) y = c1 sin x + c2 cos x − x cos x + sin x ln | sin x| 18) y = c1 sin x + c2 cos x − cos x ln | sec x + tan x| − sin x ln | csc x + cot x| 18) y + y = csc x sec x 19) y − 4y + 4y = e2x ln x x 20) y − 2y + y = e2x (ex + 1)2 19) y = c1 e2x + c2 xe2x + xe2x 74 53 (ln x)2 − ln x + 1 2 20) y = c1 ex + c2 xex + ex ln(1 + ex )
  • 22. 34 CHAPTER 5. HIGHER ORDER EQUATIONS means that all the constants c1 , c2 , . . . , cn are zero, then this set of functions is linearly independent. Otherwise, they are dependent. For example, the functions x, x2 , x3 are linearly independent. The functions cos2 x, sin2 x, cos 2x are not. Given n functions, we can check their linear dependence by calculating the Wronskian. The Wronskian is defined as Chapter 5 Higher Order Equations W (y1 , y2 , . . . , yn ) = y1 y1 . . . (n−1) y1 In this chapter, we will generalize our results about second order equations to higher orders. The basic ideas are the same. We still need the homogeneous solution to find the general nonhomogeneous solution. We will extend the two methods, undetermined coefficients and variation of parameters, to higher dimensions and this will naturally involve many more terms and constants in the solution. We also need some new notation to express nth derivatives easily. ... ... yn yn . . . (5.4) (n−1) . . . yn and the functions are linearly dependent if and only if W = 0 at some point. 5.2 Differential Operators We can denote differentiation with respect to x by the symbol D Dy = dy =y, dx D2 y = d2 y =y dx2 (5.5) etc. A differential operator is 5.1 An n form Linear Equations of Order n th L = a0 Dn + a1 Dn−1 + · · · + an−1 D + an order differential equation is called linear if it can be written in the a0 (x) dn y dn−1 y dy + a1 (x) n−1 + · · · + an−1 (x) + an (x)y = r(x) n dx dx dx (5.1) and nonlinear if it is not linear.(Note that a0 = 0) If the coefficients a0 (x), a1 (x), . . . an (x) are continuous, then the equation has exactly n linearly independent solutions. The general solution is y = c1 y1 + c2 y2 + · · · + cn yn (5.2) We will only work with operators where coefficients are constant. We can add, multiply, expand and factor constant coefficient differential operators using common rules of algebra. In this respect, they are like polynomials. So, the following expressions are all equivalent: (D − 2)(D − 3)y = (D − 3)(D − 2)y = (D2 − 5D + 6)y = y − 5y + 6y Let’s apply some simple operators to selected functions: (D − 2)ex = Dex − 2ex = ex − 2ex = −ex Linear Independence: If c1 y1 + c2 y2 + · · · + cn yn = 0 33 (5.3) (5.6)
  • 23. 5.3. HOMOGENEOUS EQUATIONS 35 36 CHAPTER 5. HIGHER ORDER EQUATIONS (D − 2)e2x = De2x − 2e2x = 2e2x − 2e2x = 0 Now we are in a position to solve very complicated-looking homogeneous equations. (D − 2)2 xe2x = (D − 2)(D − 2)xe2x = (D − 2)(e2x + 2xe2x − 2xe2x ) = (D − 2)e2x = 0 Method of Solution: • Express the given equation using operator notation (D notation). (D2 − 4) sin(2x) = (D − 2)(D + 2) sin(2x) = (D − 2)(2 cos(2x) + 2 sin(2x)) = −4 sin(2x) + 4 cos(2x) − 4 cos(2x) − 4 sin(2x) = −8 sin 2x • Factor the polynomial. • Find the solution for each component. • Add the components to obtain the general solution. 5.3 Homogeneous Equations Example 5.1 Find the general solution of y (4) − 7y + y − 7y = 0. Based on the examples in the previous section, we can easily see that: The general solution of the equation (D − a)n y = 0 is y = eax (c0 + c1 x + . . . + cn−1 xn−1 ) In operator notation, we have (D4 − 7D3 + D2 − 7D)y = 0 (5.7) Factoring this, we obtain if a is real. Some special cases are: D(D − 7)(D2 + 1)y = 0 Dn y = 0 ⇒ y = c0 + c1 x + . . . + cn−1 xn−1 (D − a)y = 0 ⇒ y = eax (D − a)2 y = 0 ⇒ y = c1 eax + c2 xeax We know that (5.8) We can extend these results to the case of complex roots. If z = a + ib is a root of the characteristic polynomial, then so is z = a − ib. (Why?) Consider the equation (D − a − ib)n (D − a + ib)n y = (D2 − 2aD + a2 + b2 )n y = 0 (5.9) The solution is y = eax cos bx(c0 + c1 x + . . . + cn−1 xn−1 ) +eax sin bx(k0 + k1 x + . . . + kn−1 xn−1 ) (5.10) 2 (D + b )y = 0 ⇒ Therefore the general solution is y = c1 + c2 e7x + c3 sin x + c4 cos x Note that the equation is fourth order and the solution has four arbitrary constants. Example 5.2 Solve D3 (D − 2)(D − 3)2 (D2 + 4)y = 0. Using the same method, we find: A special case is obtained if a = 0. 2 Dy = 0 ⇒ y = c (D − 7)y = 0 ⇒ y = ce7x (D2 + 1)y = 0 ⇒ y = c1 sin x + c2 cos x y = c1 cos bx + c2 sin bx (5.11) y = c1 + c2 x + c3 x2 + c4 e2x + c5 e3x + c6 e3x x + c7 cos 2x + c8 sin 2x
  • 24. 5.4. NONHOMOGENEOUS EQUATIONS 5.4 37 Nonhomogeneous Equations 38 CHAPTER 5. HIGHER ORDER EQUATIONS Then we will proceed similarly to simplify the steps. Eventually, we will obtain the system In this section, we will generalize the methods of undetermined coefficients and variation of parameters to nth order equations. Undetermined Coefficients: Method of undetermined coefficients is the same as given on page 27. We will use the same table, but this time the modification rule is more general. It should be: • In case one of the terms of yp occurs in yh , multiply it by xk where k is the smallest integer which will eliminate any duplication between yp and yh . v1 y1 v1 y1 . . . (n−1) v1 y1 (n) v1 y1 yp = y1 The homogeneous solution is yh = (c0 + c1 x + c2 x2 + c3 x3 )ex . According to the table, we should choose yp as Aex + Bxex , but this already occurs in the homogeneous solution. Multiplying by x, x2 , x3 are not enough, so, we should multiply by x4 . yp = Ax4 ex + Bx5 ex Inserting this in the equation, we obtain: 24Aex + 120Bxex = xex 1 5 x xe 120 n−1 d y d y dy + a1 (x) n−1 + · · · + an−1 (x) + an (x)y = r(x) n dx dx dx (5.12) Let the homogeneous solution be yh = c1 y1 + · · · + cn yn Then the particular solution is yp = v1 y1 + · · · + vn yn Here, vi are functions of x. Since we have n functions, we can impose n − 1 conditions on them. The first condition will be v1 y1 + · · · + vn yn = 0 = = + + (n−1) vn y n (n) vn yn = = 0 0 . . . (5.14) 0 r(x) a0 (x) v1 dx + · · · + yn vn dx (5.15) x3 y − 6x2 y + 15xy − 15y = 8x6 We can find the homogeneous solution yh = c1 x + c2 x3 + c3 x5 using our method for Cauchy-Euler equations. Then, the particular solution will be yp = xv1 + x3 v2 + x5 v3 . Using the above equations, we obtain the system xv1 + x3 v2 + x5 v3 = 0 v1 + 3x2 v2 + 5x4 v3 = 0 6xv2 + 20x3 v3 = 8x3 Variation of Parameters: The idea is the same as in second order equations, but there are more unknowns to find and more integrals to evaluate. Consider a0 (x) vn yn vn yn . . . Example 5.4 Find the general solution of Therefore A = 0, B = 1/120 and the general solution is n + ··· + ··· + + Then, we will solve this linear system to find vi , and integrate them to obtain yp . Example 5.3 Solve the equation (D − 1)4 y = xex . y = (c0 + c1 x + c2 x2 + c3 x3 )ex + + ··· + ··· (5.13) The solution of this system is v1 = x4 , v2 = −2x2 , v3 = 1 therefore the particular solution is yp = x x4 dx + x3 (−2x2 ) dx + x5 and the general solution is y = c1 x + c2 x 3 + c3 x 5 + 8 6 x 15 dx = 8 6 x 15
  • 25. EXERCISES 39 Exercises CHAPTER 5. HIGHER ORDER EQUATIONS Answers 1) y = c0 + c1 x + c2 x2 + c3 x3 + c4 x4 1) D5 y = 0 2) y = c1 ex + c2 xex + c3 x2 ex 2) (D − 1)3 y = 0 3) y − 4y + 13y = 0 4) (D − 2)2 (D + 3)3 y = 0 5) (D2 + 2)3 y = 0 d4 y d2 y + 5 2 + 4y = 0 dx4 dx 7) (D2 + 9)2 (D2 − 9)2 y = 0 6) 4 40 3 2 dy dy dy −2 3 +2 2 =0 4 dx dx dx 9) y − 3y + 12y − 10y = 0 8) 3) y = c1 e2x cos 3x + c2 e2x sin 3x + c3 4) y = c1 e2x + c2 xe2x + c3 e−3x + c4 xe−3x + c5 x2 e−3x √ √ √ √ 5) y = c1 cos 2x + c2 sin 2x + c3 x cos 2x + c4 x sin 2x √ √ + c5 x2 cos 2x + c6 x2 sin 2x 6) y = c1 cos 2x + c2 sin 2x + c3 cos x + c4 sin x 7) y = c1 e3x + c2 xe3x + c3 e−3x + c4 xe−3x + c5 cos 3x + c6 sin 3x + c7 x cos 3x + c8 x sin 3x 8) y = c1 + c2 x + c3 ex cos x + c4 ex sin x 10) (D2 + 2D + 17)2 y = 0 9) y = c1 ex + c2 ex cos 3x + c3 ex sin 3x 11) (D4 + 2D2 + 1)y = x2 10) y = c1 e−x sin 4x + c2 e−x cos 4x + c3 xe−x sin 4x + c4 xe−x cos 4x 12) (D3 + 2D2 − D − 2)y = 1 − 4x3 11) y = c1 cos x + c2 sin x + c3 x cos x + c4 x sin x + x2 − 4 √ √ √ 13) (2D4 + 4D3 + 8D2 )y = 40e−x [ 3 sin( 3x) + 3 cos( 3x)] 14) (D3 − 4D2 + 5D − 2)y = 4 cos x + sin x 15) (D3 − 9D)y = 8xex 12) y = c1 ex + c2 e−x + c3 e−2x + 2x3 − 3x2 + 15x − 8 √ √ √ 13) y = c1 + c2 x + c3 e−x cos 3x + c4 e−x sin 3x + 5xe−x cos 3x 14) y = c1 ex + c2 xex + c3 e2x + 0.2 cos x + 0.9 sin x 3 15) y = c1 + c2 e3x + c3 e−3x + ex − xex 4
  • 26. 42 CHAPTER 6. SERIES SOLUTIONS term wise, i.e. ∞ (an ± bn )(x − x0 )n f (x) ± g(x) = n=0 • In the interval of convergence, the series can be multiplied or divided to give another power series. Chapter 6 ∞ cn (x − x0 )n f (x)g(x) = Series Solutions n=0 where cn = a0 bn + a1 bn−1 + · · · + an b0 If none of the methods we have studied up to now works for a differential equation, we may use power series. This is usually the only choice if the solution cannot be expressed in terms of the elementary functions. (That is, exponential, logarithmic, trigonometric and polynomial functions). If the solution can be expressed as a power series, in other words, if it is analytic, this method will work. But it takes time and patience to reach the solution. Remember, we are dealing with infinitely many coefficients! 6.1 Power Series • In the interval of convergence, derivatives and integrals of f (x) can be found by term wise differentiation and integration, for example ∞ n an (x − x0 )n−1 f (x) = a1 + 2a2 (x − x0 ) + · · · = n=1 (n) (x • The series ∞ f n! 0 ) (x − x0 )n is called the Taylor Series of the funcn=0 tion f (x). The function f (x) is called analytic if its Taylor series converges. Examples of some common power series are: ∞ Let’s remember some facts about the series ex = ∞ an (x − x0 )n = a0 + a1 (x − x0 ) + a2 (x − x0 )2 + · · · n=0 (6.1) n=0 ∞ cos x = n=0 from calculus. ∞ sin x = • There is a nonnegative number ρ, called the radius of convergence, such that the series converges absolutely for |x − x0 | < ρ and diverges for |x − x0 | > ρ . The series defines a function f (x) = ∞ an (x − x0 )n n=0 in its interval of convergence. n=0 1 = 1−x 41 =1+x+ (−1)n x2n 2n! =1− x2 + ··· 2! x2 x4 + − ··· 2! 4! (−1)n x2n+1 x3 x5 =x− + − ··· (2n + 1)! 3! 5! ∞ xn = 1 + x + x2 + · · · (−1)n+1 xn n =x− n=0 ∞ ln(1 + x) = • In the interval of convergence, the series can be added or subtracted xn n! n=1 x2 x3 + − ··· 2 3
  • 27. 6.2. CLASSIFICATION OF POINTS 6.2 43 Classification of Points 44 CHAPTER 6. SERIES SOLUTIONS Example 6.1 Solve y + 2xy + 2y = 0 around x0 = 0. First we should classify the point. Obviously, x = 0 is an ordinary point, so we can use power series method. Consider the equation R(x)y + P (x)y + Q(x)y = 0 (6.2) ∞ ∞ n y= If both of the functions P (x) , R(x) Q(x) R(x) an x , y = n=0 (6.3) are analytic at x = x0 , then the point x0 is an ordinary point. Otherwise, x0 is a singular point. Suppose that x0 is a singular point of the above equation. If both of the functions Q(x) P (x) , (x − x0 )2 (6.4) (x − x0 ) R(x) R(x) are analytic at x = x0 , then the point x0 is called a regular singular point. Otherwise, x0 is an irregular singular point. For example, the functions 1+x+x2 , sin x, ex (1+x4 ) cos x are all analytic cos x 1 ex 1 + x2 at x = 0. But, the functions , , , are not. x x x x3 We will use power series method around ordinary points and Frobenius’ method around regular singular points. We will not consider irregular singular points. ∞ nan x n−1 n(n − 1)an xn−2 , y = n=1 n=2 Inserting these in the equation, we obtain ∞ ∞ n(n − 1)an x n−2 ∞ + 2x n=2 nan x n−1 an x n = 0 +2 n=1 ∞ n=0 ∞ ∞ n(n − 1)an xn−2 + n=2 2nan xn + n=1 2an xn = 0 n=0 To equate the powers of x, let us replace n by n + 2 in the first sigma. (n → n + 2) ∞ ∞ ∞ n=1 n=0 2an xn = 0 2nan xn + (n + 2)(n + 1)an+2 xn + n=0 Now we can express the equation using a single sigma, but we should start the index from n = 1. Therefore we have to write n = 0 terms separately. ∞ [(n + 2)(n + 1)an+2 + (2n + 2)an ] xn = 0 2a2 + 2a0 + n=1 6.3 Power Series Method If x0 is an ordinary point of the equation R(x)y + P (x)y + Q(x)y = 0, then the general solution is ∞ an (x − x0 )n y= −2(n + 1) −2 an = an (n + 2)(n + 1) (n + 2) This is called the recursion relation. Using it, we can find all the constants in terms of a0 and a1 . a2 = −a0 , an+2 = (6.5) 2 1 a4 = − a2 = a0 4 2 2 1 a6 = − a4 = − a0 6 6 2 a3 = − a1 , 3 2 4 a 5 = − a3 = a1 5 15 n=0 The coefficients an can be found by inserting y in the equation and setting the coefficients of all powers to zero. Two coefficients (Usually a0 and a1 ) must be arbitrary, others must be defined in terms of them. We expect two linearly independent solutions because the equation is second order linear. We can find as many coefficients as we want in this way. Collecting them together, the solution is : 1 1 y = a0 1 − x 2 + x 4 − x 6 + · · · 2 6 2 4 + a1 x − x 3 + x 5 + · · · 3 15
  • 28. 6.3. POWER SERIES METHOD 45 In most applications, we want a solution close to 0, therefore we can neglect the higher order terms of the series. Remark: Sometimes we can express the solution in closed form (in terms of elementary functions rather than an infinite summation) as in the next example: Example 6.2 Solve (x − 1)y + 2y = 0 around x0 = 0. Once again, first we should classify the given point. The function analytic at x = 0, therefore x = 0 is an ordinary point. ∞ ∞ ∞ an x n , y = y= n=0 2 is x−1 nan xn−1 , y = n=1 n(n − 1)an xn−2 n=2 Inserting these in the equation, we obtain ∞ (x − 1) ∞ n(n − 1)an x n−2 nan xn−1 = 0 +2 n=2 n=1 ∞ ∞ ∞ n(n − 1)an xn−2 + n(n − 1)an xn−1 − n=1 n=2 n=2 2nan xn−1 = 0 46 CHAPTER 6. SERIES SOLUTIONS Exercises Find the general solution of the following differential equations in the form of series. Find solutions around the origin (use x0 = 0). Write the solution in closed form if possible. 1) (1 − x2 )y − 2xy = 0 2) y + x4 y + 4x3 y = 0 3) (2 + x3 )y + 6x2 y + 6xy = 0 4) (1 + x2 )y − xy − 3y = 0 5) (1 + 2x2 )y + xy + 2y = 0 6) y − xy + ky = 0 7) (1 + x2 )y − 4xy + 6y = 0 8) (1 − 2x2 )y + (2x + 4x3 )y − (2 + 4x2 )y = 0 9) (1 + 8x2 )y − 16y = 0 10) y + x2 y = 0 The following equations give certain special functions that are very important in applications. Solve them for n = 1, 2, 3 around origin. Find polynomial solutions only. To equate the powers of x, let us replace n by n+1 in the second summation. ∞ ∞ n(n − 1)an x n−1 − n=2 ∞ (n + 1)nan+1 x n−1 + n=1 2nan x n−1 =0 n=1 Now we can express the equation using a single sigma. 11) 12) 13) 14) (1 − x2 )y − 2xy + n(n + 1)y = 0 y − 2xy + 2ny = 0 xy + (1 − x)y + ny = 0 (1 − x2 )y − xy + n2 y = 0 (Legendre’s Equation) (Hermite’s Equation) (Laguerre’s Equation) (Chebyshev’s Equation) ∞ [(n(n − 1) + 2n)an − n(n + 1)an+1 ] xn−1 = 0 (−2a2 + 2a1 ) + n=2 a2 = a1 , an+1 = n2 − n + 2n an for n n(n + 1) 2 So the recursion relation is: an+1 = an All the coefficients are equal to a1 , except a0 . We have no information about it, so it must be arbitrary. Therefore, the solution is: y = a0 + a1 x + x 2 + x 3 + · · · x y = a0 + a1 1−x Solve the following initial value problems. Find the solution around the point where initial conditions are given. 15) 16) 17) 18) xy + (x + 1)y − 2y = 0, y + 2xy − 4y = 0, 4y + 3xy − 6y = 0, (x2 − 4x + 7)y + y = 0, x0 x0 x0 x0 = −1, = 0, =0 =2 y(−1) = 1, y(0) = 1, y(0) = 4, y(2) = 4, y (−1) = 0 y (0) = 0 y (0) = 0 y (2) = 10 19) Find the recursion relation for (p + x2 )y + (1 − q − r)xy + qry = 0 around x = 0. (Here p, q, r are real numbers, p = 0) 20) Solve (1 + ax2 )y + bxy + cy = 0 around x0 = 0
  • 29. EXERCISES 47 Answers x3 x5 + + ··· 3 5 n=3 x3 x6 x9 + − + ··· 2 4 8 a1 x + 3 1+ x 2 3) y = a0 1 − a0 3 1+ x 2 3 1 3 4) y = a0 1 + x2 + x4 − x6 + · · · 2 8 16 2 2 5) y = a0 1 − x2 + x4 − x6 + · · · 3 3 6) y = a0 1 − +a1 x − 2 + a1 x + x3 3 17 1 + a1 x − x 3 + x 5 + · · · 2 40 k − 1 3 (k − 1)(k − 3) 5 (k − 1)(k − 3)(k − 5) 7 x + x − x + ··· 3! 5! 7! x3 3 x4 x6 1+x + + + ··· 2 6 2 10) y = a0 1 − ⇒ ⇒ n=3 ⇒ x8 x4 + + ··· 12 672 + a1 x y = a1 x y = a0 (1 − 3x2 ) 5 y = a1 (x − x3 ) 3 + a1 x − ⇒ 14) n = 1 n=2 ⇒ ⇒ ⇒ x5 x9 + + ··· 20 1440 y = a1 x y = a0 (1 − 2x2 ) 2 y = a1 (x − x3 ) 3 y = a0 (1 − x) 1 y = a0 (1 − 2x + x2 ) 2 3 1 y = a0 (1 − 3x + x2 − x3 ) 2 6 y = a1 x y = a0 (1 − 2x2 ) 4 y = a1 (x − x3 ) 3 1 1 15) y = 1 − (x + 1)2 − (x + 1)3 − (x + 1)4 − · · · 3 6 16) y = 1 + 2x2 17) y = 4 + 3x2 1 1 18) y = 4 1 − (x − 2)2 + (x − 2)4 + · · · 6 72 1 7 + 10 (x − 2) − (x − 2)3 + (x − 2)5 + · · · 18 1080 19) an+2 = − 8 64 9) y = a0 (1 + 8x2 ) + a1 x + x3 − x5 + · · · 3 15 11) n = 1 n=2 OR k 2 k(k − 2) 4 k(k − 2)(k − 4) 6 x + x − x + ··· 2! 4! 6! 7) y = a0 (1 − 3x2 ) + a1 x − 8) y = a0 x4 x7 x10 + − + ··· 2 4 8 ⇒ n=3 + a1 x − ⇒ n=3 x5 x10 x15 + − + ··· 5 5 · 10 5 · 10 · 15 x11 x16 x6 + − + ··· x− 6 6 · 11 6 · 11 · 16 ⇒ n=2 1 1+x ln 2 1−x ⇒ ⇒ 13) n = 1 OR y = a0 + a1 2) y = a0 1 − y= CHAPTER 6. SERIES SOLUTIONS 12) n = 1 n=2 1) y = a0 + a1 x + +a1 48 (n − q)(n − r) an p(n + 2)(n + 1) x4 x2 + c(2a + 2b + c) 2 4! x6 + ··· − c(2a + 2b + c)(12a + 4b + c) 6! x3 x5 + a1 x − (b + c) + (b + c)(6a + 3b + c) 3! 5! x7 −(b + c)(6a + 3b + c)(20a + 5b + c) + ··· 7! 20) y = a0 1 − c
  • 30. 50 CHAPTER 7. FROBENIUS’ METHOD Case 2 - Equal roots: A basis of solutions is ∞ ∞ an x n , y 1 = xr y2 = y1 ln x + xr bn x n (7.3) n=1 n=0 Case 3 - Roots differing by an integer: A basis of solutions is Chapter 7 ∞ ∞ an xn , y 1 = xr 1 bn x n y2 = ky1 ln x + xr2 n=1 (7.4) n=0 In this chapter, we will extend the methods of the previous chapter to regular singular points. The calculations will be considerably longer, but the basic ideas are the same. The classification of the given point is necessary to make a choice of methods. where r1 − r2 = N > 0 (r1 is the greater root) and k may or may not be zero. In all three cases, there is at least one relatively simple solution of the form y = xr ∞ an xn . The equation is second order, so there must be a n=0 second linearly independent solution. In Cases 2 and 3, it may be difficult to find the second solution. You may use the method of reduction of order. This is convenient especially if y1 is simple enough. Alternatively, you may use the above formulas directly, and determine bn one by one using the an and the equation. 7.1 7.2 Frobenius’ Method An Extension of Power Series Method Suppose x0 is a regular singular point. For simplicity, assume x0 = 0. Then p(x) q(x) the differential equation can be written as y + y + 2 y = 0 where x x p(x) and q(x) are analytic. We can try a solution of the form ∞ y = xr an x n (7.1) n=0 The equation corresponding to the lowest power xr−2 , in other words r(r − 1) + p0 r + q0 = 0 is called the indicial equation, where p0 = p(0), and q0 = q(0). Now we can find r, insert it in the series formula, and proceed as we did in the previous chapter. We can classify the solutions according to the roots of the indicial equation. Case 1 - Distinct roots not differing by an integer: A basis of solutions is ∞ ∞ y 1 = xr 1 an x n , y2 = xr2 bn x n n=0 n=0 49 (7.2) Examples Example 7.1 Solve 4xy + 2y + y = 0 around x0 = 0. 2 First we should classify the given point. The function 4x is not analytic at x = 0 therefore x = 0 is a singular point. We should make a further test to determine whether it is regular or not. x2 The functions 2x and 4x are analytic therefore x = 0 is a R.S.P., we can 4x use the method of Frobenius. ∞ ∞ an xn+r , y = y= n=0 ∞ (n + r)an xn+r−1 , y = n=0 (n + r)(n + r − 1)an xn+r−2 n=0 Note that the summation for the derivatives still starts from 0, because r does not have to be an integer. This is an important difference between methods of power series and Frobenius. Inserting these in the equation, we obtain ∞ ∞ (n + r)(n + r − 1)an xn+r−2 + 2 4x n=0 ∞ (n + r)an xn+r−1 + n=0 an xn+r = 0 n=0
  • 31. 7.2. EXAMPLES 51 ∞ ∞ 4(n + r)(n + r − 1)an x n+r−1 ∞ + n=0 2(n + r)an x n=0 n+r−1 an xn+r = 0 + 52 CHAPTER 7. FROBENIUS’ METHOD For simplicity, we may choose a0 = 1. Then an = We want to equate the powers of x, so n → n + 1 in the first two terms. ∞ ∞ ∞ 4(n + r + 1)(n + r)an+1 xn+r + n=−1 2(n + r + 1)an+1 xn+r + n=−1 an xn+r = 0 Therefore the second solution is : ∞ n=0 Now we can express the equation using a single sigma, but the index of the common sigma must start from n = 0. Therefore we have to write n = −1 terms separately. [4r(r−1)+2r]a0 xr−1 + {[4(n + r + 1)(n + r) + 2(n + r + 1)]an+1 + an } xn+r = 0 n=0 We know that a0 = 0, therefore 4r2 − 2r = 0. This is the indicial equation. Its solutions are r = 0, r = 1 . Therefore this is Case 1. 2 If r = 0, the recursion relation is n=0 The general solution is y = c1 y1 + c2 y2 2 First we should classify the given point. The function x x−x is not analytic 2 at x = 0 therefore x = 0 is a singular point. The functions x − 1 and 1 + x are analytic at x = 0 therefore x = 0 is a R.S.P., we can use the method of Frobenius. Evaluating the derivatives of y and inserting them in the equation, we obtain −1 1 an 4(n + 1)(n + 2 ) an+1 = ∞ For simplicity, we may choose a0 = 1. Then n=0 ∞ − (n + r)an x (−1) 2n! y1 = n=0 n=0 + an x n+r ∞ an xn+r+1 = 0 + n=0 n=0 ∞ ∞ (n + r)(n + r − 1)an x n=0 ∞ n n (n + r)an xn+r+1 + Let’s replace n by n − 1 in the second and fifth terms. Therefore the first solution is: ∞ n+r ∞ n+r n=0 n an = ∞ (n + r)(n + r − 1)an x a0 a1 a0 a2 a0 a1 = − , a 2 = − , a3 = − ,... 3 = 5 = − 2 4! 6! 4.2. 2 4.3. 2 − √ (−1) x = cos x 2n! 1 If r = , the recursion relation is 2 a1 = − √ (−1)n xn = sin x (2n + 1)! y2 = x1/2 Example 7.2 Solve x2 y + (x2 − x)y + (1 + x)y = 0 around x0 = 0. ∞ an+1 = (−1)n (2n + 1)! n=0 −1 −an an = (2n + 3)(2n + 2) 4(n + 3 )(n + 1) 2 a0 a1 a0 a2 a0 , a2 = − = , a3 = − = − ,... 3.2 5.4 5! 7.6 7! n+r n=1 ∞ (n + r)an x n=0 n+r an x + n=0 (n + r − 1)an−1 xn+r + n+r ∞ an−1 xn+r = 0 + n=1 [r2 − 2r + 1]a0 xr + ∞ {[(n + r)(n + r − 1) − (n + r) + 1]an + [(n + r − 1) + 1]an−1 } xn+r = 0 n=1 The indicial equation is r2 − 2r + 1 = 0 ⇒ r = 1 (double root). Therefore this is Case 2. The recursion relation is an = − n+1 an−1 n2
  • 32. 7.2. EXAMPLES 53 54 CHAPTER 7. FROBENIUS’ METHOD Exercises For simplicity, let a0 = 1. Then 3 3 4 2 a1 = −2, a2 = − a1 = , a3 = − a2 = − 4 2 9 3 Find two linearly independent solutions of the following differential equations in the form of series. Find solutions around the origin (use x0 = 0). Write the solution in closed form if possible. 1) 2x2 y − xy + (1 + x)y = 0 Therefore the first solution is : 3 2 y1 = x 1 − 2x + x2 − x3 + · · · 2 3 2) 2xy + (1 + x)y − 2y = 0 To find the second solution, we will use reduction of order. Let y2 = uy1 . Inserting y2 in the equation, we obtain 3) (x2 + 2x)y + (3x + 1)y + y = 0 4) xy − y − 4x3 y = 0 2 2 2 x y1 u + (2x y1 − xy1 + x y1 )u = 0 Let w = u then −2 To evaluate the integral u = 1−x+ w= u= 1 x ⇒ w= 8) (2x2 + 2x)y − y − 4y = 0 xe−x 2 y1 9) 2x2 y + (2x2 − x)y + y = 0 1 w dx we need to find 2 . This is also a series. y1 3 2 1 − 2x + x2 − x3 + · · · 2 3 xe−x =x 2 y1 7) x2 y + (x2 − x)y + y = 0 y1 1 + − 1 dx y1 x ln w = −2 ln y1 + ln x − x w= 6) 3x2 y + (−10x − 3x2 )y + (14 + 4x)y = 0 1 y w + 2 1 − +1 w =0 y1 x dw = w 1 1 = 2 2 y1 x 5) xy + y − xy = 0 −2 = x2 x3 − + ··· 2! 3! 1 + 3x + 1 x2 1 x2 1 + 4x + 9x2 + 1 + 4x + 9x2 + 11 2 13 3 x + x + ··· 2 6 w dx = ln x + 3x + 11 2 13 3 x + x + ··· 4 18 13 3 y2 = uy1 = y1 ln x + x 3x − x2 + x3 + · · · 4 2 46 3 x + ··· 3 46 3 x + ··· 3 10) 4x2 y + (2x2 − 10x)y + (12 − x)y = 0 11) (x2 + 2x)y + (4x + 1)y + 2y = 0 Use Frobenius’ method to solve the following differential equations around origin. Find the roots of the indicial equation, find the recursion relation, and two linearly independent solutions. 12) (x2 + cx)y + [(2 + b)x + c(1 − d)]y + by = 0 (b = 0, c = 0, d is not an integer). 13) x2 y + [(1 − b − d)x + cx2 ]y + [bd + (1 − b)cx]y = 0 (c = 0, b − d is not an integer). 14) x2 y + [(1 − 2d)x + cx2 ]y + (d2 + (1 − d)cx)y = 0 (c = 0) 15) xy + [1 − d + cx2 ]y + 2cxy = 0 (c = 0, d is not an integer).
  • 33. EXERCISES 55 ∞ n=1 (−1)n xn n! · 3 · 5 · 7 · · · (2n + 1) 1 1+ n=1 2) y = c1 (−1)n xn n! · 1 · 3 · 5 · · · (2n − 1) 1 1 + 2x + x2 3 ∞ + c2 x 1 2 4) y = a0 n=0 1+ n=1 6 2 3) y1 = 1−x+ x2 − x3 +· · · , 3 15 ∞ n+b an c (n + 1 − d) b b(b + 1) y1 = 1 − x+ 2 x2 − · · · c(1 − d) c (1 − d)(2 − d) 12) r = 0 ∞ + c2 x 2 (−1)n 3xn 2n n!(2n − 3)(2n − 1)(2n + 1) 3 15 35 3 y2 = x1/2 1 − x + x2 − x + ··· 4 32 128 2 x4 x6 x2 + + + ··· 22 (2 · 4)2 (2 · 4 · 6)2 x2 3x4 11x6 y2 = y1 ln x − − − − ··· 4 8 · 16 64 · 6 · 36 9 27 3 3 6) y1 = x7/3 1 + x + x2 + x + ··· 4 28 280 x2 x3 y 2 = x2 1 + x + + + · · · = x2 e x 2! 3! x x − + · · · = xe−x 2! 3! x2 x3 y2 = xe−x ln x + xe−x x + + + ··· 2 · 2! 3 · 3! 1 1 1 y2 = x3/2 1 + x − x2 + x3 − · · · 2 8 16 ∞ y2 = x 1 + n=1 (−1)n (2x)n 1 · 3 · 5 · · · (2n + 1) ∞ 10) y1 = x2 e−x/2 , ⇒ an+1 = − y2 = x3/2 1 + n=1 (−1)n xn 1 · 3 · 5 · · · (2n − 1) an+1 = − ⇒ an = − n+b+d an c (n + 1) d+b (d + b)(d + b + 1) 2 1− x+ x − ··· c 2! c2 y 2 = xd c an−1 n+b−d c2 c x+ x2 − · · · 1+b−d (1 + b − d)(2 + b − d) c r = d ⇒ an = − an−1 n c2 2 c3 3 y2 = xd 1 − c x + x − x + · · · = xd e−cx 2! 3! y 1 = xb 1 − c an−1 n c3 c2 1 − c x + x2 − x3 + · · · 2! 3! 14) r = d (double root) an = − y 1 = xd 3 7) y1 = x 1 − x + 8) y1 = 1 − 4x − 8x2 , r=d 2 OR y = c1 ex + c2 e−x 5) y1 = 1 + 2 ⇒ 13) r = b ∞ x4n x4n+2 + a2 , (2n)! (2n + 1)! n=0 9) y1 = x1/2 e−x , CHAPTER 7. FROBENIUS’ METHOD 8 11) y1 = 1 − 2x + 2x2 − x3 + · · · 5 5 35 2 105 3 1/2 1− x+ y2 = x x − x + ··· 4 32 128 Answers 1) y = c1 x 1 + 56 y2 = xd e−cx ecx dx x y2 = xd e−cx ln x + xd e−cx cx + 15) r = 0 ⇒ = xd e−cx an+2 = − c2 2 c3 3 x + x + ··· 2 · 2! 3 · 3! c an (n + 2 − d) c c2 c3 x2 + x4 − x6 + · · · 2−d (2 − d)(4 − d) (2 − d)(4 − d)(6 − d) c r = d ⇒ an+2 = − an n+2 c c2 4 c3 y 2 = xd 1 − x2 + x − x6 + · · · 2 2·4 2·4·6 y1 = 1 −
  • 34. 58 CHAPTER 8. LAPLACE TRANSFORM I Example 8.1 Evaluate the Laplace transform of the following functions: a) f (t) = 1 ∞ 0 Chapter 8 0 1 = , s s>0 b) f (t) = eat ∞ Laplace Transform I 0 c) f (t) = Laplace transform provides an alternative method for many equations. We first transform the differential equation to an algebraic equation, then solve it, and then make an inverse transform. Laplace transform has a lot of interesting properties that make these operations easy. In this chapter, we will see the definition and the basic properties. We will also compare this method to the method of undetermined coefficients, and see in what ways Laplace transform is more convenient. 0 1 if if ∞ 1 ∞ e−st f (t)dt (8.1) 0 then, the inverse transform will be f (t) = L−1 {F (s)} (8.2) Note that we use lowercase letters for functions and capital letters for their transforms. 57 = 0 1 , s>a s−a ∞ = 1 e−s , s s>0 d) f (t) = t ∞ te−st dt L {t} = 0 Using integration by parts, we obtain L {t} = − The Laplace transform of a function f (t) is defined as: e−st −s e−st dt = L {f } = Definition, Existence and Inverse of Laplace Transform ∞ 0<t<1 1 t L {t} = −t F (s) = L {f (t)} = e(a−s)t a−s eat e−st dt = L eat = 8.1 ∞ e−st −s e−st dt = L {1} = e−st s ∞ e−st s2 ∞ ∞ + 0 0 = 0 e−st dt s 1 , s>0 s2 The integral that defines the Laplace transform is an improper integral, it may or may not converge. In the above examples, the transform is defined for a certain range of s. In practice, we can use Laplace transform on most of the functions we encounter in differential equations. The following definitions and the theorem answer the question Which functions have a Laplace transform? Piecewise Continuous Functions: A function f (t) is piecewise continuous on [a, b] if the interval can be subdivided into subintervals [ti , tj ], a = t0 < t1 < t2 · · · < tn = b such that f (t) is continuous on each interval and has finite one-sided limits at the endpoints (from the interior). An example can be seen on Figure 8.1.
  • 35. 8.2. PROPERTIES 59 60 CHAPTER 8. LAPLACE TRANSFORM I • Transform of Derivatives L {f } L {f } L f (n) = sL {f } − f (0) = s2 L {f } − sf (0) − f (0) = sn L {f } − sn−1 f (0) − sn−2 f (0) − · · · − f (n−1) (0) • Transform of Integrals t L f (x) dx = 0 F (s) s Example 8.2 Find the Laplace transform of sin at and cos at. Hint: Use Euler’s formula eix = cos x + i sin x and linearity. Figure 8.1: A piecewise continuous function Exponential Order: f (t) is of exponential order as t → ∞ if there exist real constants M, c, T such that |f (t)| M ect for all t T . In other words, a function is of exponential order if it does not grow faster than ect . Theorem 8.1: If f (t) is of exponential order and piecewise continuous on [0, k] for all k > 0, then its Laplace transform exists for all s > c. For example, all the polynomials have a Laplace transform. The function t2 e does NOT have a Laplace transform. 8.2 Basic Properties of Laplace Transforms It is difficult to evaluate the Laplace transform of each function by performing an integration. Instead of this, we use various properties of Laplace transform. Let L {f (t)} = F (s), then, some basic properties are: (assuming these transforms exists) • Linearity L {af + bg} = aL {f } + bL {g} • Shifting L {eiat } − L {e−iat } 2i 1 1 1 a L {sin at} = − = 2 2i s − ia s + ia s + a2 Similarly, we can show that the transform of f (t) = cos at is s F (s) = 2 s + a2 1 Example 8.3 Find the inverse Laplace transform of F (s) = . (s + 5)2 Hint: Use shifting. sin at = eiat − e−iat 2i We know that L−1 1 s2 L−1 {F (s − a)} = eat f (t) L {sin at} = = t. Therefore 1 (s + 5)2 L−1 = te−5t Example 8.4 Find the Laplace transform of f (t) = t2 . Hint: Use Derivatives. Using L {f } = sL {f } − f (0), we obtain L {2t} 2 = 3 s s Example 8.5 Find the Laplace transform of f (t) = t3 . Hint: Use Integrals. L {2t} = sL t2 − 0 ⇒ L t2 = Using the integral rule, we see that L L eat f (t) = F (s − a) ⇒ L {t2 } 2 = 4 s s 6 L t3 = 4 s t3 3 =
  • 36. 8.3. INITIAL VALUE PROBLEMS 8.3 61 Initial Value Problems Consider the constant-coefficient equation y + ay + by = r(t) (8.3) y(0) = p, y (0) = q 62 CHAPTER 8. LAPLACE TRANSFORM I The only disadvantage is that, sometimes finding the inverse Laplace transform is too difficult. We have to find roots of the polynomial s2 + as + b, which is the same as the characteristic polynomial we would encounter if we were using method of undetermined coefficients. (8.4) with initial values Here y is a function of t (y = y(t)). We can solve it by the method of undetermined coefficients. The method of Laplace transform will be an alternative that is more efficient in certain cases. It also works for discontinuous r(t). Let us evaluate the Laplace transform of both sides. Example 8.6 Solve the initial value problem y + 4y = 0, y(0) = 5, y (0) = 3. Let’s start by finding the transform of the equation. L {y } + 4L {y} = 0 L {y } + aL {y } + bL {y} = L {r(t)} (8.5) Using L {y} = Y (s) and L {r(t)} = R(s) s2 Y − sp − q + a(sY − p) + bY = R (8.6) (s2 + as + b)Y = R + (s + a)p + q s2 Y − 5s − 3 + 4Y = 0 ⇒ (s2 + 4)Y = 5s + 3 5s + 3 Y = 2 s +4 Now, we have to find the inverse transform of Y to obtain y(t). (8.7) Y = y = L−1 R + (s + a)p + q s2 + as + b R + sp + ap + q s2 + as + b (8.8) (8.9) Y = 5s 3 2 + + 4 2 s2 + 4 s2 3 sin 2t 2 Note that we did not first find the general solution containing arbitrary constants. We directly found the result. y(t) = L−1 {Y } = 5 cos 2t + Note that this method can be generalized to higher order equations. The advantages compared to the method of undetermined coefficients are: Example 8.7 Solve the initial value problem • The initial conditions are built in the solution, we don’t need to determine constants after obtaining the general solution. • There is no distinction between homogeneous and nonhomogeneous equations, or single and multiple roots. The same method works in all cases the same way. y − 4y + 3y = 1, y(0) = 0, y (0) = − Transform both sides: L {y − 4y + 3y} = L {1} Use the derivative rule • The function on the right hand side r(t) belongs to a wider class. For example, it can be discontinuous. s2 Y − s.0 + 1 1 − 4(sY − 0) + 3Y = 3 s 1 3
  • 37. 8.3. INITIAL VALUE PROBLEMS Isolate Y 63 3−s 1 1 − = s 3 3s s−3 (s − 1)(s − 3)Y = − 3s 1 1 1 1 = − Y =− 3s(s − 1) 3 s s−1 64 CHAPTER 8. LAPLACE TRANSFORM I (s2 − 4s + 3)Y = f (t) F (s) f (t) F (s) 1 1 s eat − ebt a−b 1 (s − a)(s − b) t 1 s2 aeat − bebt a−b s (s − a)(s − b) n! eat sin bt b (s − a)2 + b2 s−a (s − a)2 + b2 Find the inverse transform y(t) = L−1 {Y } = 1 1 t − e 3 3 As you can see, there’s no difference between homogeneous and nonhomogeneous equations. Laplace transform works for both types in the same way. tn sn+1 eat Example 8.8 Solve the initial value problem y + 4y + 4y = 42te−2t , y(0) = 0, y (0) = 0 1 s−a eat cos bt teat 1 (s − a)2 a s 2 + a2 s s 2 + a2 tn eat −2t L {y } + 4L {y } + 4L {y} = 42L te 1 s2 Y + 4sY + 4Y = 42 · (s + 2)2 42 (s2 + 4s + 4)Y = (s + 2)2 42 Y = (s + 2)4 sin at cos at sinh at cosh at n! (s − a)n+1 a s 2 − a2 s s 2 − a2 42 3 −2t te 3! 2as (s2 + a2 )2 t sinh at 2as (s2 − a2 )2 t cos at s 2 − a2 (s2 + a2 )2 t cosh at s 2 + a2 (s2 − a2 )2 sin at − at cos at y(t) = L−1 {Y (s)} = t sin at 2a3 (s2 + a2 )2 sin at + at cos at 2as2 (s2 + a2 )2 y(t) = 7t3 e−2t If you try the method of undetermined coefficients on this problem, you will appreciate the efficiency of Laplace transforms better. Table 8.1: A Table of Laplace Transforms
  • 38. EXERCISES 65 Exercises 3) f (t) = 2e−t cos2 t 4) f (t) = (t + 1)2 et 5) f (t) = t3 e3t 6) f (t) = 7) f (t) = CHAPTER 8. LAPLACE TRANSFORM I Answers Find the Laplace transform of the following functions: t 2) f (t) = et sin 3t 1) f (t) = cos2 2 t 0<t<a 0 a<t 66 1 0<t<a 0 a<t   t 0<t<a  8) f (t) = 1 a<t<b   0 b<t 1) F (s) = 2) F (s) = 3) F (s) = 4) F (s) = 5) F (s) = 1 s + 2 2s 2s + 2 3 (s − 1)2 + 9 1 s+1 + s + 1 s2 + 2s + 5 2 2 1 + + (s − 1)3 (s − 1)2 s − 1 6 (s − 3)4 1 − e−as s 1 ae−as e−as 7) F (s) = 2 − − 2 s s s −as −as 1−e e − ae−as − e−bs 8) F (s) = + s2 s 9) f (t) = cosh 2t − 2 sinh 2t 6) F (s) = Find the inverse Laplace transform of the following functions: s−4 9) F (s) = 2 s −4 3 10) F (s) = (s − 2)2 6 11) F (s) = s(s + 4) 1 12) F (s) = s(s2 + 9) 1 13) F (s) = 2 s (s + 1) 5s + 1 14) F (s) = 2 s +4 15) F (s) = 1 s+8 1 16) F (s) = (s − a)n Solve the following initial value problems using Laplace transform: 17) y − 2y + y = 0, y(0) = 4, y (0) = −3 18) y − 2y + 2y = 0, y(0) = 0, y (0) = 1 19) y + 2y = 4t2 + 12, y(0) = 4, y (0) = 0 20) y + 6y + 9y = e−3t , y(0) = 0, y (0) = 0 10) f (t) = 3te2t 11) f (t) = (3 − 3e−4t )/2 12) f (t) = (1 − cos 3t)/9 13) f (t) = e−t + t − 1 1 14) f (t) = 5 cos 2t + sin 2t 2 −8t 15) f (t) = e 16) f (t) = tn−1 eat (n − 1)! 17) y(t) = 4et − 7tet 18) y(t) = et sin t 19) y(t) = 4 + 2t2 1 20) y(t) = e−3t t2 2
  • 39. 68 CHAPTER 9. LAPLACE TRANSFORM II Reversing the order of integration, we obtain: ∞ ∞ f (x) g(t − x)e−st dt dx = 0 x Making the substitution u = t − x, we obtain: ∞ ∞ f (x) g(u)e−su−sx dudx L {f ∗ g} = Chapter 9 0 ∞ 0 ∞ 0 0 Laplace Transform II g(u)e−su du f (x)e−sx dx = =F (s) G(s) Example 9.1 Find the inverse Laplace transform of F (s) = In this chapter, we will study more advanced properties of Laplace transform. At the end, we will be able to find transform and inverse transform of a wider range of functions. This will enable us to solve almost any linear constant coefficient equation, including discontinuous inputs. 9.1 L−1 1 s2 1 s+4 = t, L−1 t (9.1) 0 The convolution operation is commutative, in other words f ∗ g = g ∗ f Theorem 9.1: The transform of convolution of two functions is equal to the product of their transforms, i.e. L {f ∗ g} = F (s) · G(s) −1 L {F (s) · G(s)} = f ∗ g where L {f } = F (s) and L {g} = G(s). Proof: Using the definitions of convolution and Laplace transform, t L {f ∗ g} =L f (x) g(t − x) dx 0 ∞ t f (x) g(t − x)e−st dx dt = 0 0 67 L−1 1 1 · 2 s+4 s (9.2) (9.3) = t ∗ e−4t xe−4(t−x) dx 0 xe4x e4x − 4 16 −4t t 1 e = − + 4 16 16 t = e−4t 0 Example 9.2 Find the inverse Laplace transform of F (s) = The convolution of two functions f and g is defined as f (x)g(t − x) dx ⇒ 1 . + 4s2 t f (t) = t ∗ e−4t = Convolution h(t) = (f ∗ g)(t) = = e−4t s3 s . (s2 + 1)2 s 1 · 2 = L {cos t} · L {sin t}, + 1) (s + 1) we will see that f (t) = L−1 {F } = cos t ∗ sin t. If we express F as F (s) = (s2 t cos(x) sin(t − x) dx f (t) = 0 t = 0 1 = 2 = 1 [sin(t − x + x) + sin(t − x − x)] dx 2 t [sin(t) + sin(t − 2x)] dx 0 1 cos(t − 2x) x sin t + 2 2 t 0 1 1 = t sin t + (cos t − cos t) 2 2 1 = t sin t 2
  • 40. 9.2. UNIT STEP FUNCTION 9.2 69 70 CHAPTER 9. LAPLACE TRANSFORM II Unit Step Function ∞ e−st f (t)dt F (s) = The Heaviside step function (or unit step function) is defined as 0 F (s) = ua (t) = u(t − a) = 0 1 if if t<a t a (9.4) This is a simple on off function. It is especially useful to express discontinuous inputs. Theorem 9.2: [t−shifting] Let L {f (t)} = F (s), then (9.5) Proof: Using the definition, ∞ e−st f (t − a) u(t − a) dt ∞ e−st f (t − a) dt a ∞ e−sa−sx f (x) dx = 0 −as =e ( where x = t − a) F (s) Example 9.3 Find the Laplace transform of g(t) = 0 t if if t<5 t 5 We can express g(t) as g(t) = u(t − 5)f (t − 5) where f (t) = (t + 5). Then F (s) = L {f (t)} = 9.3 In other words L {tf (t)} = −F (s) 5 1 + 2 s s ⇒ (9.7) Repeating this procedure n times, we obtain: dn F (s) dsn Using the derivative formula, we find L {t sin t} = − = f (t)dt 0 Example 9.4 Find the Laplace transform of f (t) = t sin t. L {f (t − a) u(t − a)} = e−as F (s) 0 (−t)e −st L {tn f (t)} = (−1)n Figure 9.1: u(t − a) and its effect on f (t) L {f (t − a) u(t − a)} = (9.6) ∞ L {g(t)} = e−5s 1 5 + 2 s s Differentiation of Transforms If f (t) is piecewise continuous and of exponential order, then we can differentiate its Laplace transform integral. d ds 1 1 + s2 = 2s (1 + s2 )2 (9.8)
  • 41. 9.4. PARTIAL FRACTIONS EXPANSION 9.4 71 Partial Fractions Expansion CHAPTER 9. LAPLACE TRANSFORM II 9.5 In many applications of Laplace transform, we need to expand a rational function in partial fractions. Here, we will review this technique by examples. 2x + 1 A B C = + + (x − 2)(x + 3)(x − 1) x−2 x+3 x−1 2 x + 4x − 5 B C A D + + = + (x − 2)(x − 1)3 x − 2 x − 1 (x − 1)2 (x − 1)3 x3 + 1 Dx + E A Bx + C + 2 + 2 = 2 + 4)2 x(x x x +4 (x + 4)2 3 A B x3 − 4x2 + x + 9 = x+1+ 2 =x+1+ + x2 − 5x + 6 x − 5x + 6 x−2 x−3 • We can express any polynomial as a product of first and second order polynomials. • For second order polynomials in the expansion, we have to use Ax + B (not simply a constant) in the numerator. • If numerator’s degree is greater or equal to the denominator, we should first divide them using polynomial division. 2 Example 9.5 Find the inverse Laplace transform of F (s) = 72 −s + 7s − 1 . (s − 2)(s − 5)2 Applications Now we are in a position to solve a wider class of differential equations using Laplace transform. Example 9.6 Solve the initial value problem y − 6y + 8y = 2e2t , y(0) = 11, y (0) = 37 We will first find the Laplace transform of both sides, then find Y (s) L {y } − 6L {y } + 8L {y} = L 2e2t s2 Y − 11s − 37 − 6(sY − 11) + 8Y = (s2 − 6s + 8)Y = −s2 + 7s − 1 = A(s − 5)2 + B(s − 2)(s − 5) + C(s − 2) Inserting s = 2, we see that 9 = 9A ⇒ A = 1. Inserting s = 5, we see that 9 = 3C ⇒ C = 3. The coefficient of s2 : A + B = −1 therefore B = −2. So 1 −s2 + 7s − 1 2 3 = − + 2 (s − 2)(s − 5) s − 2 s − 5 (s − 5)2 Now we can easily find the inverse Laplace transform: L−1 {F (s)} = e2t − 2e5t + 3te5t 2 + 11s − 29 s−2 The factors of s2 − 6s + 8 are (s − 2) and (s − 4), so Y = 2 11s − 29 + (s − 2)(s − 2)(s − 4) (s − 2)(s − 4) Y = First, we have to express F (s) in terms of simpler fractions: −s2 + 7s − 1 A B C = + + (s − 2)(s − 5)2 s − 2 s − 5 (s − 5)2 2 s−2 11s2 − 51s + 60 (s − 2)2 (s − 4) Now we need to find the inverse Laplace transform. Using partial fractions expansion Y = A C B + + 2 s − 2 (s − 2) s−4 After some algebra we find that A = 3, B = −1, C = 8 so Y (s) = 8 1 3 − + 2 s − 2 (s − 2) s−4 y(t) = L−1 {Y (s)} = 3e2t − te2t + 8e4t
  • 42. 9.5. APPLICATIONS 73 Example 9.7 Solve the initial value problem 74 CHAPTER 9. LAPLACE TRANSFORM II Example 9.8 Solve the initial value problem y + y = f (t), y(0) = 0, y (0) = 3 where f (t) = 0 2 cos t if if 0 < t < 5π 5π < t y + 2y + y = r(t), y(0) = 0, y (0) = 0 where r(t) = As you can see, the input function is discontinuous, but this makes no difference for Laplace transform. L {y } + L {y} = L {f } t 0 if if 0<t<1 1<t Once again we have a discontinuous input. This time we will use unit step function. First, we have to express r(t) with a single formula. r(t) = t − u(t − 1)t = t − u(t − 1)(t − 1) − u(t − 1) 2 s Y −3+Y =F F +3 Y = 2 s +1 1 Using the fact that L {sin t} = 2 , we can obtain y(t) by convolution: s +1 y(t) = L−1 {Y } = f (t) ∗ sin t + 3 sin t Its Laplace transform is R(s) = L {r(t)} = Finding the Laplace transform of the equation, we obtain (s2 + 2s + 1)Y = R Using the definition of convolution, t f ∗ sin t = Y = f (x) sin(t − x) dx 0 If t < 5π, f = 0 therefore this integral is also zero. If t > 5π we have Y = t f ∗ sin t = 1 e−s e−s − 2 − 2 s s s 2 cos x sin(t − x) dx s2 (s R (s + 1)2 1 e−s − 2 2 + 1) s (s + 1) Using partial fractions expansion 5π Using the trigonometric identity 2 sin A cos B = sin(A + B) + sin(A − B) we obtain t f ∗ sin t = sin t + sin(t − 2x) dx 2 1 2 1 1 1 1 Y =− + 2 + + − e−s − + 2 + s s s + 1 (s + 1)2 s s s+1 Using the fact that L−1 {e−as F (s)} = f (t − a)u(t − a), we obtain 5π = x sin t + cos(t − 2x) 2 t 5π = (t − 5π) sin t Therefore y(t) = −2 + t + 2e−t + te−t − u(t − 1) −1 + (t − 1) + e−(t−1) We know that u(t − 1) = 0 for t > 1 and u(t − 1) = 1 for t > 1 so y(t) = y(t) = 3 sin t (t − 5π + 3) sin t if if 0 < t < 5π 5π < t −2 + t + 2e−t + te−t (2 − e)e−t + te−t if if 0<t<1 1<t
  • 43. EXERCISES 75 Exercises 1) F (s) = 3 s+3 (s2 + 4)2 8) F (s) = 2 2) F (s) = s s4 + 4a4 10) F (s) = sin 4t − 4t cos 4t 128 4t sin 2t + 3 sin 2t − 6t cos 2t 7) f (t) = 16 1 t 9) f (t) = sin 2t + cos 2t 4 2 5) f (t) = 6) f (t) = 10) f (t) = e2t + 2 cos 3t + 12) y = 3 cos t + (4 + t) sin t 13) y = −25 + 8t2 Solve the following initial value problems : (where y = y(t)) 1 −t e − et cos t + 7et sin t 5 15) y = t − sin t 14) y = 11) y − y − 2y = 0, y(0) = 8, y (0) = 7 12) y + y = 2 cos t, y(0) = 3, y (0) = 4 13) y + 0.64y = 5.12t2 , y(0) = −25, y (0) = 0 1 0 < t < 2π cos t 2π < t 16) y = 14) y − 2y + 2y = e−t , y(0) = 0, y (0) = 1 15) y + y = t, y(0) = 0, y (0) = 0 16) y + y = r(t), y(0) = 1, y (0) = 0 where r(t) = 1 0 if if 0 < t < 2π 2π < t 5 0 if if 17) y + y = e−2t sin t, y(0) = 0, y (0) = 0 18) y +2y +5y = r(t), y(0) = 0, y (0) = 0 where r(t) = 0<t<π π<t 19) 4y + 4y + 17y = g(t), y(0) = 0, y (0) = 0 sin t 0 if if 0 < t < 3π 1 7 , y(0) = , y (0) = − 50 50 3π < t 1 1 17) y = (sin t − cos t) + e−2t (sin t + cos t) 8 8    1 − e−t cos 2t + sin 2t , 0 < t < π  2 18) y =  −t π  e (e − 1) cos 2t + sin 2t π<t  2 19) y = 1 8    t 1 e− 2 (t−x) sin 2(t − x)g(x) dx 0 1 (cos t − 7 sin t) 50 20) y = 1 −9π 3t 2   e e − e6π e−2t 50 50 1 e−t e−3t − + 3 2 6 8) f (t) = cosh at cos at 11) y = 3e−t + 5e2t 3s − 2s + 5 (s − 2)(s2 + 9) 12s2 − 16 (s2 + 4)3 4) f (t) = u(t − 1) cos(2t − 2) 2 s (s2 + 4)2 20) y − y − 6y = s2 + 2s (s2 + 2s + 2)2 3) f (t) = u(t − 3) sin(t − 3) Find the inverse Laplace transform transform of the following functions: se−s e−3s 4) F (s) = 2 3) F (s) = 2 s +1 s +4 1 1 5) F (s) = 2 6) F (s) = 3 2 (s + 16) s + 4s2 + 3s 9) F (s) = CHAPTER 9. LAPLACE TRANSFORM II Answers Find the Laplace transform transform of the following functions: 1) f (t) = te−t cos t 2) f (t) = t2 sin 2t 7) F (s) = 76 if 0 < t < 3π if 3π < t 2 sin 3t 3
  • 44. 78 CHAPTER 10. FOURIER ANALYSIS I L cos nπx mπx cos dx = 0 (m = n) L L (10.3) sin nπx mπx sin dx = 0 (m = n) L L (10.4) −L L Chapter 10 −L L cos2 Fourier Analysis I −L −L nπx mπx sin dx = 0 (for all m, n) L L 77 kπx dx = L L a0 cos −L ∞ + + (10.1) (10.2) kπx dx L L cos nπx kπx cos dx L L sin an nπx kπx cos dx L L −L n=1 ∞ ∞ If possible, this expansion would be very useful in all kinds of applications. Once we solve a question for sine and cosine functions, we will be able to solve it for any periodic f . Here, an and bn are the coordinates of f in the space of sine and cosine functions. But then how can we find an and bn ? The following identities will help us: (10.6) Now, suppose the expansion (10.1) exists. To find ak , we will multiply both sides by cos kπx and then integrate from −L to L. L L nπx nπx f (x) = a0 + an cos + bn sin L L n=1 n=1 −L (10.5) 2 cos A sin B = sin(A + B) − sin(A − B) Let f (x) be a periodic function with period 2L. It is sufficient that f be defined on [−L, L]. Is it possible to express f as a linear combination of sine and cosine functions? cos −L nπx dx = L L 2 sin A sin B = cos(A − B) − cos(A + B) f (x) cos L sin2 2 cos A cos B = cos(A − B) + cos(A + B) Fourier Series ∞ L In the terminology of linear algebra, the trigonometric functions form an orthogonal coordinate basis. We can easily prove these formulas if we remember the following trigonometric identities: The trigonometric functions sine and cosine are the simplest periodic functions. If we can express an arbitrary periodic function in terms of these, many problems would be simplified. In this chapter, we will see how to find the Fourier series of a periodic function. Fourier series is important in many applications. We will also need them when we solve partial differential equations. 10.1 nπx dx = L L bn n=1 −L (10.7) Using the property of orthogonality, we can see that all those integrals are zero, except the kth one. Therefore L f (x) cos −L kπx dx = ak L L ⇒ ak = 1 L L f (x) cos −L kπx dx L (10.8) We can apply the same procedure to find a0 and bn . In the end, we will obtain the following formulas for a function f defined on [−L, L].
  • 45. 10.1. FOURIER SERIES 79 a0 = Fourier coefficients: an = bn = 1 2L 1 L 1 L CHAPTER 10. FOURIER ANALYSIS I L f (x) dx −L L f (x) cos nπx dx L f (x) sin nπx dx L −L L −L (10.9) ∞ ∞ Fourier series: f (x) = a0 + 80 an cos n=1 nπx nπx + bn sin L L n=1 (10.10) Example 10.1 Find the Fourier series of the periodic function f (x) = x2 , −L x L having period= 2L. a0 = = 1 2L L x2 dx −L 3 L 1 x 2L 3 = −L L2 3 Using integration by parts two times we find: an = = bn = 1 L L x2 cos −L nπx dx L 4L2 cos nπ n2 π 2 1 L L x2 sin −L nπx dx = 0 L Therefore the Fourier series is: ∞ x2 = 4L2 nπx L2 + (−1)n 2 2 cos 3 nπ L n=1 The plot of the Fourier series up to n = 1, 2 and 3 is given in Figure 10.1. Figure 10.1: Fourier Series of f = x2 for n = 1, 2, 3
  • 46. 10.2. CONVERGENCE OF FOURIER SERIES 10.2 81 82 CHAPTER 10. FOURIER ANALYSIS I Convergence of Fourier Series Like any infinite series, Fourier series is of no use if it is divergent. But most functions that we are interested in have Fourier series that converge and converge to the function. Theorem 10.1: Let f be periodic with period 2L and let f and f be piecewise continuous on the interval [−L, L]. Then the Fourier expansion of f converges to: • f (x) if f is continuous at x. f (x+ ) + f (x− ) if f is discontinuous at x. 2 Example 10.2 Find the Fourier series of the periodic function • a b f (x) = Figure 10.2: Convergence at a discontinuity 10.3 Theorem 10.2: Let f be continuous on [−L, L], f (L) = f (−L) and let f be piecewise continuous. Then the Fourier coefficients of f satisfy: ∞ 2a2 + 0 having period= 2L. Then evaluate the series at x = L. a0 = 1 2L a dx + −L 1 2L L b dx = 0 a+b 2 L 1 L f (x)2 dx ∞ f 2 (x) = a0 f (x) + ∞ L f 2 (x) dx = a0 f (x) dx + −L ∞ n=1 an cos nπx + L bn = 1 L 0 nπx 1 a cos dx + L L −L 0 a sin −L nπx 1 dx + L L a L nπx =− cos L nπ L 0 L 0 n=1 ∞ L an f (x) cos −L nπx bn dx + L n=1 b sin 0 L 0 b−a = (1 − (−1)n ) nπ Therefore the Fourier series is: ∞ a+b b−a nπx + [1 − (−1)n ] sin f (x) = 2 nπ L n=1 a + b 2(b − a) + 2 π πx 1 3πx 1 5πx + sin + sin + ··· L 3 L 5 L a+b If we insert x = L in that series, we obtain f (L) = . Thus the value at 2 discontinuity is the average of left and right limits. The summation of the series up to n = 1, 5 and 9 is plotted on Figure 10.2. = f (x) sin −L 1 1 1 = 1 + 4 + 4 + ··· 4 n 2 3 n=1 2 (Hint: Use the Fourier series of f (x) = x on the interval −π < x < π) Example 10.3 Find the sum of the series S = nπx dx L b L nπx − cos L nπ L −L L Using equation (10.9) to evaluate these integrals, we can obtain the result. nπx b cos dx = 0 L L sin nπx . L nπx nπx + bn f (x) sin L L n=1 ∞ 1 an = L ∞ n=1 bn ∞ an f (x) cos n=1 L (10.11) −L Proof: We can express f (x) as f (x) = a0 + Now multiply both sides by f and integrate −L 0 (a2 + b2 ) = n n n=1 −L < x < 0 0<x<L if if Parseval’s Identity Evaluating the integrals in (10.9) for f (x) = x2 we obtain π2 4(−1)n and bn = 0 so a0 = , a n = 3 n2 f (x) = 1 π2 1 − 4 cos x − cos 2x + cos 3x − · · · 3 4 9 Using Parseval’s theorem, we have sin 1 2π 4 1 + 16 1 + 4 + 4 + · · · 9 2 3 Therefore 1 π 4 x dx π −π 2 = π4 5 = nπx d L
  • 47. 10.3. PARSEVAL’S IDENTITY 1 1 2 2 − 16 1 + 4 + 4 + · · · = π 4 2 3 5 9 1 1 π4 S = 1 + 4 + 4 + ··· = 2 3 90 83 84 CHAPTER 10. FOURIER ANALYSIS I Exercises Find the Fourier series of the periodic function f (x) defined on the given interval 1) f (x) = x, −π < x < π 3) f (x) = 0 1 2) f (x) = x, 0 < x < 2π −π < x < 0 0<x<π if if 5) f (x) = sin2 x, −π < x < π 7) f (x) = −π/4 π/4 if if −1 < x < 0 0<x<1 9) f (x) = |x|, −2 < x < 2 11) f (x) = x 1−x if if 4) f (x) = x2 , 0 < x < 2π 6) f (x) = x + |x|, −π < x < π 8) f (x) = π x if if 10) f (x) = | sin x|, −π < x < π 0<x<1 1<x<2 13) f (x) = ax + b, −L < x < L 15) f (x) = x3 , −π < x < π 12) f (x) = −a a 1 π2 1 + + ··· = . 9 25 8 −L < x < 0 0<x<L 16) f (x) = ex , −π < x < π x sin ax cos ax + a a2 x cos ax sin ax + x sin ax dx = − a a2 2 x sin ax 2x cos ax 2 sin ax x2 cos ax dx = + − a a2 a3 2 x cos ax 2x sin ax 2 cos ax x2 sin ax dx = − + + a a2 a3 x cos ax dx = if if 14) f (x) = 1 − x2 , −1 < x < 1 17) Using integration by parts, show that: 18) Show that 1 + −π < x < 0 0<x<π
  • 48. EXERCISES 85 Answers 1 1 sin 2x + sin 3x − · · · 2 3 2) f (x) = π − 2 sin x + 1 1 sin 2x + sin 3x + · · · 2 3 1 1 sin x + sin 3x + sin 5x + · · · 3 5 4π 2 1 1 + 4 cos x + cos 2x + cos 3x + · · · 3 4 9 −4π sin x + 5) f (x) = 6) f (x) = π 4 − 2 π 1 1 sin 2x + sin 3x + · · · 2 3 1 1 − cos 2x 2 2 cos x + +2 sin x − 7) f (x) = sin πx + 1 1 cos 3x + cos 5x + · · · 9 25 1 1 1 sin 2x + sin 3x − sin 4x + · · · 2 3 4 1 1 sin 3πx + sin 5πx + · · · 3 5 ∞ 3π (−1)n − 1 1 + cos nx − sin nx 2 4 πn n n=1 9) f (x) = 1 − 10) f (x) = 8 π2 cos 2 4 − π π 11) f (x) = − + 4 π2 2 π sin 13) f (x) = b + 2aL π 14) f (x) = 2 4 + 2 3 π πx 1 3πx 1 5πx + sin + sin + ··· L 3 L 5 L sin ∞ n=1 3πx 1 5πx πx 1 + cos + cos + ··· 2 9 2 25 2 cos 2nx 4n2 − 1 cos πx + sin πx + 1 1 cos 3πx + cos 5πx + · · · 9 25 1 sin 3πx + · · · 3 (−1)n+1 15) f (x) = 2 n=1 πx 1 2πx 1 3πx − sin + sin − ··· L 2 L 3 L cos πx − ∞ ∞ 1 − (−1)n 1 2 1 sin nx = + 3) f (x) = + 2 n=1 nπ 2 π 8) f (x) = CHAPTER 10. FOURIER ANALYSIS I 4a π 12) f (x) = 1) f (x) = 2π sin x − 4) f (x) = 86 1 1 cos 2πx + cos 3πx + · · · 4 9 (nπ)2 − 6 sin nx n3 ∞ 16) f (x) = 2 sinh π 1 (−1)n + (cos nx − n sin nx) π 2 n=1 1 + n2 18) Use the function in exercise 12 in Parseval’s identity
  • 49. 88 CHAPTER 11. FOURIER ANALYSIS II Figure 11.1: Plots of Some Even and Odd Functions Chapter 11 Fourier Analysis II In this chapter, we will study more advanced properties of Fourier series. We will find the even and odd periodic extensions of a given function, we will express the series using complex notation and finally, we will extend the idea of Fourier series to nonperiodic functions in the form of a Fourier integral. As you can see in Figure 11.1, an even function is symmetric with respect to y−axis, an odd function is symmetric with respect to origin. Half Range Extensions: Let f be a function defined on [0, L]. If we want to expand it in terms of sine and cosine functions, we can think of it as periodic with period 2L. Now we need to define f on the interval [−L, 0]. There are infinitely many possibilities, but for simplicity, we are interested in making f an even or an odd function. If we define f for negative x values as f (x) = f (−x), we obtain the even periodic extension of f , which is represented by a Fourier cosine series. If we define f for negative x values as f (x) = −f (−x), we obtain the odd periodic extension of f , which is represented by a Fourier sine series. Half-Range Cosine Expansion: (or Fourier cosine series) ∞ f (x) = a0 + 11.1 n=1 Fourier Cosine and Sine Series If f (−x) = f (x), f is an even function. If f (−x) = −f (x), f is an odd function. We can easily see that, for functions: even × even = even, odd × odd = even, even × odd = odd For example |x|, x2 , x4 , cos x, cos nx, cosh x are even functions. x, x3 , sin x, sin nx, sinh x are odd functions. ex is neither even nor odd. L L If f is even: f (x) dx f (x) dx = 2 −L L If f is odd: (11.1) 0 f (x) dx = 0 (11.2) −L Using the above equations, we can see that in the Fourier expansion of an even function, bn = 0, and in the expansion of an odd function, an = 0. This will cut our work in half if we can recognize the given function as odd or even. 87 an cos where a0 = 1 L nπx , (0 < x < L) L L f (x) dx, 0 an = 2 L L f (x) cos 0 nπx dx L (11.3) (11.4)
  • 50. 11.1. FOURIER COSINE AND SINE SERIES 89 ∞ bn sin n=1 nπx , (0 < x < L) L CHAPTER 11. FOURIER ANALYSIS II 11.2 Half-Range Sine Expansion: (or Fourier sine series) f (x) = 90 (11.5) Complex Fourier Series Consider the Fourier series of f (x): ∞ ∞ f (x) = a0 + where L 2 L bn = f (x) sin 0 nπx dx L 0 f (x) = bn sin nx (11.7) n=1 Using Euler’s formula eix = cos x + i sin x we can express the sine and cosine functions as: 0<x< π 2 π <x<π 2 if if π 2 n=1 (11.6) Example 11.1 Find the half-range cosine and sine expansions of an cos nx + cos nx = einx + e−inx , 2 sin nx = einx − e−inx 2i (11.8) an + ibn 2 (11.9) Therefore Here, L = π, therefore 1 a0 = π 2 π an = π π 2 π π 2 sin nx = n π 2 einx + e−inx If we define c0 = a0 and π cos nx dx 2 π an − ibn 2 an cos nx + bn sin nx = π π dx = 2 4 cn = nπ 2 sin =− n an + ibn an − ibn , c−n = , n = 1, 2, 3, . . . 2 2 (11.10) We will obtain ∞ cn einx (11.11) f (x)e−inx dx n = 0, ±1, ±2, . . . (11.12) f (x) = Therefore half-range cosine series of f is n=−∞ ∞ sin nπ π π 1 1 2 f (x) = − cos nx = − cos x − cos 3x + cos 5x − · · · 4 n=1 n 4 3 5 where cn = On the other hand, 2 bn = π π π 2 π sin nx dx 2 − cos nx = n π π 2 cos nπ − cos nπ 2 = n Therefore half-range sine series of f is ∞ f (x) = n=1 cos nπ − cos nπ 1 1 2 sin nx = sin x − sin 2x + sin 3x + sin 5x + · · · n 3 5 1 2π π −π For a function of period 2L we have ∞ cn einπx/L , f (x) = cn = n=−∞ 1 2L L f (x)e−inπx/L dx −L Example 11.2 Find the complex Fourier series of f (x) = x if −π < x < π and f (x + 2π) = f (x). We have to evaluate the integral cn = 1 2π π xe−inx dx −π (11.13)
  • 51. 11.2. COMPLEX FOURIER SERIES 91 For n = 0 this integral is zero, so we have c0 = 0. For n = 0 cn = −inx π xe −in 1 2π π − −π inπ −π 1 2π Therefore CHAPTER 11. FOURIER ANALYSIS II 11.3 Fourier Integral Representation −inx e dx −in πe−inπ + πe −0 −in 1 einπ + e−inπ cos nπ =− =− in 2 in i n = (−1) n = 92 In this section, we will apply the basic idea of the Fourier series to nonperiodic functions. Consider a periodic function with period= 2L and its Fourier series. In the limit L → ∞, the summation will be an integral, and f will be a nonperiodic function. Then we will obtain the Fourier integral representation: ∞ i (−1)n einx , n n=−∞ n=0 where A(u) = Note that we can obtain the real Fourier series from the complex one. If we add nth and −nth terms we get cos nx + i sin nx cos(−nx) + i sin(−nx) sin nx i(−1) + i(−1)−n = (−1)n+1 n −n n B(u) = n ∞ n+1 sin nx (−1) x= n n=1 1 π 1 π ∞ f (x) cos ux dx (11.15) f (x) sin ux dx (11.16) −∞ ∞ −∞ Like the Fourier series, we have A(u) = 0 for odd functions and B(u) = 0 for even functions. Theorem 11.1: If f and f are piecewise continuous in every finite interval ∞ This is the real Fourier series. |f | dx is convergent, then the Fourier integral of f converges to: and if −∞ Example 11.3 Find the complex Fourier series of f (x) = k • f (x) if f is continuous at x. π 1 ke−inx dx 2π −π π k e−inx = (n = 0) 2π −in −π cn = inπ • f (x+) + f (x−) if f is discontinuous at x. 2 Example 11.4 Find the Fourier integral representation of −inπ k e −e nπ 2i k = sin nπ nπ =0 = If n = 0 we have (11.14) 0 ∞ x= [A(u) cos ux + B(u) sin ux] du f (x) = 1 c0 = 2π =k f (x) = π/2 0 if if |x| < 1 1 < |x| Note that f is even therefore B(u) = 0 π k dx A(u) = −π ∞ 1 π f (x) cos ux dx = −∞ 1 = cos ux dx = 0 sin ux u 1 π 1 = 0 1 −1 π cos ux dx 2 sin u u
  • 52. 11.3. FOURIER INTEGRAL REPRESENTATION 93 Therefore, Fourier integral representation of f is ∞ f (x) = 0 sin u cos ux du u 94 CHAPTER 11. FOURIER ANALYSIS II Exercises For the following functions defined on 0 < x < L, find the half-range cosine and half-range sine expansions: Example 11.5 Prove the following formulas using two different methods: eax e cos bx dx = 2 (a cos bx + b sin bx) a + b2 eax eax sin bx dx = 2 (a sin bx − b cos bx) a + b2 We can obtain the formulas using integration by parts, but this is the long way. A better method is to express the integrals as a single complex integral using eibx = cos bx + i sin bx, then evaluate it at one step, and then separate the real and imaginary parts. ax Example 11.6 Find the Fourier integral representation of f (x) = −ex cos x e−x cos x if if x<0 0<x This function is odd therefore A(u) = 0. 2 ∞ −x 1 ∞ f (x) sin ux dx = e cos x sin ux dx π −∞ π 0 2 ∞ −x sin(ux + x) + sin(ux − x) e dx = π 0 2 ∞ 1 e−x = [− sin(u + 1)x − (u + 1) cos(u + 1)x] π 1 + (u + 1)2 0 ∞ e−x 1 [− sin(u − 1)x − (u − 1) cos(u − 1)x] + π 1 + (u − 1)2 0 1 u−1 u+1 = + π 1 + (u + 1)2 1 + (u − 1)2 2 u3 = π u4 + 4 B(u) = So f (x) = 2 π ∞ 0 u3 sin ux du +4 u4 1) f (x) = 2kx/L 2k(L − x)/L if if 0 < x < L/2 2) f (x) = ex L/2 < x < L 3) f (x) = k 4) f (x) = x4 5) f (x) = cos 2x 0 < x < π 6) f (x) = 0 k if if 0 < x < L/2 L/2 < x < L Find the complex Fourier series of the following functions: 7) f (x) = 0 1 if if −π < x < 0 0<x<π 9) f (x) = sin x 8) f (x) = x2 , −L < x < L 10) f (x) = cos 2x Find the Fourier integral representations of the following functions:  π π  cos x, |x| <  π − x, 0 < x < π 2 2 11) f (x) = (f odd) 12) f (x) = π  0, π<x  0, |x| > 2 13) f (x) = e−x , 0 < x ex , x < 0 π 0 14) f (x) = Prove the following formulas. (Hint: Define a suitable then find its Fourier integral representation.)   πx2 /2,    ∞ 2 cos ux 2 15) 1 − 2 sin u + cos u du = π/4,  u u u 0    0,   0, x<0  ∞ cos ux + u sin ux 16) du = π/2, x = 0  −x 1 + u2 0  πe , x > 0 if if 0<x<1 Otherwise function f and 0 x<1 x=1 1<x